This paper was converted on www.awesomepapers.org from LaTeX by an anonymous user.
Want to know more? Visit the Converter page.

\AtAppendix

An Asymptotic Orthogonality Relation for 𝐆𝐋(𝒏,)\operatorname{GL}(n,\mathbb{R})

Dorian Goldfeld and Eric Stade and Michael Woodbury Department of Mathematics
Columbia University
2990 Broadway
New York, NY 10027, USA
[email protected] Department of Mathematics
University of Colorado, Boulder
Colorado 80309, USA
[email protected] Department of Mathematics
Rutgers, The State University of New Jersey
110 Frelinghuysen Rd
Piscataway, NJ 08854-8019, USA
[email protected]
Abstract.

Orthogonality is a fundamental theme in representation theory and Fourier analysis. An orthogonality relation for characters of finite abelian groups (now recognized as an orthogonality relation on GL(1)) was used by Dirichlet to prove infinitely many primes in arithmetic progressions. Asymptotic orthogonality relations for GL(n)(n), with n3n\leq 3, and applications to number theory, have been considered by various researchers over the last 45 years. Recently, the authors of the present work have derived an explicit asymptotic orthogonality relation, with a power savings error term, for GL(4,)(4,\mathbb{R}). Here we we extend those results to GL(n,)(n,\mathbb{R}) (n2)(n\geq 2).

For n5n\leq 5, our results are contingent on the Ramanujan conjecture at the infinite place, but otherwise are unconditional. In particular, the case n=5n=5 represents a new result. The key new ingredient for the proof of the case n=5n=5 is the theorem of Kim-Shahidi that functorial products of cusp forms on GL(2)×\timesGL(3) are automorphic on GL(6). For n>5n>5 (assuming again the Ramanujan conjecture holds at the infinite place), our results are conditional on two conjectures, both of which have been verified in various special cases. The first of these conjectures regards lower bounds for Rankin-Selberg L-functions, and the second concerns recurrence relations for Mellin transforms of GL(n,)(n,\mathbb{R}) Whittaker functions.

Central to our proof is an application of the Kuznetsov Trace formula, and a detailed analysis, utilizing a number of novel techniques, of the various entities—Hecke-Maass cusp forms, Langlands Eisenstein series, spherical principal series Whittaker functions and their Mellin transforms, and so on—that arise in this application.

Dorian Goldfeld is partially supported by Simons Collaboration Grant Number 567168.

1. Introduction

1.1. Brief description of the main result of this paper

Let n1n\geq 1 be a rational integer, ss\in\mathbb{C}, and 𝔸=×𝔸f\mathbb{A}_{\mathbb{Q}}=\mathbb{R}\times\mathbb{A}_{f} denote the ring of adeles over \mathbb{Q} where 𝔸f\mathbb{A}_{f} denotes the finite adeles. The family of unitary cuspidal automorphic representations π\pi of GL(n,𝔸)\text{\rm GL}(n,\mathbb{A}_{\mathbb{Q}}) and their standard L-functions

L(s,π)=L(s,π)pLp(s,π)L(s,\pi)=L_{\infty}(s,\pi)\cdot\prod_{p}L_{p}(s,\pi)

were first introduced by Godement and Jacquet [GJ72] and have played a major role in modern number theory. In the special case of n=1n=1 the Euler products pLp(s,π)\prod_{p}L_{p}(s,\pi) are just Dirichlet L-functions.

In this paper we focus on the unitary cuspidal automorphic representations of GL(n,𝔸)\text{\rm GL}(n,\mathbb{A}_{\mathbb{Q}}) with trivial central character which are globally unramified. For n2,n\geq 2, these can be studied classically in terms of Hecke-Maass cusp forms on

SL(n,)\GL(n,)/(O(n,)×)\text{\rm SL}(n,\mathbb{Z})\backslash\text{\rm GL}(n,\mathbb{R})/\left(\text{\rm O}(n,\mathbb{R})\cdot\mathbb{R}^{\times}\right)

where

𝔥n:=GL(n,)/(O(n,)×)\mathfrak{h}^{n}:=\text{\rm GL}(n,\mathbb{R})/\left(\text{\rm O}(n,\mathbb{R})\cdot\mathbb{R}^{\times}\right)

is a generalization of the classical upper half-plane. In fact 𝔥2:={(yx01)|y>0,x}\mathfrak{h}^{2}:=\left\{\left(\begin{smallmatrix}y&x\\ 0&1\end{smallmatrix}\right)\big{|}\;y>0,\,x\in\mathbb{R}\right\} is isomorphic to the classical upper half-plane. For n2n\geq 2, Hecke-Maass cusp forms are smooth functions ϕ:𝔥n\phi:\mathfrak{h}^{n}\to\mathbb{C} which are automorphic for SL(n,)\text{\rm SL}(n,\mathbb{Z}) with moderate growth and which are joint eigenfunctions of the full ring of invariant differential operators on GL(n,)\text{\rm GL}(n,\mathbb{R}) and are also joint eigenfunctions of the Hecke operators. Such globally unramified Hecke-Maass forms can be classified in terms of Langlands parameters which (assuming the cusp form is tempered) are nn pure imaginary numbers (α1,α2,,αn)(i)n(\alpha_{1},\alpha_{2},\ldots,\alpha_{n})\in\mathbb{(}i\cdot\mathbb{R})^{n} that sum to zero. Further, the Hecke-Maass cusp forms ϕ\phi with Langlands parameters (α1,,αn)(\alpha_{1},\ldots,\alpha_{n}) can be ordered in terms of their Laplace eigenvalues λΔ(ϕ)\lambda_{\Delta}(\phi) given by

λΔ(ϕ)=n3n24α12+α22++αn22\lambda_{\Delta}(\phi)=\frac{n^{3}-n}{24}-\frac{\alpha_{1}^{2}+\alpha_{2}^{2}+\cdots+\alpha_{n}^{2}}{2}

as proved by Stephen Miller [Mil02].

Let ϕ\phi be a Hecke-Maass cusp form for SL(n,)\text{\rm SL}(n,\mathbb{Z}) for n2n\geq 2 and set

ϕ,ϕ:=SL(n,)\𝔥nϕ(g)ϕ(g)¯𝑑g\langle\phi,\phi\rangle\,:=\hskip-5.0pt\int\limits_{\text{\rm SL}(n,\mathbb{Z})\backslash\mathfrak{h}^{n}}\hskip-5.0pt\phi(g)\overline{\phi(g)}\,dg

to denote the Petersson norm of ϕ.\phi. The Hecke-Maass cusp forms form a Hilbert space over \mathbb{C} with respect to the Petersson inner product.

Definition 1.1 (L-function of a Hecke-Maass cusp form).

Let ϕ\phi be a Hecke-Maass cusp form for SL(n,)\text{\rm SL}(n,\mathbb{Z}). Then for ss\in\mathbb{C} with Re(s)\text{\rm Re}(s) sufficiently large we define the L-function L(s,ϕ):=k=1λ(k)ksL(s,\phi):=\sum\limits_{k=1}^{\infty}\lambda(k)k^{-s} where λ(k)\lambda(k) is the kthk^{th} Hecke eigenvalue of ϕ.\phi.

Definition 1.2 (Asymptotic orthogonality relation for GL(n,)\text{\rm\bf GL}(n,\mathbb{R})).

Let {ϕj}j=1,2,\{\phi_{j}\}_{j=1,2,\ldots} (with associated Langlands parameters α(j)=(α1(j),α2(j),,αn(j))\alpha^{(j)}=(\alpha_{1}^{(j)},\alpha_{2}^{(j)},\ldots,\alpha_{n}^{(j)})) denote an orthogonal basis of Hecke-Maass cusp forms for SL(n,)\operatorname{SL}(n,{\mathbb{Z}}) with L-function given by L(s,ϕj):=k=1λj(k)ks.L(s,\phi_{j}):=\sum\limits_{k=1}^{\infty}\lambda_{j}(k)k^{-s}. Fix positive integers ,m\ell,m. Then, for TT\to\infty, we have

limTj=1λj()λj(m)¯hT(α(j))jj=1hT(α(j))j={1+o(1)if=m,o(1)ifm.\lim_{T\to\infty}\,\frac{\sum\limits_{j=1}^{\infty}\lambda_{j}(\ell)\,\overline{\lambda_{j}(m)}\,\frac{h_{T}\left(\alpha^{(j)}\right)}{\mathcal{L}_{j}}}{\sum\limits_{j=1}^{\infty}\frac{h_{T}\left(\alpha^{(j)}\right)}{\mathcal{L}_{j}}}=\begin{cases}1+o\left(1\right)&\text{if}\;\;\ell=m,\\ \;\;o\left(1\right)&\text{if}\;\;\ell\neq m.\end{cases}

where j=L(1,Adϕj)\mathcal{L}_{j}=L(1,\operatorname{Ad}{\phi_{j}}) and hT(α(j))h_{T}\left(\alpha^{(j)}\right) is a smooth function of the variables α(j),T\alpha^{(j)},T (for T>0T>0) with support on the Laplace eigenvalues λΔ(ϕj)\lambda_{\Delta}(\phi_{j}) where 0<λΔ(ϕj)T.0<\lambda_{\Delta}(\phi_{j})\ll T.

Remark 1.3 (Power savings error term).

The asymptotic orthogonality relation has a power savings error term if o(1)o(1) can be replaced with 𝒪(Tθ)\mathcal{O}\left(T^{-\theta}\right) for some fixed θ>0.\theta>0. The error terms o(1),𝒪(Tθ)o(1),\mathcal{O}\left(T^{-\theta}\right) will generally depend on L,ML,M. This type of asymptotic orthogonality relation was first conjectured by Fan Zhou [Zho14].

Remark 1.4 (Normalization of Hecke-Maass cusp forms).

The approach we take in proving asymptotic orthogonality relations for GL(n,)\text{\rm GL}(n,\mathbb{R}) is the Kuznetsov trace formula presented in §4 where λj()λj(m)¯ϕj,ϕj\frac{\lambda_{j}(\ell)\overline{\lambda_{j}(m)}}{\langle\phi_{j},\phi_{j}\rangle} (which are independent of the way the ϕj\phi_{j} are normalized) appears naturally on the spectral side of the trace formula leading to an asymptotic orthogonality relation of the form

(1.5) limTj=1λj()λj(m)¯hT(α(j))ϕj,ϕjj=1hT(α(j))ϕj,ϕj={1+o(1)if=m,o(1)ifm.\lim_{T\to\infty}\,\frac{\sum\limits_{j=1}^{\infty}\lambda_{j}(\ell)\,\overline{\lambda_{j}(m)}\,\frac{h_{T}\left(\alpha^{(j)}\right)}{\langle\phi_{j},\phi_{j}\rangle}}{\sum\limits_{j=1}^{\infty}\frac{h_{T}\left(\alpha^{(j)}\right)}{\langle\phi_{j},\phi_{j}\rangle}}=\begin{cases}1+o\left(1\right)&\text{if}\;\;\ell=m,\\ \;\;o\left(1\right)&\text{if}\;\;\ell\neq m.\end{cases}

If we normalize ϕj\phi_{j} so that its first Fourier coefficient is equal to one then it is shown in Proposition 4.4 that

ϕj,ϕj=cnL(1,Adϕj)1iknΓ(1+αi(j)αk(j)2),(cn0).\langle\phi_{j},\phi_{j}\rangle=c_{n}L(1,\operatorname{Ad}{\phi_{j}})\prod\limits_{1\leq i\neq k\leq n}\Gamma\left(\frac{1+\alpha^{(j)}_{i}-\alpha^{(j)}_{k}}{2}\right),\qquad\quad(c_{n}\neq 0).

This allows us (with a modification of the test function hTh_{T}) to replace the inner product ϕj,ϕj\langle\phi_{j},\phi_{j}\rangle appearing in (1.5) with the adjoint L-function j\mathcal{L}_{j} as in Definition 1.2. The main reason for doing this is that there are much better techniques developed for bounding special values of L-functions as opposed to bounding inner products of cusp forms. So having j1\mathcal{L}_{j}^{-1} in the asymptotic orthogonality relation instead of ϕj,ϕj1\langle\phi_{j},\phi_{j}\rangle^{-1} will allow us to obtain better error terms in applications.

Orthogonality relations as in Definition 1.2 have a long history going back to Dirichlet (for the case of GL(1)) who introduced the orthogonality relation for Dirichlet characters to prove infinitely many primes in arithmetic progressions. Bruggeman [Bru78] was the first to obtain an asymptotic orthogonality relation for GL(2) which he presented in the form

limTj=1λj()λj(m)¯4π2eλΔ(ϕj)TTcosh(πλΔ(ϕj)14)={1if=m,0ifm.\lim_{T\to\infty}\sum_{j=1}^{\infty}\frac{\lambda_{j}(\ell)\overline{\lambda_{j}(m)}\cdot 4\pi^{2}e^{-\frac{\lambda_{\Delta}(\phi_{j})}{T}}}{T\cosh\left(\pi\sqrt{\lambda_{\Delta}(\phi_{j})-\tfrac{1}{4}}\right)}=\begin{cases}1&\text{if}\;\ell=m,\\ 0&\text{if}\;\ell\neq m.\end{cases}

where {ϕj}j=1,2,\big{\{}\phi_{j}\big{\}}_{j=1,2,\ldots} goes over an orthogonal basis of Hecke-Maass cusp forms for SL(2,)\text{\rm SL}(2,\mathbb{Z}). This is not quite in the form of Definition 1.2 but it can be put into that form with some work. Other versions of GL(2) type orthogonality relations with important applications were obtained by Sarnak [Sar87], and, for holomorphic Hecke modular forms, by Conrey-Duke-Farmer [CDF97] and J.P. Serre [Ser97].

The first asymptotic orthogonality relations for GL(3) with power savings error term were proved independently by Blomer [Blo13] and Goldfeld-Kontorovich [GK13] in 2013. In 2021 Goldfeld-Stade-Woodbury [GSW21] were the first to obtain a power-saving asymptotic orthogonality relation as in Definition 1.2 for GL(4).

A major breakthrough was obtained by Matz-Templier [MT21] who unconditionally proved an asymptotic orthogonality relation for SL(n,){\rm SL}(n,\mathbb{Z}), as in (1.5), for a wide class of test functions for all n2n\geq 2 (with power savings) but without the harmonic weights given by the inverse of the adjoint L-function at 1. Their results were further strengthened in Finis-Matz [FM21]. The principal tool used to prove the asymptotic orthogonality relation in [MT21] was the Arthur-Selberg trace formula, whereas our approach is the natural generalization of the earlier results [Blo13], [GK13], [GSW21], which were based on the Kuznetsov trace formula. Blomer [Blo21] presented a very nice exposition comparing the Arthur-Selberg and Kuznetsov trace formulae which we now briefly summarize for the application to asymptotic orthogonality relations.        \bullet The first key difference between these trace formulae is that the spectral side of the Kuznetsov trace formula has harmonic weights j1\mathcal{L}_{j}^{-1} while the Arthur-Selberg trace formula does not have these harmonic weights. For GL(n)\operatorname{GL}(n) with n>3n>3 it is not currently known how to remove these weights (see [BZ20] for how to remove the weights on GL(3)\operatorname{GL}(3)). In [Blo21] Blomer remarks that “for applications to L-functions involving period formulae it is often desirable to have an additional factor 1/L(1,Adϕ)1/L(1,{\rm Ad}\;\phi) in the cuspidal spectrum, but in other situations one may prefer a summation formula without an extra L-value.”        \bullet The second major difference between these trace formulae is that the spectral side of the Kuznetsov trace formula does not contain residual spectrum while the Arthur-Selberg trace formula does. As pointed out by a referee the bulk of the work in Matz-Templier [MT21] consists in bounding the unipotent contribution on the geometric side of the Arthur trace formula so that it stays in line with the error term coming from the residual Eisenstein contribution on the spectral side given by Lapid-Mueller [LM09]. These residual Eisenstein series do not appear in the Kuznetsov trace formula which leads to a very strong conjectural error term in Theorem 1.1. In fact, the largest error term on the spectral side of the Kuznetsov trace formula arises from the tempered Eisenstein series coming from the maximal parabolic having (n1,1)(n-1,1) Levi block decomposition. For explicit comparisons between our main theorem and the results of [MT21] see Remark 1.4.        \bullet There are certain applications of our results using the Kuznetsov trace formula approach that go beyond the results in [MT21], [FM21]. Recall that λj(p)\lambda_{j}(p) denotes the pthp^{th} Hecke eigenvalue of the Maass form ϕj\phi_{j}. Fan’s thesis concerns the so-called vertical Sato-Tate problem which is a conjecture about the distribution of λj(p)\lambda_{j}(p) where pp is fixed and jj varies. This problem was studied by Bruggeman [Bru78] and Sarnak [Sar87] (for Maass forms), and Serre [Ser97] and Conrey-Duke-Farmer [CDF97] (for holomorphic forms), who showed by fixing pp and varying jj, that λj(p)\lambda_{j}(p) is an equidistributed sequence with respect to the Plancherel measure which depends on pp. Strikingly, as observed by Fan Zhou ([Zho14]), if we give each Hecke eigenvalue λj(p)\lambda_{j}(p) the weight j1\mathcal{L}_{j}^{-1}, then the distribution involves the Sato-Tate measure which is independent of pp. Jana, in [Jan21], generalized the results of Zhou, but he only obtained an asymptotic formula without a power savings error term. A problem for the future would be to combine Jana’s approach with the methods of this paper. Jana also obtains bounds toward Sarnak’s density hypothesis using this strategy that are stronger than anything known using the Arthur-Selberg trace formula. The main aim of this paper is to explicitly work out an asymptotic orthogonality relation for SL(n,)\operatorname{SL}(n,\mathbb{Z}) via the Kuznetsov trace formula for a special choice of test function hT,R(n)h_{T,R}^{(n)} whose form is that of a Gaussian times a fixed polynomial. We do not address applications in this paper and leave that to future research. See [Blo21] for various applications of the Arthur-Selberg and Kuznetsov trace formulae and how they compare. We also point out that the Kuznetsov trace formula was generalized by Jacquet and Lai [JL85] who developed the relative trace formula which has had a wide following with new types of applications. See §1.1 for the statement of our main theorem. The proof we give assumes the Ramanujan conjecture at \infty but it is possible to prove a weaker result by dropping this assumption. Otherwise the proof is unconditional for n5n\leq 5. In particular, the case n=5n=5 represents a complete, new result. For n>5n>5, our result is conditional on two conjectures.

1.2. Ishii-Stade Conjecture

The Ishii-Stade Conjecture (see §8.2) concerns the normalized Mellin transform W~n,α(s)\widetilde{W}_{n,\alpha}(s) of the GL(n,)\text{\rm GL}(n,\mathbb{R}) Whittaker function Wn,α(y)W_{n,\alpha}(y) defined in Definition 2.3. Here, s=(s1,s2,,sn1)n1s=(s_{1},s_{2},\ldots,s_{n-1})\in{\mathbb{C}}^{n-1}, and α=(α1,α2,,αn)=n1\alpha=(\alpha_{1},\alpha_{2},\ldots,\alpha_{n})={\mathbb{C}}^{n-1} satisfies i=1nαi=0\sum\limits_{i=1}^{n}\alpha_{i}=0.

Suppose integers mm and δ\delta, with 1mn11\leq m\leq n-1 and δ0\delta\geq 0, are given. The Ishii-Stade Conjecture expresses W~n,α(s)\widetilde{W}_{n,\alpha}(s) as a finite linear combination, with coefficients that are rational functions of the sjs_{j}’s and αk\alpha_{k}’s, of shifted Mellin transforms

W~n,α(s+Σ),\widetilde{W}_{n,\alpha}(s+\Sigma),

where Σ(0)n1\Sigma\in({\mathbb{Z}}_{\geq 0})^{n-1} and the mmth coordinate of Σ\Sigma is δ\geq\delta. In other words, for such δ\delta and mm, the conjecture expresses the Mellin transform W~n,α(s)\widetilde{W}_{n,\alpha}(s) in terms of shifts of this Mellin transform by at least δ\delta units to the right in the variable sms_{m}.

Much as recurrence relations of the form

Γ(s)=[(s+δ1)(s+δ2)(s+1)s]1Γ(s+δ)\Gamma(s)=[(s+\delta-1)(s+\delta-2)\cdots(s+1)s]^{-1}\Gamma(s+\delta)

for Euler’s Gamma function imply concrete results concerning analytic continuation, poles, and residues of that function, so will the Ishii-Stade conjecture allow us to obtain explicit information about the behavior of W~n,a(s)\widetilde{W}_{n,a}(s) beyond its original, a priori domain of definition. This explicit information will be crucial to the analysis of our test function hTh_{T}, and consequently, to our derivation of an an asymptotic orthonality relation as in Definition 1.2.

We have been able to prove the Ishii-Stade Conjecture for GL(n,)\text{\rm GL}(n,\mathbb{R}) with 2n52\leq n\leq 5. See §8.2 below.

1.3. Lower bound conjecture for Rankin-Selberg L-functions

Fix n2n\geq 2. Let n=n1++nrn=n_{1}+\cdots+n_{r} be a partition of nn with ni>0,(i=1,,r)n_{i}\in\mathbb{Z}_{>0},(i=1,\ldots,r). The second conjecture we require for the proof of the asymptotic orthogonality relation for GL(n,)\text{\rm GL}(n,\mathbb{R}) is a conjecture on the lower bound for Rankin-Selberg L-functions L(s,ϕk×ϕk)L(s,\phi_{k}\times\phi_{k^{\prime}}) on the line Re(s)=1,\text{\rm Re}(s)=1, where ϕk,ϕk\phi_{k},\;\phi_{k^{\prime}} (for 1k<kr1\leq k<k^{\prime}\leq r) are Hecke-Maass cusp forms for SL(nk,)\text{\rm SL}(n_{k},\mathbb{Z}), SL(nk,)\text{\rm SL}(n_{k^{\prime}},\mathbb{Z}), respectively. For a Hecke-Maass cusp form ϕ\phi with Langlands parameters (α1,,αn)(\alpha_{1},\ldots,\alpha_{n}), let

(1.1) c(ϕ)=(1+|α1|)(1+|α2|)(1+|αn|)c(\phi)=(1+\lvert\alpha_{1}\rvert)(1+\lvert\alpha_{2}\rvert)\cdots(1+\lvert\alpha_{n}\rvert)

denote the analytic conductor of ϕ\phi as defined by Iwaniec and Sarnak [IS00].

Conjecture 1.2 (Lower bounds for Rankin-Selberg L-functions).

Let ε>0\varepsilon>0 be fixed. Then we have the lower bound

|L(1+it,ϕk×ϕk)|ε(c(ϕk)c(ϕk))ε(|t|+2)ε.\left|L(1+it,\phi_{k}\times\phi_{k^{\prime}})\right|\gg_{\varepsilon}\big{(}c(\phi_{k})\cdot c(\phi_{k^{\prime}})\big{)}^{-\varepsilon}\big{(}|t|+2\big{)}^{-\varepsilon}.
Remark 1.3.

Conjecture 1.2 follows from Langlands’ conjecture that ϕk×ϕk\phi_{k}\times\phi_{k^{\prime}} is automorphic for SL(nknk,)\text{\rm SL}(n_{k}\cdot n_{k^{\prime}},\;\mathbb{Z}). This can be proved via the method of de la Valée Poussin as in Sarnak [Sar04]. Interestingly, Sarnak’s approach can be extended to prove Conjecture 1.2 if ϕk\phi_{k^{\prime}} is the dual of ϕk\phi_{k} (see [GL18], [HB19]). Stronger bounds can also be obtained if one assumes the Lindelöf or Riemann hypothesis for Rankin-Selberg L-functions.

If nk=nk=2n_{k}=n_{k}^{\prime}=2 it was proved by Ramakrishnan [Ram00] that ϕk×ϕk\phi_{k}\times\phi_{k^{\prime}} is automorphic for SL(4,),\text{\rm SL}(4,\mathbb{Z}), thus proving the lower bound conjecture for n4.n\leq 4. Further, for nk=2n_{k}=2 and nk=3n_{k}^{\prime}=3, it was proved by Kim and Shahidi [KS02] that ϕk×ϕk\phi_{k}\times\phi_{k^{\prime}} is automorphic for SL(6,),\text{\rm SL}(6,\mathbb{Z}), thus proving the lower bound conjecture for n5n\leq 5.

1.4. Constructing the test functions

Fix an integer n2n\geq 2. We now construct two complex-valued test functions on the space of Langlands parameters

{α=(α1,,αn)n|α1++αn=0}\Big{\{}\alpha=\left(\alpha_{1},\ldots,\alpha_{n}\right)\in\mathbb{C}^{n}\;\Big{|}\;\alpha_{1}+\cdots+\alpha_{n}=0\Big{\}}

that will be used in our proof of the orthogonality relation for GL(n,).\text{\rm GL}(n,\mathbb{R}).

We begin by introducing an auxiliary polynomial that is used in constructing the test functions.

Definition 1.1 (The polynomial R(n)(α)\mathcal{F}_{R}^{(n)}(\alpha)).

Let R>0R\in\mathbb{Z}_{>0} and let α=(α1,,αn)\alpha=(\alpha_{1},\ldots,\alpha_{n}) be a Langlands parameter. Then we define

R(n)(α):=j=1n2K,L(1,2,,n)#K=#L=j(1+kKαkLα)R2.\mathcal{F}_{R}^{(n)}(\alpha):=\prod_{j=1}^{n-2}\;\underset{\#K=\#L=j}{\prod_{K,L\,\subseteq\,(1,2,\ldots,n)}}\left(1+\sum_{k\in K}\alpha_{k}-\sum_{\ell\in L}\alpha_{\ell}\right)^{\frac{R}{2}}.

Note that if α(i)n,\alpha\in(i\mathbb{R})^{n}, then 1(n)(α)\mathcal{F}_{1}^{(n)}(\alpha) is the square root of a polynomial in α\alpha of degree 2D(n)2D(n), where

(1.2) D(n)=j=1n212(nj)((nj)1)=12(2nn)n(n1)22n1.D(n)=\sum_{j=1}^{n-2}\frac{1}{2}\begin{pmatrix}n\\ j\end{pmatrix}\left(\begin{pmatrix}n\\ j\end{pmatrix}-1\right)=\frac{1}{2}\begin{pmatrix}2n\\ n\end{pmatrix}-\frac{n(n-1)}{2}-2^{n-1}.

By abuse of notation, we refer to R(n)\mathcal{F}_{R}^{(n)} as a polynomial although this is not strictly the case unless RR is even. For α\alpha with bounded real and imaginary parts, say, |Re(αj)|<R|\text{\rm Re}(\alpha_{j})|<R and |Im(αj)|<T1+ε|\text{\rm Im}(\alpha_{j})|<T^{1+\varepsilon} we have

(1.3) |R(n)(α)|Tε+RD(n),(T+)\left|\mathcal{F}_{R}^{(n)}(\alpha)\right|\ll T^{\varepsilon+R\cdot D(n)},\quad\qquad(T\to+\infty)

with an implicit constant depending on n,ε,Rn,\varepsilon,R.

Definition 1.4 (The test functions pT,Rn,#(α)p_{T,R}^{n,\#}(\alpha) and hT,R(n)(α)h_{T,R}^{(n)}(\alpha)).

Let R>0R\in\mathbb{Z}_{>0} and T+T\to+\infty. Then for a Langlands parameter α=(α1,,αn)\alpha=(\alpha_{1},\ldots,\alpha_{n}), we define

pT,Rn,#(α):=eα12+α22++αn22T2R(n)(α2)1jknΓ(1+2R+αjαk4),hT,R(n)(α):=|pT,Rn,#(α)|21jknΓ(1+αjαk2).p_{T,R}^{n,\#}(\alpha):=e^{\frac{\alpha_{1}^{2}+\alpha_{2}^{2}+\cdots+\alpha_{n}^{2}}{2T^{2}}}\cdot\mathcal{F}_{R}^{\left(n\right)}\left(\tfrac{\alpha}{2}\right)\prod_{1\leq j\neq k\leq n}\Gamma\left(\tfrac{1+2R+\alpha_{j}-\alpha_{k}}{4}\right),\quad h_{T,R}^{(n)}(\alpha):=\frac{\left|p_{T,R}^{n,\#}(\alpha)\right|^{2}}{\prod\limits_{1\leq j\neq k\leq n}\Gamma\left(\tfrac{1+\alpha_{j}-\alpha_{k}}{2}\right)}.

We observe that, by Stirling’s formula for the Gamma function and by (1.2) and (1.3), we have

(1.5) |hT,R(n)(α)|TR((2nn)2n)n(n1)2\big{|}h_{T,R}^{(n)}(\alpha)\big{|}\ll T^{R\cdot\left(\binom{2n}{n}-2^{n}\right)-\frac{n(n-1)}{2}}

whenever |Re(αj)||\text{\rm Re}(\alpha_{j})| is bounded and |Im(αj)|<T1+ε|\text{\rm Im}(\alpha_{j})|<T^{1+\varepsilon} for 1jn1\leq j\leq n. The implied constant in (1.5) depends on n,ε,n,\varepsilon, and RR.

Remark 1.6 (Positivity of hT,R(n)h_{T,R}^{(n)}\,).

Writing α=(α1,α2,,αn)\alpha=(\alpha_{1},\alpha_{2},\ldots,\alpha_{n}) with αj=itj\alpha_{j}=it_{j} and tjt_{j}\in\mathbb{R} for each j=1,2,,nj=1,2,\ldots,n, the function hT,R(n)(α)h_{T,R}^{(n)}(\alpha) is positive. This is the case because Γ(1+iu2)Γ(1iu2)=|Γ(1+iu2)|2\Gamma(\frac{1+iu}{2})\Gamma(\frac{1-iu}{2})=\lvert\Gamma(\frac{1+iu}{2})\rvert^{2} for uu\in\mathbb{R}.

Remark 1.7 (Whittaker transform of the test function).

The symbol #\# in the test function pT,Rn,#p_{T,R}^{n,\#} means this function is the Whittaker transform of pT,R(n)p_{T,R}^{(n)}. See § 8.

1.5. The Main Theorem

Theorem 1.1.

Fix n2.n\geq 2. Let {ϕj}j=1,2,\{\phi_{j}\}_{j=1,2,\ldots} denote an orthogonal basis of Hecke-Maass cusp forms for SL(n,)\operatorname{SL}(n,{\mathbb{Z}}) (assumed to be tempered at \infty) with associated Langlands parameter

α(j)=(α1(j),α2(j),,αn(j))(i)n\alpha^{(j)}=\big{(}\alpha^{(j)}_{1},\alpha^{(j)}_{2},\ldots,\alpha^{(j)}_{n}\big{)}\in\left(i\cdot\mathbb{R}\right)^{n}

and L-function L(s,ϕj):=k=1λj(k)ksL(s,\phi_{j}):=\sum\limits_{k=1}^{\infty}\lambda_{j}(k)\,k^{-s}. Fix positive integers ,m\ell,m. Then assuming the Ishii-Stade conjecture 8.3 and the lower bound conjecture for Rankin-Selberg L-functions 1.2, we prove that for TT\to\infty,

j=1λj()λj(m)¯hT,R(n)(α(j))j=δ,mi=1n1𝔠iTR((2nn)2n)+ni+𝒪ε,R,n((m)n2+134TR((2nn)2n)+ε)\sum\limits_{j=1}^{\infty}\lambda_{j}(\ell)\,\overline{\lambda_{j}(m)}\,\frac{h_{T,R}^{(n)}\left(\alpha^{(j)}\right)}{\mathcal{L}_{j}}=\delta_{\ell,m}\cdot\sum_{i=1}^{n-1}\mathfrak{c}_{i}\cdot T^{R\cdot\left(\binom{2n}{n}-2^{n}\right)+n-i}+\;\mathcal{O}_{\varepsilon,R,n}\left((\ell m)^{\frac{n^{2}+13}{4}}\cdot T^{R\cdot\left(\binom{2n}{n}-2^{n}\right)\,+\,\varepsilon}\right)

where δ,m\delta_{\ell,m} is the Kronecker symbol, j=L(1,Adϕj),\mathcal{L}_{j}=L(1,\operatorname{Ad}\,\phi_{j}), and 𝔠1,,𝔠n1>0\mathfrak{c}_{1},\ldots,\mathfrak{c}_{n-1}>0 are absolute constants which depend at most on RR and nn.

Because Conjectures 1.2 and 8.3 are known to be true for 2n52\leq n\leq 5 (see Remark 1.3 and §8.2), the above result is unconditional for such nn.

Remark 1.2.

Qiao Zhang [Zha22] recently proved the lower bound

(1.3) |L(1+it,ϕk×ϕk)|(c(ϕk)c(ϕk))θk,k(|t|+2)nknk2(11nk+nk)ε\left|L(1+it,\phi_{k}\times\phi_{k^{\prime}})\right|\gg\big{(}c(\phi_{k})\cdot c(\phi_{k^{\prime}})\big{)}^{-\theta_{k,k^{\prime}}}\big{(}|t|+2\big{)}^{-\frac{n_{k}n_{k^{\prime}}}{2}\big{(}1-\frac{1}{n_{k}+n_{k^{\prime}}}\big{)}-\varepsilon}

with θk,k=nk+nk+ε\theta_{k,k^{\prime}}=n_{k}+n_{k^{\prime}}+\varepsilon. This improves on Brumley’s bound who obtained nearly the same result but with the term nknk2\frac{n_{k}n_{k^{\prime}}}{2} replaced by nknkn_{k}n_{k^{\prime}} (see [Bru06] and the appendix of [Lap13]). Assuming (1.3) we can replace the error term in Theorem 1.1 with

𝒪ε,R,n,,m(TR((2nn)2n)+n1+n(n2)6(θk,k8n2)).\mathcal{O}_{\varepsilon,R,n,\ell,m}\Big{(}T^{R\cdot\left(\binom{2n}{n}-2^{n}\right)+n-1\;+\;\frac{n(n-2)}{6}\big{(}\theta_{k,k^{\prime}}-\frac{8}{n^{2}}\big{)}}\Big{)}.

So if one could prove (1.3) with θk,k<8n2\theta_{k,k^{\prime}}<\frac{8}{n^{2}} this would give a power savings error term in our main theorem and would remove the assumption of the lower bound conjecture 1.2. In fact, the proof establishes a black box by which improvements to bounds on Rankin-Selberg L-functions result in better power savings error terms for the continuous spectrum contribution to the asymptotic orthogonality relation.

Remark 1.4.

A variant of Theorem 1.1 is obtained unconditionally in [MT21], [FM21], without the arithmetic weights j1\mathcal{L}_{j}^{-1} and with different test functions, which are indicator functions of α(j)TΩ\alpha^{(j)}\in T\Omega, where Ω\Omega is a Weyl group invariant bounded open subset of i𝔞i\cdot\mathfrak{a}^{*}, where 𝔞\mathfrak{a} is the Lie algebra of the subgroup of diagonal matrices with positive entries. Additionally, the results of [MT21], [FM21] do not entail the polynomial weights of size TR((2nn)2n)n(n1)2T^{R\cdot\left(\binom{2n}{n}-2^{n}\right)-\frac{n(n-1)}{2}} coming from hT,R(n)(α)h_{T,R}^{(n)}(\alpha) (cf. (1.5)).

The error term obtained in [FM21], in the present setting of SL(n,)\operatorname{SL}(n,{\mathbb{Z}}), is T(n1)(n+2)21\ll T^{\frac{(n-1)(n+2)}{2}-1} as TT\to\infty. Here, (n1)(n+2)2\frac{(n-1)(n+2)}{2} is the dimension of the generalized upper half-plane 𝔥n\mathfrak{h}^{n}, and the error term obtained by Finis-Matz has exponent equal to that dimension minus 1. By comparison, if one removes the polynomial weights TR((2nn)2n)n(n1)2T^{R\cdot\left(\binom{2n}{n}-2^{n}\right)-\frac{n(n-1)}{2}} from the error term in Theorem 1.1 above, then one obtains an error term that is Tn(n1)2+ε\ll T^{\frac{n(n-1)}{2}+\varepsilon}. Also note that our main term is of a stronger form than that of [MT21], [FM21], in that ours entails a sum of n1n-1 different high order asymptotics.

More recently, Jana [Jan21] obtained a proof of the asymptotic orthogonality relation defined in 1.2, using the Kuznetsov trace formula and not the Selberg trace formula, with applications to the equidistribution of Satake parameters with respect to the Sato-Tate measure, second moment estimates of central values of L-functions as strong as Lindelöf on average, and distribution of low lying zeros of automorphic L-functions in the analytic conductor aspect. The paper of Jana does not contain a power saving error term.

Remark 1.5.

It is possible to remove the assumption of Ramanujan at the infinite place with more work which results in a weaker power savings error term in Theorem 1.1. For a Maass form ϕ\phi with Langlands parameter α\alpha, note that the test function hT,R(α)h_{T,R}(\alpha) is positive. This is true because, even if α\alpha is a Langlands parameter of an element in the complementary spectrum, α-\alpha is a permutation of α¯\overline{\alpha}. A weaker version of Theorem 1.1 can be proved if one assumes that almost all (except for a set of zero density) are tempered. Such results have been obtained in [MT21], [FM21].

Proof of Theorem 1.1.

Computing the inner product of certain Poincaré series in two ways (see the outline in §1.6 below), we obtain a Kuznetsov trace formula relating the so-called geometric and spectral sides. The geometric side consists of a main term \mathcal{M} and a Kloosterman contribution 𝒦\mathcal{K}. The spectral side also consists of two components: a cuspidal (i.e., discrete) contribution 𝒞\mathcal{C} and an Eisenstein (i.e., continuous) contribution \mathcal{E}.

The left hand side of the theorem is precisely 𝒞\mathcal{C}. The first set of terms on the right hand side comes from the asymptotic formula for \mathcal{M} given in Proposition 5.1. The power of TT in the error term comes from the bound for \mathcal{E} given in Theorem 7.1 (which also gives a factor of (m)121n2+1\left(\ell m\right)^{\frac{1}{2}-\frac{1}{n^{2}+1}}). A bound for 𝒦\mathcal{K}, which is a (finite) sum of terms w\mathcal{I}_{w}, with the same power of TT but with the given power of m\ell m follows as a consequence of Proposition 6.1. ∎

1.6. Outline of the key ideas in the proofs

Fix n2.n\geq 2. The GL(n,)\text{\rm GL}(n,\mathbb{R}) orthogonality relation appears directly in the spectral side of the Kuznetsov trace formula for GL(n,)\text{\rm GL}(n,\mathbb{R}) which we now discuss. The Kuznetsov trace formula is obtained by computing the inner product of two Poincaré series on SL(n,)\𝔥n\text{\rm SL}(n,\mathbb{Z})\backslash\mathfrak{h}^{n} in two different ways. The Poincaré series are constructed in a similar manner to Borel Eisenstein series by taking all Un()\SL(n,)U_{n}(\mathbb{Z})\backslash\text{\rm SL}(n,\mathbb{Z}) translates of a certain test function which we choose to be the pT,R(n)p_{T,R}^{(n)} test function in Definition 1.4 multiplied by a character and a power function (see Definition 2.7).

The first way of computing the inner product of two Poincaré series is to replace one of the Poincaré series with its spectral expansion into cusp forms and Eisenstein series and then unravel the other Poincaré series with the Rankin-Selberg method. This gives the spectral contribution which has two parts: the cuspidal contribution and the Eisenstein contribution. The second way of computing the inner product is to replace one of the Poincaré series with its Fourier Whittaker expansion and then unravel the other Poincaré series with the Rankin-Selberg method. This is called the geometric contribution to the trace formula, which also consists of two parts: a main term, and the so-called Kloosterman contribution. The precise results of these computations are given in Theorems 4.1 and 4.1, respectively.


Bounding the Eisenstein contribution

The key component of the Eisenstein contribution to the Kuznetsov trace formula is the inner product of an Eisenstein series and the Poincaré series PMP^{M} given in Definition 2.7. By unraveling the Poincaré series in the inner product (see Proposition 4.2) we essentially obtain the MthM^{th} Fourier coefficient of the Eisenstein series multiplied by the Whittaker transform of pT,R(n)p_{T,R}^{(n)}. The explicit formula for the MthM^{th} Fourier coefficient of the most general Langlands Eisenstein series given in Proposition 4.5 allows us to effectively bound all the terms in the integrals appearing in the Eisenstein contribution except for the product of adjoint L-functions

(1.1) k=1rnk1L(1,Adϕk)12\underset{n_{k}\neq 1}{\prod_{k=1}^{r}}L^{*}\big{(}1,\text{\rm Ad}\;\phi_{k}\big{)}^{-\frac{1}{2}}

appearing in that proposition. When considering the Eisenstein contribution to the Kuznetsov trace formula for GL(n,)\text{\rm GL}(n,\mathbb{R}) all the adjoint L-functions in the above product are for cusp forms ϕk\phi_{k} of lower rank nk<nn_{k}<n. Now in the special case that =m=1\ell=m=1, our Main Theorem 1.1 for GL(n,)\text{\rm GL}(n,\mathbb{R}) gives a sharp bound for the sum of reciprocals of all adjoint L-functions of lower rank. This allows us to inductively prove a power savings bound for the product (1.1).


Asymptotic formula for the geometric contribution


We prove that the geometric contribution is a sum of expressions w\mathcal{I}_{w} over elements ww in the Weyl group of SL(n,)\text{\rm SL}(n,\mathbb{Z}). The w\mathcal{I}_{w} are complicated multiple sums of multiple integrals weighted by Kloosterman sums (see (4.2)). If w1w_{1} is the trivial element of the Weyl group then we obtain an asymptotic formula for w1\mathcal{I}_{w_{1}} (see Proposition 5.1) while for all other Weyl group elements wi\mathcal{I}_{w_{i}}, with i>1i>1, we obtain error terms with strong bounds for |wi||\mathcal{I}_{w_{i}}| (see Proposition 6.1) which are bounded by the final error term on the right side of our main theorem.

The key terms in (4.2), the formula for w\mathcal{I}_{w}, are the Kloosterman sums and two appearances of the test function pT,R(n)p_{T,R}^{(n)}: one that is twisted by the Weyl group element ww and one that is not. For the Kloosterman sums, we rely on bounds given by [DR98]. The task of giving strong bounds for pT,R(n)(y)p_{T,R}^{(n)}(y) occupies Sections 8, 9 and 10. We deal with the combinatorics of the twisted pT,R(n)p_{T,R}^{(n)}-function, and we combine the bounds for it, the other pT,R(n)p_{T,R}^{(n)}-function and the Kloosterman sums in Section 6.

The function pT,R(n)p_{T,R}^{(n)} is the inverse Whittaker transform of the test function pT,Rn,#p_{T,R}^{n,\#} given in Definition 1.4 above. Thanks to a formula of Goldfeld-Kontorovich [GK12], we can realize this as an integral of the product of pT,Rn,#p_{T,R}^{n,\#}, the Whittaker function WαW_{\alpha} (see Definition 2.3), and certain additional gamma factors. We then write the Whittaker function as the inverse Mellin transform of its Mellin transform: W~n,α(s)\widetilde{W}_{n,\alpha}(s). This leads to the formula (valid for any ε>0\varepsilon>0):

pT,R(n)(y)=12n1Re(α1)=0Re(αn1)=0eα12+α22++αn2T2/2R(n)(α)1jknΓ(1+2R+αjαk4)Γ(αjαk2)Re(s1)=εRe(sn1)=ε(j=1n1yjj(nj)2(πyj)2sj)W~n,α(s)dsdα.p_{T,R}^{(n)}(y)=\frac{1}{2^{n-1}}\int\limits_{\text{\rm Re}(\alpha_{1})=0}\cdots\int\limits_{\text{\rm Re}(\alpha_{n-1})=0}e^{\frac{\alpha_{1}^{2}+\alpha_{2}^{2}+\cdots+\alpha_{n}^{2}}{T^{2}/2}}\,\mathcal{F}_{R}^{\left(n\right)}\left(\alpha\right)\prod_{1\leq j\neq k\leq n}\frac{\Gamma\left(\tfrac{1+2R+\alpha_{j}-\alpha_{k}}{4}\right)}{\Gamma\left(\frac{\alpha_{j}-\alpha_{k}}{2}\right)}\\ \cdot\int\limits_{\text{Re}(s_{1})=\varepsilon}\cdots\int\limits_{\text{Re}(s_{n-1})=\varepsilon}\left(\,\prod_{j=1}^{n-1}y_{j}^{\frac{j(n-j)}{2}}(\pi y_{j})^{-2s_{j}}\right)\widetilde{W}_{n,\alpha}\left(s\right)\,ds\,d\alpha.

To estimate the growth of pT,R(n)(y)p_{T,R}^{(n)}(y) uniformly in yy and TT as T+T\to+\infty, we shift the line of integration in the ss-integrals to Re(s)=a\text{\rm Re}(s)=-a with a=(a1,,an1)a=(a_{1},\ldots,a_{n-1}) where ai>0a_{i}>0 for i=1,,n1i=1,\ldots,n-1. We remark that this is precisely where the Ishii-Stade Conjecture is required. It is well known that

W~2,α(s)=Γ(s+α)Γ(sα),\widetilde{W}_{2,\alpha}(s)=\Gamma(s+\alpha)\Gamma(s-\alpha),

and hence understanding the values of W~2,α(s)\widetilde{W}_{2,\alpha}(s) for Re(s)<0\operatorname{Re}(s)<0 is straightforward by applying the functional equation for the gamma function or, equivalently, using an integral representation of the gamma function valid for Re(s)<0\operatorname{Re}(s)<0. A similar strategy can be used when n=3n=3. However, for n4n\geq 4, the analogous method seems intractable because the Mellin transform is not just a ratio of gamma functions, but an integral of such. To overcome this difficulty, we apply the Ishii-Stade conjecture to describe the values of W~n,α(s)\widetilde{W}_{n,\alpha}(s) in terms of sums of the Mellin transform of shifts of the ss-variables. See also Remark 8.11 below.

The Cauchy residue formula allows us to express pT,R(n)p_{T,R}^{(n)} as a sum of the shifted ss-integral (termed the shifted pT,R(n)p_{T,R}^{(n)} term and denoted pT,R(n)(y;a)p_{T,R}^{(n)}(y;-a)) and many residue terms. The description of the shifted pT,R(n)p_{T,R}^{(n)} and residue terms is given in Section 8.3. In order to bound pT,R(n)(y;a)p_{T,R}^{(n)}(y;-a) it is convenient to introduce the function T,R(a):=pT,R(n)(1;a)\mathcal{I}_{T,R}(-a):=p_{T,R}^{(n)}(1;-a).

The next step is to use a result of Ishii-Stade (see Theorem 8.5) which allows us to write the Mellin transform W~n,α(s)\widetilde{W}_{n,\alpha}(s) as an integral transformation of W~n1,β(z)\widetilde{W}_{n-1,\beta}(z) against certain additional gamma factors. It is important to note that β=(β1,,βn1)n1\beta=(\beta_{1},\ldots,\beta_{n-1})\in{\mathbb{C}}^{n-1} can be expressed in terms of α=(α1,,αn)\alpha=(\alpha_{1},\ldots,\alpha_{n}). By carefullly teasing apart the portion of α\alpha which determines β\beta and that which doesn’t, we are able to separate out the gamma factors that don’t depend on β\beta and bound T,R(n)(a)\mathcal{I}_{T,R}^{(n)}(-a) by the product of a power of TT and T,R(n1)(b)\mathcal{I}_{T,R}^{(n-1)}(-b) for a certain b=(b1,,bn1)n2b=(b_{1},\ldots,b_{n-1)}\in\mathbb{R}^{n-2}. This gives an inductive procedure, therefore, for bounding the shifted pT,R(n)p_{T,R}^{(n)} term.

In Section 10.2 we set notation for describing the (r1)(r-1)-fold shifted residue terms. This requires generalizing a result of Stade (see Theorem 10.1) on the first set of residues of W~n,α(s)\widetilde{W}_{n,\alpha}(s) (i.e., those that occur at Re(si)=0\operatorname{Re}(s_{i})=0) to, first, higher order residues (i.e., taking the residue with respect to multiple values sis_{i}), and second, to residues which occur along the lines Re(si)=k\operatorname{Re}(s_{i})=-k for k0k\in{\mathbb{Z}}_{\geq 0}. This result, together with a teasing out of the variables similar to that described above, allows us to bound an (r1)(r-1)-fold residue term as the product of certain powers of TT and the variables y1,,yn1y_{1},\ldots,y_{n-1} times

j=1rT,R(nj)(a(j)),where n=n1++nr.\prod\limits_{j=1}^{r}\mathcal{I}_{T,R}^{(n_{j})}(-a^{(j)}),\qquad\mbox{where $n=n_{1}+\cdots+n_{r}$.}

Applying the bounds on T,R(nj)\mathcal{I}_{T,R}^{(n_{j})} that we inductively established for bounding the shifted pT,R(n)p_{T,R}^{(n)} term, and keeping careful track of all of the exponents and terms a(j)a^{(j)}, we eventually show that the bound for the shifted main term is in fact valid for every residue term as well.

Remark 1.2.

In comparison to the results of [GK13] and [GSW21], we are using a slightly different normalization of the gamma functions and the auxiliary polynomial R(n)\mathcal{F}_{R}^{(n)} in the definition of the test functions pT,Rn,#p_{T,R}^{n,\#} and hT,R(n)h_{T,R}^{(n)} (see Definition 1.4). Adjusting for this difference the results obtained here when applied to n=3n=3 and n=4n=4 recover the previously proven asymptotic formulas.

2. Preliminaries

2.1. Notational conventions

Definition 2.1 (Hat notation for summation).

Suppose that m+m\in{\mathbb{Z}}_{+} and x=(x1,,xm)mx=(x_{1},\ldots,x_{m})\in{\mathbb{C}}^{m}. For any 0km0\leq k\leq m, define

x^k:=x1++xk.\widehat{x}_{k}:=x_{1}+\cdots+x_{k}.

Note that empty sums are assumed to be zero.

Definition 2.2 (Integration notation).

Let n2n\geq 2. We will often be working with nn- and (n1)(n-1)-tuples of real or complex numbers. We will denote such tuples without a subscript and use subscripts to refer to the components. For example, we set y=(y1,,yn1)>0n1y=(y_{1},\ldots,y_{n-1})\in\mathbb{R}_{>0}^{n-1}, s=(s1,,sn1)n1s=(s_{1},\ldots,s_{n-1})\in{\mathbb{C}}^{n-1} and α=(α1,,αn)n\alpha=(\alpha_{1},\ldots,\alpha_{n})\in{\mathbb{C}}^{n} such that

α1++αn=0.\alpha_{1}+\cdots+\alpha_{n}=0.

In such cases, we denote integration over all such variables x=(x1,,xk)x=(x_{1},\ldots,x_{k}) subject to a condition(s) 𝒞=(𝒞1,,𝒞k)\mathcal{C}=(\mathcal{C}_{1},\ldots,\mathcal{C}_{k}) via

𝒞F(x)𝑑x:=𝒞1𝒞kF(x1,,xk)𝑑x1𝑑x2𝑑xk.\int\limits_{\mathcal{C}}F(x)dx:=\int\limits_{\mathcal{C}_{1}}\cdots\int\limits_{\mathcal{C}_{k}}F(x_{1},\ldots,x_{k})\;dx_{1}\,dx_{2}\cdots dx_{k}.

For example, given β=(β1,,βn1)n1\beta=(\beta_{1},\ldots,\beta_{n-1})\in{\mathbb{C}}^{n-1} with β^n1=0\widehat{\beta}_{n-1}=0, we denote integration over all such β\beta with Re(βj)=bj\operatorname{Re}(\beta_{j})=b_{j} for each j=1,,n2j=1,\ldots,n-2 via

β^n1=0Re(β)=bF(β)𝑑β:=Re(β1)=b1Re(βn2)=bn2F(β1,,βn2)𝑑β1𝑑β2𝑑βn2.\int\limits_{\begin{subarray}{c}\widehat{\beta}_{n-1}=0\\ \operatorname{Re}(\beta)=b\end{subarray}}F(\beta)\,d\beta:=\int\limits_{\operatorname{Re}(\beta_{1})=b_{1}}\hskip-3.0pt\cdots\hskip-6.0pt\int\limits_{\operatorname{Re}(\beta_{n-2})=b_{n-2}}F(\beta_{1},\ldots,\beta_{n-2})\;d\beta_{1}\;d\beta_{2}\cdots d\beta_{n-2}.

We extend this notation liberally to integrals over ss, zz and α\alpha and apply it also to integrals over the imaginary parts in the sequel.

Definition 2.3 (Polynomial notation).

Our analysis will often require us to bound certain polynomials in a trivial way. Namely, for complex variables xjx_{j} with j=1,2,,kj=1,2,\ldots,k, if |xj|T1+ε|x_{j}|\ll T^{1+\varepsilon} for each jj and P(x1,x2,,xk)P(x_{1},x_{2},\ldots,x_{k}) is a polynomial, then |P(x1,x2,,xk)|Tε+degP\lvert P(x_{1},x_{2},\ldots,x_{k})\rvert\ll T^{\varepsilon+\deg{P}}. So, the relevant information about PP is its degree. This being the case, we will use the notation 𝒫d(x)\mathcal{P}_{d}(x) (with x=(x1,,xk)x=(x_{1},\ldots,x_{k})) to represent an unspecified polynomial of degree less than or equal to dd in the variable(s) xx. Note that this notation agrees with the commonly employed practice (also used throughout these notes) of using ε\varepsilon to represent an unspecified positive real number whose precise value is not specified and may differ from one usage to another.

Definition 2.4 (Vector or matrix notation depending on context).

Given a vector a=(a1,,an1)n1a=(a_{1},\ldots,a_{n-1})\in\mathbb{R}^{n-1}, we shall define the diagonal matrix

t(a):=diag(a1a2an1,a1a2an2,,a1,1).t(a):=\mathrm{diag}(a_{1}a_{2}\cdots a_{n-1},a_{1}a_{2}\cdots a_{n-2},\ldots,a_{1},1).

2.2. Structure of GL(n)\operatorname{GL}(n)

Suppose nn is a positive integer. Let Un()GL(n,)U_{n}(\mathbb{R})\subseteq\operatorname{GL}(n,\mathbb{R}) denote the set of upper triangular unipotent matrices.

Definition 2.1 (Character of Un()U_{n}(\mathbb{R})).

Let M=(m1,,mn1)n1M=(m_{1},\ldots,m_{n-1})\in{\mathbb{Z}}^{n-1}. For an element xUn()x\in U_{n}(\mathbb{R}) of the form

(2.2) x=(1x1,2x1,3x1,n1x2,3x2,n1xn1,n1),x=\left(\begin{smallmatrix}1&x_{1,2}&x_{1,3}&\cdots&&x_{1,n}\\ &1&x_{2,3}&\cdots&&x_{2,n}\\ &&\hskip 2.0pt\ddots&&&\vdots\\ &&&&1&x_{n-1,n}\\ &&&&&1\end{smallmatrix}\right),

we define the character

(2.3) ψM(x):=m1x1,2+m2x2,3++mn1xn1,n.\psi_{M}(x):=m_{1}x_{1,2}+m_{2}x_{2,3}+\cdots+m_{n-1}x_{n-1,n}.
Definition 2.4 (Generalized upper half plane).

We denote the set of (real) orthogonal matrices O(n,)GL(n,)\text{O}(n,\mathbb{R})\subseteq\operatorname{GL}(n,\mathbb{R}), and we set

𝔥n:=GL(n,)/(O(n,)×).\mathfrak{h}^{n}:=\operatorname{GL}(n,\mathbb{R})/\left(\text{O}(n,\mathbb{R})\cdot\mathbb{R}^{\times}\right).

Every element (via the Iwasawa decomposition of GL(n)\operatorname{GL}(n) [Gol06]) of 𝔥n\mathfrak{h}^{n} has a coset representative of the form g=xyg=xy, with xx as above and

(2.5) y=(y1y2yn1y1y2yn2y11),y=\left(\begin{smallmatrix}y_{1}y_{2}\cdots y_{n-1}&&&\\ &\hskip-30.0pty_{1}y_{2}\cdots y_{n-2}&&\\ &\ddots&&\\ &&\hskip-5.0pty_{1}&\\ &&&1\end{smallmatrix}\right),

where yi>0y_{i}>0 for each 1in11\leq i\leq n-1. The group GL(n,)\operatorname{GL}(n,\mathbb{R}) acts as a group of transformations on 𝔥n\mathfrak{h}^{n} by left multiplication.

Definition 2.6 (Weyl group and relevant elements).

Let WnSnW_{n}\cong S_{n} denote the Weyl group of GL(n,).\operatorname{GL}(n,\mathbb{R}). We consider it as the subgroup of GL(n,)\operatorname{GL}(n,\mathbb{R}) consisting of permutation matrices, i.e., matrices that have exactly one 11 in each row/column and all zeros otherwise. An element wWmw\in W_{m} is called relevant if

w=w(n1,n2,,nr):=(InrIn1),w=w_{(n_{1},n_{2},\ldots,n_{r})}:=\left(\begin{smallmatrix}&&I_{n_{r}}\\ &\displaystyle\cdot^{\displaystyle\cdot^{\displaystyle\cdot}}&\\ I_{n_{1}}&&\end{smallmatrix}\right),

where IniI_{n_{i}} is the identity matrix of size ni×nin_{i}\times n_{i} and n=n1++nrn=n_{1}+\cdots+n_{r} is a composition (a way of writing nn as a sum of positive integers; see Section 8.3). The long element of WnW_{n} is wlong:=w(1,1,,1)w_{\mathrm{long}}:=w_{(1,1,\ldots,1)}.

Definition 2.7 (Other subgroups of GL(n,)\operatorname{GL}(n,\mathbb{R})).

We define

U¯w:=(w1tUn()w)Un(),\overline{U}_{w}:=\Big{(}w^{-1}\cdot\hskip-8.0pt\phantom{U}^{t}U_{n}(\mathbb{R})\cdot w\Big{)}\cap U_{n}(\mathbb{R}),

and

Γw:=(w1tUn()w)Un()=SL(n,)U¯w,\Gamma_{w}:=\Big{(}w^{-1}\cdot\hskip-8.0pt\phantom{U}^{t}U_{n}(\mathbb{Z})\cdot w\Big{)}\cap U_{n}(\mathbb{Z})=\operatorname{SL}(n,{\mathbb{Z}})\cap\overline{U}_{w},

where Unt{}^{t}U_{n} denotes the transpose of UnU_{n}, i.e., the set of lower triangular unipotent matrices.

2.3. Basic functions on the generalized upper half plane 𝔥n\mathfrak{h}^{n}

Definition 2.1 (Power function).

Let α=(α1,,αn)n\alpha=(\alpha_{1},\ldots,\alpha_{n})\in{\mathbb{C}}^{n} with α^n=0\widehat{\alpha}_{n}=0. Let ρ=(ρ1,,ρn)\rho=(\rho_{1},\ldots,\rho_{n}), where ρi=n+12i\rho_{i}=\frac{n+1}{2}-i for i=1,2,,ni=1,2,\ldots,n. We define a power function on xy𝔥nxy\in\mathfrak{h}^{n} by

(2.2) I(xy,α)=i=1ndiαi+ρi=i=1n1yiα^ni+ρ^ni,I(xy,\alpha)=\prod_{i=1}^{n}d_{i}^{\alpha_{i}+\rho_{i}}=\prod_{i=1}^{n-1}y_{i}^{{\widehat{\alpha}_{n-i}}+{\widehat{\rho}_{n-i}}},

where di=jniyjd_{i}=\prod\limits_{j\leq n-i}y_{j} is the jj-th diagonal entry of the matrix g=xyg=xy as above.

Definition 2.3 (Jacquet’s Whittaker function).

Let gGL(n,)g\in\operatorname{GL}(n,\mathbb{R}) with n2n\geq 2. Let α=(α1,α2,,αn)n\alpha=(\alpha_{1},\alpha_{2},\;\ldots,\;\alpha_{n})\in{\mathbb{C}}^{n} with α^n=0\widehat{\alpha}_{n}=0. We define the completed Whittaker function Wα±:GL(n,)/(O(n,)×)W^{\pm}_{\alpha}:\operatorname{GL}(n,\mathbb{R})\big{/}\left(\text{O}(n,\mathbb{R})\cdot\mathbb{R}^{\times}\right)\to\mathbb{C} by the integral

Wα±(g):=1j<knΓ(1+αjαk2)π1+αjαk2U4()I(wlongug,α)ψ1,,1,±1(u)¯𝑑u,W^{\pm}_{\alpha}(g):=\prod_{1\leq j<k\leq n}\frac{\Gamma\big{(}\frac{1+\alpha_{j}-\alpha_{k}}{2}\big{)}}{\pi^{\frac{1+\alpha_{j}-\alpha_{k}}{2}}}\cdot\int\limits_{U_{4}(\mathbb{R})}I(w_{\mathrm{long}}ug,\alpha)\,\overline{\psi_{1,\ldots,1,\pm 1}(u)}\,du,

which converges absolutely if Re(αiαi+1)>0\operatorname{Re}(\alpha_{i}-\alpha_{i+1})>0 for 1in11\leq i\leq n-1 (cf. [GMW21]), and has meromorphic continuation to all αn\alpha\in{\mathbb{C}}^{n} satisfying α^n=0\widehat{\alpha}_{n}=0.

Remark 2.4.

With the additional Gamma factors included in this definition (which can be considered as a “completed” Whittaker function) there are n!n! functional equations which is equivalent to the fact that the Whittaker function is invariant under all permutations of α1,α2,,αn\alpha_{1},\alpha_{2},\ldots,\alpha_{n}. Moreover, even though the integral (without the normalizing factor) often vanishes identically as a function of α\alpha, this normalization never does.

If gg is a diagonal matrix in GL(n,)\operatorname{GL}(n,\mathbb{R}) then the value of Wn,α±(g)W^{\pm}_{n,\alpha}(g) is independent of sign, so we drop the ±\pm. We also drop the ±\pm if the sign is +1+1.

Definition 2.5 (Whittaker transform and its inverse).

Assume n2n\geq 2. Let α=(α1,α2,,αn)n\alpha=(\alpha_{1},\alpha_{2},\ldots,\alpha_{n})\in{\mathbb{C}}^{n} with α^n=0\widehat{\alpha}_{n}=0. Set y:=(y1,y2,yn1)y:=(y_{1},y_{2},\ldots y_{n-1}) and t(y)t(y) as in Definition 2.4. Let f:+n1f:\mathbb{R}_{+}^{n-1}\to\mathbb{C} be an integrable function. Then we define the Whittaker transform f#:Hnf^{\#}:H^{n}\to\mathbb{C} (where Hn:={αnα^n=0}H^{n}:=\{\alpha\in{\mathbb{C}}^{n}\mid\widehat{\alpha}_{n}=0\}) by

(2.6) f#(α):=y1=0yn1=0f(y)Wα(t(y))k=1n1dykykk(nk)+1,f^{\#}(\alpha):=\int\limits_{y_{1}=0}^{\infty}\cdots\int\limits_{y_{n-1}=0}^{\infty}f(y)\,W_{\alpha}\big{(}t(y)\big{)}\prod_{k=1}^{n-1}\frac{dy_{k}}{y_{k}^{k(n-k)+1}},

provided the above integral converges absolutely and uniformly on compact subsets of +n1\mathbb{R}_{+}^{n-1}. The inverse Whittaker transform [GK12, Theorem 1.6] is

f(y)=1πn1α^n=0Re(α)=0f#(α)Wα(t(y))1knΓ(αkα2)𝑑α,f(y)=\frac{1}{\pi^{n-1}}\int\limits_{\begin{subarray}{c}\widehat{\alpha}_{n}=0\\ \operatorname{Re}(\alpha)=0\end{subarray}}\frac{f^{\#}(\alpha)W_{-\alpha}\big{(}t(y)\big{)}}{\prod\limits_{1\leq k\neq\ell\leq n}\Gamma\left(\frac{\alpha_{k}-\alpha_{\ell}}{2}\right)}\,d\alpha,

provided the above integral converges absolutely and uniformly on compact subsets of (i)n(i\mathbb{R})^{n}.

Definition 2.7 (Normalized Poincaré series).

Let M=(m1,m2,,mn1)n1M=(m_{1},m_{2},\ldots,m_{n-1})\in{\mathbb{Z}}^{n-1} with mi0m_{i}\neq 0 for each i=1,,n1i=1,\ldots,n-1. As with yy, we may think of MM as a matrix. Let g𝔥ng\in\mathfrak{h}^{n}. Then we define

(2.8) PM(g,α):=1𝔠nk=1n1mkk(nk)2γUn()\SL(n,)ψM(γg)pT,R(n)(Mγg)I(γg,α),P^{M}(g,\alpha):=\frac{1}{\sqrt{\mathfrak{c}_{n}}}\cdot\prod_{k=1}^{n-1}m_{k}^{-\frac{k(n-k)}{2}}\sum_{\gamma\in U_{n}({\mathbb{Z}})\backslash\operatorname{SL}(n,{\mathbb{Z}})}\psi_{M}(\gamma g)\cdot p_{T,R}^{(n)}(M\gamma g)\cdot I(\gamma g,\alpha),

where 𝔠n\mathfrak{c}_{n} is the (nonzero) constant given in Proposition 4.4. We extend the definition of ψM\psi_{M} and pT,R(n)p_{T,R}^{(n)} to all of 𝔥n\mathfrak{h}^{n} by setting ψM(xy):=ψM(x)\psi_{M}(xy):=\psi_{M}(x) and pT,R(n)(xy):=pT,R(n)(y)p_{T,R}^{(n)}(xy):=p_{T,R}^{(n)}(y).

Remark 2.9.

This definition, up to the normalizing factor 𝔠nk=1n1mkk(nk)/2\sqrt{\mathfrak{c}_{n}}\prod\limits_{k=1}^{n-1}m_{k}^{k(n-k)/2}, of the Poincaré series agrees with that used in [GSW21] with the minor caveat that pT,Rp_{T,R} takes on a slightly different normalization in terms of the polynomial R(n)\mathcal{F}_{R}^{(n)} and in the Gamma factors appearing in Definition 1.4. The normalizing factor is inserted so that in the Kuznetsov trace formula the cuspidal term is precisely the orthogonality relation in Theorem 1.1.

2.4. Fourier expansion of the Poincaré series

Definition 2.1 (Twisted Character).

Let

Vn:={v=(v1v2vn)|v1,,vn{±1},v1vn=1}.V_{n}:=\left\{v=\left.\left(\begin{smallmatrix}v_{1}&&&\\ &v_{2}&&\\ &&\ddots&\\ &&&v_{n}\end{smallmatrix}\right)\right|\;v_{1},\ldots,v_{n}\in\{\pm 1\},\;v_{1}\cdots v_{n}=1\right\}.

Let M=(m1,,mn1)n1,M=(m_{1},\ldots,m_{n-1})\in\mathbb{Z}^{n-1}, and consider ψM\psi_{M} the additive character (see (2.3)) of Un()U_{n}(\mathbb{R}). Then for vVn,v\in V_{n}, we define the twisted character ψMv:Un()\psi_{M}^{v}:U_{n}(\mathbb{R})\to\mathbb{C} by ψMv(g):=ψM(v1gv).\psi_{M}^{v}(g):=\psi_{M}\left(v^{-1}gv\right).

Definition 2.2 (Kloosterman Sum).

Fix L=(1,,n1),M=(m1,,mn1)n1.L=(\ell_{1},\ldots,\ell_{n-1}),\,M=(m_{1},\ldots,m_{n-1})\in\mathbb{Z}^{n-1}. Let ψL,ψM\psi_{L},\psi_{M} be characters of Un().U_{n}(\mathbb{R}). Let wWnw\in W_{n} where WnW_{n} is the Weyl group of GL(n).\operatorname{GL}(n). Let

c=(1/cn1cn1/cn2c2/c1c1)c=\left(\begin{smallmatrix}1/c_{n-1}&&&&\\ &c_{n-1}/c_{n-2}&&&\\ &&\ddots&&\\ &&&c_{2}/c_{1}&\\ &&&&c_{1}\end{smallmatrix}\right)

with ci>0.c_{i}\in{\mathbb{Z}_{>0}}. Then the Kloosterman sum is defined as

Sw(ψL,ψM,c):=γ=Un()\ΓGw/Γwγ=β1cwβ2ψL(β1)ψM(β2),S_{w}(\psi_{L},\psi_{M},c):=\underset{\gamma=\beta_{1}cw\beta_{2}}{\sum\limits_{\gamma=U_{n}(\mathbb{Z})\backslash\Gamma\cap G_{w}/\Gamma_{w}}}\psi_{L}(\beta_{1})\,\psi_{M}(\beta_{2}),

with notation as in Definition 11.2.2 of [Gol06]. The Kloosterman sum Sw(ψ,ψ,c)S_{w}(\psi,\psi^{\prime},c) is well defined (i.e. independent of the choice of Bruhat decomposition for γ\gamma) if and only if it satisfies the compatibility condition ψ(cwuw1)=ψ(u).\psi(cwuw^{-1})=\psi^{\prime}(u). It is defined to be zero otherwise. (See [Fri87].)

Proposition 2.3 (MthM^{\mathrm{th}} Fourier coefficient of the Poincaré series PLP^{L}).

Let L=(1,,n1)L=(\ell_{1},\ldots,\ell_{n-1}) and M=(m1,,mn1)n1M=(m_{1},\ldots,m_{n-1})\in\mathbb{Z}^{n-1} satisfy i=1n1i0\prod\limits_{i=1}^{n-1}\ell_{i}\neq 0 and i=1n1mi0\prod\limits_{i=1}^{n-1}m_{i}\neq 0. If Re(αkαk+1)\operatorname{Re}(\alpha_{k}-\alpha_{k+1}) is sufficiently large for each k=1,,n1k=1,\ldots,n-1, then

Un()\Un()PL(ug,α)ψM(u)¯du=wWnvVnc1=1cn1=1Sw(ψL,ψMv,c)Jw(g;α,ψL,ψMv,c)𝔠nk=1n1(kk(nk)2ckαkαk+1+1),\int\limits_{U_{n}(\mathbb{Z})\backslash U_{n}(\mathbb{R})}P^{L}\left(ug,\,\alpha\right)\cdot\overline{\psi_{M}(u)}\;d^{*}u=\;\sum_{w\in W_{n}}\sum_{v\in V_{n}}\sum_{c_{1}=1}^{\infty}\cdots\sum_{c_{n-1}=1}^{\infty}\frac{S_{w}(\psi_{L},\psi_{M}^{v},c)J_{w}(g;\alpha,\psi_{L},\psi_{M}^{v},c)}{\sqrt{\mathfrak{c}_{n}}\prod\limits_{k=1}^{n-1}\left(\ell_{k}^{\frac{k(n-k)}{2}}c_{k}^{\alpha_{k}-\alpha_{k+1}+1}\right)},

where

Jw(g;α,ψL,ψMv,c)=Uw()\Uw()U¯w()ψL(wug)pT,R(n)(Lcwug)I(wug,α)ψMv(u)¯du,J_{w}(g;\alpha,\psi_{L},\psi_{M}^{v},c)=\int\limits_{U_{w}(\mathbb{Z})\backslash{U}_{w}(\mathbb{R})}\;\int\limits_{\overline{U}_{w}(\mathbb{R})}\psi_{L}(wug)\,p_{T,R}^{(n)}\big{(}Lcwug\big{)}\,I(wug,\alpha)\;\overline{\psi_{M}^{v}(u)}\;d^{*}u,
Uw()=(w1Un()w)Un(),U¯w()=(w1tUn()w)Un(),{U}_{w}(\mathbb{R})=\Big{(}w^{-1}\cdot U_{n}(\mathbb{R})\cdot w\Big{)}\cap U_{n}(\mathbb{R}),\qquad\overline{U}_{w}(\mathbb{R})=\Big{(}w^{-1}\cdot\hskip-10.0pt\phantom{U}^{t}U_{n}(\mathbb{R})\cdot w\Big{)}\cap U_{n}(\mathbb{R}),

and mt\phantom{m}{}^{t}m denotes the transpose of a matrix mm.

Proof.

See Theorem 11.5.4 of [Gol06]. ∎

3. Spectral decomposition of 𝟐(𝐒𝐋(n,)\𝔥n)\mathcal{L}^{2}(\operatorname{SL}(n,{\mathbb{Z}})\backslash\mathfrak{h}^{n})

3.1. Hecke-Maass cusp forms for SL(n,)\operatorname{SL}(n,{\mathbb{Z}})

Definition 3.1 (Langlands parameters).

Let n2n\geq 2. A vector α=(α1,,αn)n\alpha=(\alpha_{1},\ldots,\alpha_{n})\in{\mathbb{C}}^{n} is termed a Langlands parameter if α^n=0\widehat{\alpha}_{n}=0.

Definition 3.2 (Hecke-Maass cusp forms).

Fix n2.n\geq 2. A Hecke-Maass cusp form with Langlands parameter αn\alpha\in{\mathbb{C}}^{n} for SL(n,)\operatorname{SL}(n,\mathbb{Z}) is a smooth function ϕ:𝔥n\phi:\mathfrak{h}^{n}\to\mathbb{C} which satisfies ϕ(γg)=ϕ(g)\phi(\gamma g)=\phi(g) for all γSL(n,)\gamma\in\operatorname{SL}(n,{\mathbb{Z}}), g𝔥ng\in\mathfrak{h}^{n}. In addition ϕ\phi is square integrable, is an eigenfunction of the algebra of Hecke operators on 𝔥n\mathfrak{h}^{n}, and is an eigenfunction of the algebra of GL(n,)\operatorname{GL}(n,\mathbb{R}) invariant differential operators on 𝔥n\mathfrak{h}^{n}, with the same eigenvalues under this action as the power function I(,α)I(*,\alpha). The Laplace eigenvalue of ϕ\phi is given by

n3n24α12+α22++αn22.\frac{n^{3}-n}{24}-\frac{\alpha_{1}^{2}+\alpha_{2}^{2}+\cdots+\alpha_{n}^{2}}{2}.

See Section 6 in [Mil02]. The Hecke-Maass cusp form ϕ\phi is said to be tempered at \infty if the Langlands parameters α1,,αn\alpha_{1},\ldots,\alpha_{n} are all pure imaginary.

Proposition 3.3 (Fourier expansion of Hecke-Maass cusp forms).

Assume n2.n\geq 2. Let ϕ:𝔥n\phi:\mathfrak{h}^{n}\to\mathbb{C} be a Hecke-Maass cusp form for SL(n,)\operatorname{SL}(n,\mathbb{Z}) with Langlands parameters αn\alpha\in{\mathbb{C}}^{n}. Then for g𝔥ng\in\mathfrak{h}^{n}, we have the following Fourier-Whittaker expansion:

ϕ(g)=γUn1()\SLn1()m1=1mn2=1mn10Aϕ(M)k=1n1|mk|k(nk)2Wαsgn(mn1)(t(M)(γ001)g),\phi(g)=\sum_{\gamma\in{U}_{n-1}(\mathbb{Z})\backslash\operatorname{SL}_{n-1}(\mathbb{Z})}\;\sum_{m_{1}=1}^{\infty}\cdots\sum_{m_{n-2}=1}^{\infty}\;\sum_{m_{n-1}\neq 0}\,\frac{A_{\phi}(M)}{\prod\limits_{k=1}^{n-1}|m_{k}|^{\frac{k(n-k)}{2}}}\;W^{\text{\rm sgn}(m_{n-1})}_{\alpha}\left(t(M)\bigg{(}\begin{matrix}\gamma&0\\ 0&1\end{matrix}\bigg{)}g\right),

where M=(m1,m2,,mn1)M=(m_{1},m_{2},\;\ldots,\;m_{n-1}), t(M)t(M) is the toric matrix in Definition 2.4 and Aϕ(M)A_{\phi}(M) is the MthM^{th} Fourier coefficient of ϕ\phi.

Proof.

See Section 9.1 of [Gol15]. ∎

Definition 3.4 (L-function associated to a Hecke-Maass form ϕ\boldsymbol{\phi}).

Let ss\in\mathbb{C} with Re(s)\operatorname{Re}(s) sufficiently large. Then the L-function associated to a Hecke-Maass cusp form ϕ\phi is defined as

L(s,ϕ):=m=1Aϕ(m,1,,1)msL(s,\phi):=\sum_{m=1}^{\infty}\frac{A_{\phi}(m,1,\ldots,1)}{m^{s}}

and has holomorphic continuation to all ss\in\mathbb{C} and satisfies a functional equation s1s.s\to 1-s. If ϕ\phi is a simultaneous eigenfunction of all the Hecke operators then L(s,ϕ)L(s,\phi) has the following Euler product:

L(s,ϕ)=p(1A(p,1,,1)ps+A(1,p,1,,1)p2sA(1,1,p,,1)p3s\displaystyle L(s,\phi)=\prod_{p}\Bigg{(}1-\frac{A(p,1,\ldots,1)}{p^{s}}+\frac{A(1,p,1,\ldots,1)}{p^{2s}}-\frac{A(1,1,p,\ldots,1)}{p^{3s}}
++(1)n1A(1,,,1,p)p(n1)s+(1)npns)1.\displaystyle\hskip 170.0pt+\;\;\;\cdots\;\;\;+\;(-1)^{n-1}\frac{A(1,,\ldots,1,p)}{p^{(n-1)s}}+\frac{(-1)^{n}}{p^{ns}}\Bigg{)}^{-1}.

3.2. Langlands Eisenstein series for SL(n,)\operatorname{SL}(n,\mathbb{Z})

Definition 3.1 (Parabolic Subgroup).

For n2n\geq 2 and 1rn,1\leq r\leq n, consider a partition of nn given by n=n1++nrn=n_{1}+\cdots+n_{r} with positive integers n1,,nr.n_{1},\cdots,n_{r}. We define the standard parabolic subgroup

𝒫:=𝒫n1,n2,,nr:={(GL(n1)0GL(n2)00GL(nr))}.\mathcal{P}:=\mathcal{P}_{n_{1},n_{2},\ldots,n_{r}}:=\left\{\left(\begin{matrix}\operatorname{GL}(n_{1})&*&\cdots&*\\ 0&\operatorname{GL}(n_{2})&\cdots&*\\ \vdots&\vdots&\ddots&\vdots\\ 0&0&\cdots&\operatorname{GL}(n_{r})\end{matrix}\right)\right\}.

Letting IrI_{r} denote the r×rr\times r identity matrix, the subgroup

N𝒫:={(In10In200Inr)}N^{\mathcal{P}}:=\left\{\left(\begin{matrix}I_{n_{1}}&*&\cdots&*\\ 0&I_{n_{2}}&\cdots&*\\ \vdots&\vdots&\ddots&\vdots\\ 0&0&\cdots&I_{n_{r}}\end{matrix}\right)\right\}

is the unipotent radical of 𝒫\mathcal{P}. The subgroup

M𝒫:={(GL(n1)000GL(n2)000GL(nr))}M^{\mathcal{P}}:=\left\{\left(\begin{matrix}\operatorname{GL}(n_{1})&0&\cdots&0\\ 0&\operatorname{GL}(n_{2})&\cdots&0\\ \vdots&\vdots&\ddots&\vdots\\ 0&0&\cdots&\operatorname{GL}(n_{r})\end{matrix}\right)\right\}

is the standard choice of Levi subgroup of 𝒫\mathcal{P}.

Definition 3.2 (Hecke-Maass form Φ\Phi associated to a parabolic 𝒫\mathcal{P}).

Let n2n\geq 2. Consider a partition n=n1++nrn=n_{1}+\cdots+n_{r} with 1<r<n1<r<n. Let 𝒫:=𝒫n1,n2,,nrGL(n,).\mathcal{P}:=\mathcal{P}_{n_{1},n_{2},\ldots,n_{r}}\subset\operatorname{GL}(n,\mathbb{R}). For i=1,2,,ri=1,2,\ldots,r, let ϕi:GL(ni,)\phi_{i}:\operatorname{GL}(n_{i},\mathbb{R})\to\mathbb{C} be either the constant function 1 (if ni=1n_{i}=1) or a Hecke-Maass cusp form for SL(ni,)\operatorname{SL}(n_{i},\mathbb{Z}) (if ni>1n_{i}>1). The form Φ:=ϕ1ϕr\Phi:=\phi_{1}\otimes\cdots\otimes\phi_{r} is defined on GL(n,)=𝒫()\operatorname{GL}(n,\mathbb{R})=\mathcal{P}(\mathbb{R}) (where K=O(n,)K={\rm O}(n,\mathbb{R})) by the formula

Φ(nmk):=i=1rϕi(mi),(nN𝒫,mM𝒫,kK)\Phi(nmk):=\prod_{i=1}^{r}\phi_{i}(m_{i}),\qquad(n\in N^{\mathcal{P}},m\in M^{\mathcal{P}},k\in K)

where mM𝒫m\in M^{\mathcal{P}} has the form m=(m1000m2000mr)m=\left(\begin{smallmatrix}m_{1}&0&\cdots&0\\ 0&m_{2}&\cdots&0\\ \vdots&\vdots&\ddots&\vdots\\ 0&0&\cdots&m_{r}\end{smallmatrix}\right), with miGL(ni,).m_{i}\in\operatorname{GL}({n_{i}},\mathbb{R}). In fact, this construction works equally well if some or all of the ϕi\phi_{i} are Eisenstein series.

Definition 3.3 (Character of a parabolic subgroup).

Let n2.n\geq 2. Fix a partition n=n1+n2++nrn=n_{1}+n_{2}+\cdots+n_{r} with associated parabolic subgroup 𝒫:=𝒫n1,n2,,nr.\mathcal{P}:=\mathcal{P}_{n_{1},n_{2},\ldots,n_{r}}. Define

(3.4) ρ𝒫(j)={nn12,j=1nnj2n1nj1,j2.\rho_{{}_{\mathcal{P}}}(j)=\left\{\begin{array}[]{ll}\frac{n-n_{1}}{2},&j=1\\ \frac{n-n_{j}}{2}-n_{1}-\cdots-n_{j-1},&j\geq 2.\end{array}\right.

Let s=(s1,s2,,sr)rs=(s_{1},s_{2},\ldots,s_{r})\in\mathbb{C}^{r} satisfy i=1rnisi=0.\sum\limits_{i=1}^{r}n_{i}s_{i}=0. Consider the function (see Definition 2.1)

||𝒫s:=I(,α)|\cdot|_{{}_{\mathcal{P}}}^{s}:=I(\cdot,\alpha)

on GL(n,)\operatorname{GL}(n,\mathbb{R}), where

α=(s1ρ𝒫(1)+1n12,s1ρ𝒫(1)+3n12,,s1ρ𝒫(1)+n112n1terms,s2ρ𝒫(2)+1n22,s2ρ𝒫(2)+3n22,,s2ρ𝒫(2)+n212n2terms,,srρ𝒫(r)+1nr2,srρ𝒫(r)+3nr2,,srρ𝒫(r)+nr12nrterms).\alpha=(\overbrace{s_{1}-\rho_{{}_{\mathcal{P}}}(1)+{\textstyle{\frac{1-n_{1}}{2}}},\;s_{1}-\rho_{{}_{\mathcal{P}}}(1)+{\textstyle{\frac{3-n_{1}}{2}}},\;\ldots\;,s_{1}-\rho_{{}_{\mathcal{P}}}(1)+{\textstyle{\frac{n_{1}-1}{2}}}}^{n_{1}\;\,\text{\rm terms}},\\ \overbrace{s_{2}-\rho_{{}_{\mathcal{P}}}(2)+{\textstyle{\frac{1-n_{2}}{2}}},\;s_{2}-\rho_{{}_{\mathcal{P}}}(2)+{\textstyle{\frac{3-n_{2}}{2}}},\;\ldots\;,s_{2}-\rho_{{}_{\mathcal{P}}}(2)+{\textstyle{\frac{n_{2}-1}{2}}}}^{n_{2}\;\,\text{\rm terms}},\\ \vdots\\ \ldots\;\;,\overbrace{s_{r}-\rho_{{}_{\mathcal{P}}}(r)+{\textstyle{\frac{1-n_{r}}{2}}},\;s_{r}-\rho_{{}_{\mathcal{P}}}(r)+{\textstyle{\frac{3-n_{r}}{2}}},\;\ldots\;,s_{r}-\rho_{{}_{\mathcal{P}}}(r)+{\textstyle{\frac{n_{r}-1}{2}}}}^{n_{r}\;\,\text{\rm terms}}).

The conditions i=1rnisi=0\sum\limits_{i=1}^{r}n_{i}s_{i}=0 and i=1rniρ𝒫(i)=0\sum\limits_{i=1}^{r}n_{i}\rho_{{}_{\mathcal{P}}}(i)=0 guarantee that α\alpha’s entries sum to zero. When g𝒫g\in\mathcal{P}, with diagonal block entries miGL(ni,)m_{i}\in\operatorname{GL}(n_{i},\mathbb{R}), one has

|g|𝒫s=i=1r|det(mi)|si,|g|_{{}_{\mathcal{P}}}^{s}=\prod_{i=1}^{r}\left|\text{\rm det}(m_{i})\right|^{s_{i}},

so that ||𝒫s|\cdot|_{{}_{\mathcal{P}}}^{s} restricts to a character of 𝒫\mathcal{P} which is trivial on N𝒫N^{\mathcal{P}}.

Definition 3.5 (Langlands Eisenstein series twisted by Hecke-Maass forms of lower rank).

Let Γ=SL(n,)\Gamma=\operatorname{SL}(n,\mathbb{Z}) with n2.n\geq 2. Consider a parabolic subgroup 𝒫=𝒫n1,,nr\mathcal{P}=\mathcal{P}_{n_{1},\ldots,n_{r}} of GL(n,)\operatorname{GL}(n,\mathbb{R}) and functions Φ\Phi and ||𝒫s|\cdot|_{{}_{\mathcal{P}}}^{s} as given in Definitions 3.2 and 3.3, respectively. Let

s=(s1,s2,,sr)r,wherei=1rnisi=0.s=(s_{1},s_{2},\ldots,s_{r})\in\mathbb{C}^{r},\;\;\text{where}\;\;\sum_{i=1}^{r}n_{i}s_{i}=0.

The Langlands Eisenstein series determined by this data is defined by

(3.6) E𝒫,Φ(g,s):=γ(𝒫Γ)\ΓΦ(γg)|γg|𝒫s+ρ𝒫E_{\mathcal{P},\Phi}(g,s):=\sum_{\gamma\,\in\,(\mathcal{P}\,\cap\,\Gamma)\backslash\Gamma}\Phi(\gamma g)\cdot|\gamma g|^{s+\rho_{{}_{\mathcal{P}}}}_{{}_{\mathcal{P}}}

as an absolutely convergent sum for Re(si)\operatorname{Re}(s_{i}) sufficiently large, and extends to all srs\in{\mathbb{C}}^{r} by meromorphic continuation.

For k=1,2,,r,k=1,2,\ldots,r, let α(k):=(αk,1,,αk,nk)\alpha^{(k)}:=(\alpha_{k,1},\ldots,\alpha_{k,n_{k}}) denote the Langlands parameters of ϕk.\phi_{k}. We adopt the convention that if nk=1n_{k}=1 then αk,1=0.\alpha_{k,1}=0. Then the Langlands parameters of E𝒫,Φ(g,s)E_{\mathcal{P},\Phi}(g,s) (denoted α𝒫,Φ(s)\alpha_{{}_{\mathcal{P},\Phi}}(s)) are

(3.7) (α1,1+s1,,α1,n1+s1n1terms,α2,1+s2,,α2,n2+s2n2terms,,αr,1+sr,,αr,nr+srnrterms).\bigg{(}\overbrace{\alpha_{1,1}+s_{1},\;\ldots\;,\alpha_{1,n_{1}}+s_{1}}^{n_{1}\;\,\text{\rm terms}},\quad\overbrace{\alpha_{2,1}+s_{2},\;\ldots\;,\alpha_{2,n_{2}}+s_{2}}^{n_{2}\;\,\text{\rm terms}},\quad\ldots\quad,\overbrace{\alpha_{r,1}+s_{r},\;\ldots\;,\alpha_{r,n_{r}}+s_{r}}^{n_{r}\;\,\text{\rm terms}}\bigg{)}.\phantom{xxxxx}
Definition 3.8.

(The M𝐭𝐡M^{\rm{th}} Fourier coefficient of E𝒫,𝚽E_{\mathcal{P},\Phi}) Let s=(s1,s2,,sr)r,s=(s_{1},s_{2},\ldots,s_{r})\in\mathbb{C}^{r}, where i=1rnisi=0.\sum\limits_{i=1}^{r}n_{i}s_{i}=0. Consider E𝒫,Φ(,s)E_{\mathcal{P},\Phi}(*,s) with associated Langlands parameters α𝒫,Φ(s)\alpha_{{}_{\mathcal{P},\Phi}}(s) as defined in (3.7). Let M=(m1,m2,,mn1)>0n1M=(m_{1},m_{2},\ldots,m_{n-1})\in\mathbb{Z}_{>0}^{n-1}. Then the MthM^{th} term in the Fourier-Whittaker expansion of E𝒫,ΦE_{\mathcal{P},\Phi} is

Un()\Un()E𝒫,Φ(ug,s)ψM(u)¯𝑑u=AE𝒫,Φ(M,s)k=1n1mkk(nk)/2Wα𝒫,Φ(s)(Mg),\displaystyle\int\limits_{U_{n}(\mathbb{Z})\backslash U_{n}(\mathbb{R})}E_{\mathcal{P},\Phi}(ug,s)\,\overline{\psi_{M}(u)}\;du\;=\;\frac{A_{E_{\mathcal{P},\Phi}}(M,s)}{\prod\limits_{k=1}^{n-1}m_{k}^{k(n-k)/2}}\;W_{\alpha_{{}_{\mathcal{P},\Phi}}(s)}\big{(}Mg\big{)},

3.3. Langlands spectral decomposition for SL(n,)\operatorname{SL}(n,\mathbb{Z})

Definition 3.1 (Petersson inner product).

Let n2n\geq 2. For F,G2(SL(n,)\𝔥n)F,G\in\mathcal{L}^{2}(\operatorname{SL}(n,{\mathbb{Z}})\backslash\mathfrak{h}^{n}) we define the Petersson inner product to be

F,G:=SL(n,)\𝔥nF(g)G(g)¯𝑑g.\big{\langle}F,G\big{\rangle}:=\int\limits_{\operatorname{SL}(n,{\mathbb{Z}})\backslash\mathfrak{h}^{n}}F(g)\overline{G(g)}\,dg.

For g=xy𝔥ng=xy\in\mathfrak{h}^{n}, with

x=(1x1,2x1,3x1,n1x2,3x2,n1xn1,n1),y=(y1y2yn1y1y2yn2y11),x=\left(\begin{smallmatrix}1&x_{1,2}&x_{1,3}&\cdots&&x_{1,n}\\ &1&x_{2,3}&\cdots&&x_{2,n}\\ &&\hskip 2.0pt\ddots&&&\vdots\\ &&&&1&x_{n-1,n}\\ &&&&&1\end{smallmatrix}\right),\qquad y=\left(\begin{smallmatrix}y_{1}y_{2}\cdots y_{n-1}&&&\\ &\hskip-30.0pty_{1}y_{2}\cdots y_{n-2}&&\\ &\ddots&&\\ &&\hskip-5.0pty_{1}&\\ &&&1\end{smallmatrix}\right),

the measure dgdg is given by dxdydx\,dy, with

dx=1i<jndxi,j,dy=k=1n1dykyk(nk)+1.dx=\prod_{1\leq i{<}j\leq n}dx_{i,j},\qquad dy=\prod_{k=1}^{n-1}\frac{dy_{k}}{y^{k(n-k)+1}}.

The Langlands spectral decomposition for SL(n,)\operatorname{SL}(n,\mathbb{Z}) states that

2(SL(n,)\𝔥n)=(Cuspidal spectrum)(Residual spectrum)(Continuous spectrum).\boxed{\mathcal{L}^{2}(\operatorname{SL}(n,\mathbb{Z})\backslash\mathfrak{h}^{n})=\text{(Cuspidal spectrum)}\oplus\text{(Residual spectrum)}\oplus\text{(Continuous spectrum)}}.

We shall be applying the Langlands spectral decomposition to Poincaré series which are orthogonal to the residual spectrum.

Theorem 3.2 (Langlands spectral decomposition for SL(n,)\operatorname{SL}(n,\mathbb{Z})).

Let ϕ1,ϕ2,ϕ3,\phi_{1},\phi_{2},\phi_{3},\ldotsdenote an orthogonal basis of Hecke-Maass forms for SL(n,)\operatorname{SL}(n,\mathbb{Z}). Assume that F,G2(SL(n,)\𝔥n)F,G\in\mathcal{L}^{2}(\operatorname{SL}(n,\mathbb{Z})\backslash\mathfrak{h}^{n}) are orthogonal to the residual spectrum. Then for gGL(n,)g\in\operatorname{GL}(n,\mathbb{R}) we have

F(g)=j=1F,ϕjϕj(g)ϕj,ϕj+𝒫Φc𝒫n1s1++nrsr=0Re(s)=0F,E𝒫,Φ(,s)E𝒫,Φ(g,s)ds,\displaystyle F(g)=\sum_{j=1}^{\infty}\langle F,\phi_{j}\rangle\frac{\phi_{j}(g)}{\langle\phi_{j},\phi_{j}\rangle}\;+\;\sum_{\mathcal{P}}\sum_{\Phi}\;c_{\mathcal{P}}\hskip-10.0pt\underset{\text{\rm Re}(s)=0}{\int\limits_{n_{1}s_{1}+\cdots+n_{r}s_{r}=0}}\hskip-14.0pt\Big{\langle}F,E_{\mathcal{P},\Phi}(*\,,s)\Big{\rangle}E_{\mathcal{P},\Phi}(g\,,s)\;ds,
F,G=j=1F,ϕjϕj,Gϕj,ϕj+𝒫Φc𝒫n1s1++nrsr=0Re(s)=0F,E𝒫,Φ(,s)E𝒫,Φ(,s),Gds,\displaystyle\langle F,G\rangle=\sum_{j=1}^{\infty}\frac{\langle F,\phi_{j}\rangle\,\langle\phi_{j},G\rangle}{\langle\phi_{j},\phi_{j}\rangle}\;+\sum_{\mathcal{P}}\sum_{\Phi}\;c_{\mathcal{P}}\hskip-10.0pt\underset{\text{\rm Re}(s)=0}{\int\limits_{n_{1}s_{1}+\cdots+n_{r}s_{r}=0}}\hskip-10.0pt\Big{\langle}F,\,E_{\mathcal{P},\Phi}(*\,,s)\Big{\rangle}\Big{\langle}E_{\mathcal{P},\Phi}(*\,,s),\,G\Big{\rangle}\;ds,

where the sum over 𝒫\mathcal{P} ranges over parabolics associated to partitions n=k=1rnkn=\sum\limits_{k=1}^{r}n_{k}, while the sum over Φ\Phi (see Definition 3.2) ranges over an orthonormal basis of Hecke-Maass forms associated to 𝒫\mathcal{P}. Furthermore, c𝒫c_{\mathcal{P}} is a fixed non-zero constant.

Proof.

For proofs see [Art79], [Lan76], [MW95]. ∎

4. Kuznetsov trace formula

The Kuznetsov trace formula is derived by computing the inner product of two Poincaré series in two different ways. More precisely, let L=(1,,m1),M=(m1,,mn1)n1L=(\ell_{1},\ldots,\ell_{m-1}),M=(m_{1},\ldots,m_{n-1})\in{\mathbb{Z}}^{n-1} with i=1n1mi0\prod\limits_{i=1}^{n-1}m_{i}\neq 0 and i=1n1i0\prod\limits_{i=1}^{n-1}\ell_{i}\neq 0, and consider the Petersson inner product PL,PM\big{\langle}P^{L},P^{M}\big{\rangle}.

In particular since PL,PM2(SL(n,)\𝔥n)P^{L},P^{M}\in\mathcal{L}^{2}\left(\operatorname{SL}(n,\mathbb{Z})\backslash\mathfrak{h}^{n}\right) (see [Fri87]), the inner product can be computed with the spectral expansion of the Poincaré series. The geometric approach utilizes the Fourier Whittaker expansion of the Poincaré series which involve Kloosterman sums.

The trace formula takes the following form.

(4.1) 𝒞+spectral side=+𝒦geometric side.\boxed{\boxed{\underset{\mbox{\scriptsize{spectral side}}}{\underbrace{\mathcal{C}\;+\;\mathcal{E}}}\;\;=\;\;\underset{\mbox{\scriptsize{geometric side}}}{\underbrace{\mathcal{M}\;+\;\mathcal{K}}}.}}

Here 𝒞\mathcal{C} is the cuspidal contribution and \mathcal{E} is the Eisenstein contribution. See Theorem 4.1 for their precise definitions. The geometric side consists of terms corresponding to elements of the Weyl group. The identity element gives the main term \mathcal{M}, and the Kloosterman contribution 𝒦\mathcal{K} is the sum of the remaining terms. See Theorem 4.1 for their precise definitions. The Kloosterman term 𝒦\mathcal{K} and the Eisenstein contribution \mathcal{E} will be small with the special choice of the test function pT,Rp_{T,R}, and they constitute the error term in the main theorem.

4.1. Spectral side of the Kuznetsov trace formula

The first way to compute the inner product of the Poincaré series uses the spectral decomposition of the Poincaré series.

Recall also the definition of the adjoint L-function: L(s,Adϕ):=L(s,ϕ×ϕ¯)/ζ(s)L(s,\operatorname{Ad}{\phi}):=L(s,\phi\times\overline{\phi})/\zeta(s) where L(s,ϕ×ϕ¯)L(s,\phi\times\overline{\phi}) is the Rankin-Selberg convolution L-function as in §12.1 of [Gol15].

Theorem 4.1 (Spectral decomposition for the inner product of Poincaré series).

Fix n2n\geq 2 and L=(1,,n1),M=(m1,,mn1)n1L=(\ell_{1},\;\ldots\;,\ell_{n-1}),\,M=(m_{1},\;\ldots\;,m_{n-1})\in\mathbb{Z}^{n-1} . Then For α0:=(n12+j1)j=1,,n\alpha_{0}:=\big{(}-\tfrac{n-1}{2}+j-1\big{)}_{j=1,\ldots,n} we have

PL(,α0),PM(,α0)=𝒞+.\boxed{\ \Big{\langle}P^{L}(*,\alpha_{0}),\;P^{M}(*,\alpha_{0})\Big{\rangle}=\mathcal{C}+\mathcal{E}.\ }

With the notation of the Spectral Decomposition Theorem 3.2, the cuspidal contribution to the Kuznetsov trace formula is

𝒞:=i=1λϕi(M)λϕi(L)¯|pT,Rn,#(α(i))|2L(1,Adϕi)1jknΓ(1+αj(i)αk(i)2),\displaystyle\mathcal{C}:=\;\sum_{i=1}^{\infty}\frac{\lambda_{\phi_{i}}(M)\overline{\lambda_{\phi_{i}}(L)}\cdot\left|p_{T,R}^{n,\#}\left(\,\alpha^{(i)}\,\right)\right|^{2}}{L(1,\operatorname{Ad}{\phi_{i}})\cdot\prod\limits_{1\leq j\neq k\leq n}\Gamma\left(\frac{1+\alpha_{j}^{(i)}-\alpha_{k}^{(i)}}{2}\right)},

and the Eisenstein contribution to the Kuznetsov trace formula is

:=𝒫Φc𝒫n1s1++nrsr=0Re(sj)=0AE𝒫,Φ(L,s)AE𝒫,Φ(M,s)¯|pT,Rn,#(α(𝒫,Φ)(s))|2ds,\mathcal{E}:=\sum_{\mathcal{P}}\sum_{\Phi}\;c_{\mathcal{P}}\hskip-4.0pt\underset{\text{\rm Re}(s_{j})=0}{\int\limits_{n_{1}s_{1}+\cdots+n_{r}s_{r}=0}}\hskip-11.0ptA_{E_{\mathcal{P},\Phi}}(L,s)\,\overline{A_{E_{\mathcal{P},\Phi}}(M,s)}\cdot\Big{|}p_{T,R}^{n,\#}\big{(}\alpha_{{}_{(\mathcal{P},\Phi)}}(s)\big{)}\Big{|}^{2}\;ds,

for constants c𝒫>0c_{\mathcal{P}}>0.

Proof.

The proof follows from the Langlands Spectral Decomposition Theorem 3.2 with the choices F=PLF=P^{L} and G=PMG=P^{M}. We have

PL,PM=j=1PL,ϕjϕj,PMϕj,ϕj+𝒫Φc𝒫n1s1++nrsr=0Re(sj)=0F,E𝒫,Φ(,s)E𝒫,Φ(,s),Gds.\displaystyle\langle P^{L},P^{M}\rangle=\sum_{j=1}^{\infty}\frac{\langle P^{L},\phi_{j}\rangle\,\langle\phi_{j},P^{M}\rangle}{\langle\phi_{j},\phi_{j}\rangle}\;+\sum_{\mathcal{P}}\sum_{\Phi}\;c_{\mathcal{P}}\hskip-10.0pt\underset{\text{\rm Re}(s_{j})=0}{\int\limits_{n_{1}s_{1}+\cdots+n_{r}s_{r}=0}}\hskip-10.0pt\Big{\langle}F,\,E_{\mathcal{P},\Phi}(*\,,s)\Big{\rangle}\Big{\langle}E_{\mathcal{P},\Phi}(*\,,s),\,G\Big{\rangle}\;ds.

We then insert the inner products given in Proposition 4.2 below. Doing so, we see that the cuspidal spectrum is

i=1PL,ϕiϕi,PMϕi,ϕi\displaystyle\sum_{i=1}^{\infty}\frac{\big{\langle}P^{L},\phi_{i}\big{\rangle}\big{\langle}\phi_{i},P^{M}\big{\rangle}}{\langle\phi_{i},\phi_{i}\rangle} =i=1Aϕi(M)Aϕi(L)¯𝔠nϕi,ϕi|pT,Rn,#(α(i))|2.\displaystyle=\sum_{i=1}^{\infty}\frac{A_{\phi_{i}}(M)\overline{A_{\phi_{i}}(L)}}{\mathfrak{c}_{n}\cdot\langle\phi_{i},\phi_{i}\rangle}\left|p_{T,R}^{n,\#}(\alpha^{(i)})\right|^{2}.

From Proposition 4.4, we see that

Aϕ(M)Aϕ(L)¯=|Aϕ(1)|2λϕ(M)λϕ(L)¯=𝔠nϕ,ϕλϕ(M)λϕ(L)¯L(1,Adϕ)1jknΓ(1+αjαk2).A_{\phi}(M)\overline{A_{\phi}(L)}=\lvert A_{\phi}(1)\rvert^{2}\lambda_{\phi}(M)\overline{\lambda_{\phi}(L)}=\frac{\mathfrak{c}_{n}\cdot\big{\langle}\phi,\,\phi\big{\rangle}\cdot\lambda_{\phi}(M)\overline{\lambda_{\phi}(L)}}{L(1,\operatorname{Ad}\;\phi)\prod\limits_{1\leq j\neq k\leq n}\Gamma\big{(}\frac{1+\alpha_{j}-\alpha_{k}}{2}\big{)}}.

The cuspidal part is now immediate. The contributions from the Eisenstein series are computed in like manner using Proposition 4.5. ∎

Proposition 4.2 (The inner product of PMP^{M} with an Eisenstein series or Hecke-Maass form).

Let M=(m1,m2,,mn1)M=(m_{1},m_{2},\ldots,m_{n-1}). Consider the Eisenstein series E𝒫,Φ(,s)E_{\mathcal{P},\Phi}(*,s), with associated Langlands parameters α𝒫,Φ(s)\alpha_{{}_{\mathcal{P},\Phi}}(s). Let ϕ\phi denote a Hecke-Maass cusp form for SL(n,)\operatorname{SL}(n,\mathbb{Z}) with Langlands parameter α\alpha and MthM^{th} Fourier coefficient Aϕ(M).A_{\phi}(M). Then for α0:=(n12+j1)j=1,,n\alpha_{0}:=\big{(}-\tfrac{n-1}{2}+j-1\big{)}_{j=1,\ldots,n},

ϕ,PM(,α0)\displaystyle\Big{\langle}\phi,\;P^{M}(*,\alpha_{0})\Big{\rangle} =1𝔠nAϕ(M)pT,Rn,#(α),\displaystyle=\;\frac{1}{\sqrt{\mathfrak{c}_{n}}}\;A_{\phi}(M)\cdot p_{T,R}^{n,\#}(\alpha),
E𝒫,Φ(,s),PM(,α0)\displaystyle\Big{\langle}E_{\mathcal{P},\Phi}(*,s),\;P^{M}(*,\alpha_{0})\Big{\rangle} =1𝔠nAE𝒫,Φ(M,s)pT,Rn,#(α𝒫,Φ(s)),\displaystyle=\;\frac{1}{\sqrt{\mathfrak{c}_{n}}}\;A_{E_{\mathcal{P},\Phi}}(M,s)\cdot p_{T,R}^{n,\#}(\alpha_{{}_{\mathcal{P},\Phi}}(s)),

where the inner products on the left are defined by analytic continuation and 𝔠n\mathfrak{c}_{n} is the nonzero constant (depending only on nn) from Proposition 4.4.

Proof.

We outline the case of the Hecke-Maass forms. The series definition of the Poincaré series converges absolutely for sufficiently large Re(αiαi+1)\operatorname{Re}(\alpha_{i}^{\prime}-\alpha_{i+1}^{\prime}) (1in11\leq i\leq n-1). It follows that for such α\alpha^{\prime} we may unravel the Poincaré series PM(,α)P^{M}(*,\alpha^{\prime}) in the inner product ϕ,PM\langle\phi,P^{M}\rangle with the Rankin-Selberg Method. The inner product picks out the MthM^{th} Fourier coefficient of ϕ\phi multiplied by a certain Whittaker transform of pT,R(n)(My)I(y,α)p_{T,R}^{(n)}\big{(}My\big{)}\cdot I(y,\alpha^{\prime}). This Whittaker transform has analytic continuation in α\alpha^{\prime} to a region including α0\alpha_{0}. For sufficiently large Re(αiαi+1)\operatorname{Re}(\alpha_{i}^{\prime}-\alpha_{i+1}^{\prime}), we have from (2.8) that

(4.3) ϕ,PM(,α)\displaystyle\big{\langle}\phi,\;P^{M}(*,\alpha^{\prime})\big{\rangle}\; =Aϕ(M)𝔠nk=1n1mkk(nk)y1=0yn1=0pT,R(n)(My)I(y,α)¯Wα(My)k=1n1dykykk(nk)+1.\displaystyle=\;\frac{A_{\phi}(M)}{\sqrt{\mathfrak{c}_{n}}\prod\limits_{k=1}^{n-1}m_{k}^{{k(n-k)}}}\int\limits_{y_{1}=0}^{\infty}\cdots\int\limits_{y_{n-1}=0}^{\infty}\overline{p_{T,R}^{(n)}\big{(}My\big{)}{\cdot I(y,\alpha^{\prime})}}\cdot W_{\alpha}\big{(}My\big{)}\;\prod_{k=1}^{n-1}\frac{dy_{k}}{y_{k}^{k(n-k)+1}}.

Note that I(y,α0)=1I(y,\alpha_{0})=1. The integral in (4.3) converges (as a function of α\alpha^{\prime}) to a region which includes α0\alpha_{0}. It follows that the analytic continuation in α\alpha^{\prime} to α0\alpha_{0} of the inner product satisfies

ϕ,PM(,α0)=1𝔠nAϕ(M)pT,Rn,#(α).\big{\langle}\phi,\;P^{M}(*,\alpha_{0})\big{\rangle}\;=\;\frac{1}{\sqrt{\mathfrak{c}_{n}}}\cdot A_{\phi}(M)\cdot p_{T,R}^{n,\#}(\alpha).

The proof for E𝒫,ΦE_{\mathcal{P},\Phi} is the same. ∎

For n2n\geq 2, consider a Hecke-Maass cusp form ϕ\phi for SL(n,)\operatorname{SL}(n,\mathbb{Z}) with Fourier Whittaker expansion given by Proposition 3.3. Assume ϕ\phi is a Hecke eigenform. Let Aϕ(1):=Aϕ(1,1,,1)A_{\phi}(1):=A_{\phi}(1,1,\ldots,1) denote the first Fourier-Whittaker coefficient of ϕ.\phi. Then we have

Aϕ(M)=Aϕ(1)λϕ(M)A_{\phi}(M)=A_{\phi}(1)\cdot\lambda_{\phi}(M)

where λϕ(M)\lambda_{\phi}(M) is the Hecke eigenvalue (see Section 9.3 in [Gol15]), and λϕ(1)=1\lambda_{\phi}(1)=1.

Proposition 4.4 (First Fourier-Whittaker coefficient of a Hecke-Maass cusp form).

Assume n2.n\geq 2. Let ϕ\phi be a Hecke-Maass cusp form for SL(n,)\operatorname{SL}(n,\mathbb{Z}) with Langlands parameters α=(α1,,αn)\alpha=(\alpha_{1},\ldots,\alpha_{n}). Then the first coefficient Aϕ(1)A_{\phi}(1) is given by

|Aϕ(1)|2=𝔠nϕ,ϕL(1,Adϕ)1jknΓ(1+αjαk2)|A_{\phi}(1)|^{2}=\frac{\mathfrak{c}_{n}\cdot\big{\langle}\phi,\,\phi\big{\rangle}}{L(1,\operatorname{Ad}{\phi})\prod\limits_{1\leq j\neq k\leq n}\Gamma\big{(}\frac{1+\alpha_{j}-\alpha_{k}}{2}\big{)}}

where 𝔠n0\mathfrak{c}_{n}\neq 0 is a constant depending on nn only.

Proof.

See [GMW21]. ∎

Proposition 4.5 (The M𝐭𝐡M^{\rm{th}} Fourier coefficient of E𝒫,𝚽E_{\mathcal{P},\Phi}).

Let s=(s1,s2,,sr)r,s=(s_{1},s_{2},\ldots,s_{r})\in\mathbb{C}^{r}, where i=1rnisi=0.\sum\limits_{i=1}^{r}n_{i}s_{i}=0. Consider E𝒫,Φ(,s)E_{\mathcal{P},\Phi}(*,s) with associated Langlands parameters α𝒫,Φ(s)\alpha_{{}_{\mathcal{P},\Phi}}(s) as defined in (3.7). Assume that each Hecke-Maass form ϕk\phi_{k} (with 1kr1\leq k\leq r) occurring in Φ\Phi has Langlands parameters α(k):=(αk,1,,αk,nk)\alpha^{(k)}:=(\alpha_{k,1},\ldots,\alpha_{k,n_{k}}) with the convention that if nk=1n_{k}=1 then αk,1=0.\alpha_{k,1}=0. We also assume that each ϕk\phi_{k} is normalized to have Petersson norm ϕk,ϕk=1.\langle\phi_{k},\phi_{k}\rangle=1.

Let L(1+sjs,ϕj×ϕ)L^{*}(1+s_{j}-s_{\ell},\phi_{j}\times\phi_{\ell}) denote the completed Rankin-Selberg L-function if nj1nn_{j}\neq 1\neq n_{\ell}; otherwise define

L(1+sjs,ϕj×ϕ)={L(1+sjs,ϕj)ifn=1andnj1,L(1+sjs,ϕ)ifnj=1andn1,ζ(1+sjs)ifnj=n=1,L^{*}(1+s_{j}-s_{\ell},\;\phi_{j}\times\phi_{\ell})=\begin{cases}L^{*}(1+s_{j}-s_{\ell},\,\phi_{j})&\text{if}\;n_{\ell}=1\;\text{and}\;n_{j}\neq 1,\\ L^{*}(1+s_{j}-s_{\ell},\,\phi_{\ell})&\text{if}\;n_{j}=1\;\text{and}\;n_{\ell}\neq 1,\\ \zeta^{*}(1+s_{j}-s_{\ell})&\text{if}\;n_{j}=n_{\ell}=1,\end{cases}

where ζ(w)=πw2Γ(w2)ζ(w)\zeta^{*}(w)=\pi^{-\frac{w}{2}}\Gamma\left(\frac{w}{2}\right)\zeta(w) is the completed Riemann ζ\zeta-function. Also define

L(1,Adϕk)=L(1,Adϕk)1ijnkΓ(1+αk,iαk,j2),L^{*}(1,\,\text{\rm Ad}\;\phi_{k})=L(1,\,\text{\rm Ad}\;\phi_{k})\prod_{1\leq i\neq j\leq n_{k}}\Gamma\left(\frac{1+\alpha_{k,i}-\alpha_{k,j}}{2}\right),

with the convention that L(1,Ad 1)=1L^{*}(1,\,\text{\rm Ad}\;1)=1.

Let M=(m1,m2,,mn1)>0n1M=(m_{1},m_{2},\ldots,m_{n-1})\in\mathbb{Z}_{>0}^{n-1}. Per our convention (Definition 2.4), we may think of MM as a vector or a diagonal matrix. Then the MthM^{th} term in the Fourier-Whittaker expansion of E𝒫,ΦE_{\mathcal{P},\Phi} is

Un()\Un()E𝒫,Φ(ug,s)ψM(u)¯𝑑u=AE𝒫,Φ(M,s)k=1n1mkk(nk)/2Wα𝒫,Φ(s)(Mg),\displaystyle\int\limits_{U_{n}(\mathbb{Z})\backslash U_{n}(\mathbb{R})}E_{\mathcal{P},\Phi}(ug,s)\,\overline{\psi_{M}(u)}\;du\;=\;\frac{A_{E_{\mathcal{P},\Phi}}(M,s)}{\prod\limits_{k=1}^{n-1}m_{k}^{k(n-k)/2}}\;W_{\alpha_{{}_{\mathcal{P},\Phi}}(s)}\big{(}Mg\big{)},

where AE𝒫,Φ(M,s)=AE𝒫,Φ((1,,1),s)λE𝒫,Φ(M,s),A_{E_{\mathcal{P},\Phi}}(M,s)=A_{E_{\mathcal{P},\Phi}}\big{(}(1,\ldots,1),s\big{)}\cdot\lambda_{E_{\mathcal{P},\Phi}}(M,s),

(4.6) λEP,Φ((m,1,,1),s)\displaystyle\lambda_{E_{P,\Phi}}\big{(}(m,1,\ldots,1),s\big{)} =c1,,cn>0c1c2cn=mλϕ1(c1)λϕr(cr)c1s1crsr\displaystyle=\underset{c_{1}c_{2}\cdots c_{n}=m}{\sum_{c_{1},\ldots,c_{n}\in\mathbb{Z}_{>0}}}\hskip-10.0pt\lambda_{\phi_{1}}(c_{1})\cdots\lambda_{\phi_{r}}(c_{r})\cdot c_{1}^{s_{1}}\cdots c_{r}^{s_{r}}

is the (m,1,,1)th(m,1,\ldots,1)^{th} (or more informally the mthm^{th}) Hecke eigenvalue of E𝒫,ΦE_{\mathcal{P},\Phi}, and

AE𝒫,Φ((1,,1),s)=d0k=1rnk1L(1,Adϕk)121j<rL(1+sjs,ϕj×ϕ)1\displaystyle A_{E_{\mathcal{P},\Phi}}\big{(}(1,\ldots,1),s\big{)}=d_{0}\underset{n_{k}\neq 1}{\prod_{k=1}^{r}}L^{*}\big{(}1,\text{\rm Ad}\;\phi_{k}\big{)}^{-\frac{1}{2}}\prod_{1\leq j<\ell\leq r}L^{*}\big{(}1+s_{j}-s_{\ell},\;\phi_{j}\times\phi_{\ell}\big{)}^{-1}

for some constant d00d_{0}\neq 0 depending only on nn.

Proof.

See [GSW24].∎

4.2. Geometric side of the Kuznetsov trace formula

In this section, we obtain explicit descriptions of the terms \mathcal{M} and 𝒦\mathcal{K} appearing on the geometric side of the Kuznetsov trace formula. In order to do this, we introduce Kloosterman sums for SL(n,)\operatorname{SL}(n,{\mathbb{Z}}), which appear in the Fourier expansion of the Poincaré series. In the inner product PL,PM\big{\langle}P^{L},P^{M}\big{\rangle}, we replace PLP^{L} with its Fourier expansion and unravel PMP^{M} following the Rankin-Selberg method.

Theorem 4.1 (Geometric side of the trace formula).

Fix L=(1,,n1)L=(\ell_{1},\ldots,\ell_{n-1}) andM=(m1,,mn1)n1M=(m_{1},\ldots,m_{n-1})\in{\mathbb{Z}}^{n-1} (𝔠n\mathfrak{c}_{n} is a nonzero constant; see Proposition 4.4). It follows that for α0:=(n12+j1)j=1,,n\alpha_{0}:=\big{(}-\tfrac{n-1}{2}+j-1\big{)}_{j=1,\ldots,n},

PL(,α0),PM(,α0)=+𝒦.\boxed{\ \big{\langle}P^{L}(*,\alpha_{0}),\;P^{M}(*,\alpha_{0})\big{\rangle}\;=\;\mathcal{M}\;+\;\mathcal{K}.\ }

For w1w_{1} the trivial element in the Weyl group WnW_{n}, we define

:=w1,and𝒦:=wWnww1w,\mathcal{M}:=\mathcal{I}_{w_{1}},\qquad\mbox{and}\qquad\mathcal{K}:=\sum_{\begin{subarray}{c}w\in W_{n}\\ w\neq w_{1}\end{subarray}}\mathcal{I}_{w},

where

(4.2) w:=vVnc1=1cn1=1Sw(ψL,ψMv,c)𝔠nk=1n1(mkk)k(nk)2y1=0yn1=0Uw()\Uw()U¯w()ψL(wuy)ψMv(u)¯pT,R(n)(Lcwuy)pT,R(n)(My)¯dudy1dyn1k=1n1ykk(nk)+1.\mathcal{I}_{w}:=\;\sum_{v\in V_{n}}\sum_{c_{1}=1}^{\infty}\cdots\sum_{c_{n-1}=1}^{\infty}\frac{S_{w}(\psi_{L},\psi_{M}^{v},c)}{\mathfrak{c}_{n}\cdot\prod\limits_{k=1}^{n-1}(m_{k}\ell_{k})^{\frac{k(n-k)}{2}}}\int\limits_{y_{1}=0}^{\infty}\cdots\int\limits_{y_{n-1}=0}^{\infty}\;\;\int\limits_{{U}_{w}(\mathbb{Z})\backslash{U}_{w}(\mathbb{R})}\;\int\limits_{\overline{{U}}_{w}(\mathbb{R})}\\ \cdot\psi_{L}(wuy)\,\overline{\psi_{M}^{v}(u)}\;p_{T,R}^{(n)}(Lcwuy)\,\overline{p_{T,R}^{(n)}(My)}\;d^{*}u\,\frac{dy_{1}\cdots dy_{n-1}}{\prod\limits_{k=1}^{n-1}y_{k}^{k(n-k)+1}}.
Proof.

We compute the inner product

limαα0PL(,α),PM(,α)=limαα0SL(n,)\𝔥nPL(g,α)PM(g,α)¯𝑑g\displaystyle\lim_{\alpha\to\alpha_{0}}\big{\langle}P^{L}\left(*,\,\alpha\right),\,P^{M}\left(*,\,\alpha\right)\big{\rangle}=\lim_{\alpha\to\alpha_{0}}\int\limits_{\operatorname{SL}(n,\mathbb{Z})\backslash\mathfrak{h}^{n}}P^{L}\left(g,\alpha\right)\cdot\overline{P^{M}\left(g,\alpha\right)}\;dg
=\displaystyle= 1𝔠nk=1n1mkk(nk)2limαα0Un()\𝔥nPL(g,α)ψM(g)pT,R(n)(Mg)I(g,α)¯𝑑g\displaystyle\frac{1}{\sqrt{\mathfrak{c}_{n}}\prod\limits_{k=1}^{n-1}m_{k}^{\frac{k(n-k)}{2}}}\lim_{\alpha\to\alpha_{0}}\int\limits_{U_{n}(\mathbb{Z})\backslash\mathfrak{h}^{n}}P^{L}\left(g,\alpha\right)\cdot\overline{\psi_{M}(g)\,p_{T,R}^{(n)}(Mg)\,I(g,\alpha)}\;dg
=\displaystyle= 1𝔠n(k=1n1mkk(nk)2)limαα0yn1y>0(Un()\Un()PL(uy,α)ψM(u)¯𝑑u)pT,R(n)(My)I(y,α)¯𝑑y.\displaystyle\frac{1}{\sqrt{\mathfrak{c}_{n}}}\left(\prod_{k=1}^{n-1}m_{k}^{-\frac{k(n-k)}{2}}\right)\lim_{\alpha\to\alpha_{0}}\hskip-1.0pt\int\limits_{\begin{subarray}{c}y\in\mathbb{R}^{n-1}\\ y>0\end{subarray}}\hskip-1.0pt\left(\;\int\limits_{U_{n}(\mathbb{Z})\backslash U_{n}(\mathbb{R})}\hskip-3.0ptP^{L}\left(uy,\alpha\right)\cdot\overline{\psi_{M}(u)}\;du\right)\overline{p_{T,R}^{(n)}(My)\,I(y,\alpha)}\;\,dy.

Note that, as αα0\alpha\to\alpha_{0}, the function I(g,α)1I(g,\alpha)\to 1 (for any g𝔥ng\in\mathfrak{h}^{n}) and k=1n1ckαkαk+1+11\prod\limits_{k=1}^{n-1}c_{k}^{\alpha_{k}-\alpha_{k+1}+1}\to 1. It follows from this and Proposition 2.3 above that

𝔠n\displaystyle\mathfrak{c}_{n}\cdot k=1n1(mkk)k(nk)/2limαα0PL(,α),PM(,α)\displaystyle\prod\limits_{k=1}^{n-1}(m_{k}\ell_{k})^{k(n-k)/2}\cdot\lim_{\alpha\to\alpha_{0}}\big{\langle}P^{L}\left(*,\,\alpha\right),\,P^{M}\left(*,\,\alpha\right)\big{\rangle}
=limαα0wWnvVnc1=1cn1=1Sw(ψL,ψMv,c)𝔠nk=1n1(mii)i(ni)2k=1n1ckαkαk+1+1y1=0yn1=0Uw()\Uw()U¯w()\displaystyle\quad=\lim_{\alpha\to\alpha_{0}}\sum_{w\in W_{n}}\sum_{v\in V_{n}}\sum_{c_{1}=1}^{\infty}\cdots\sum_{c_{n-1}=1}^{\infty}\frac{S_{w}(\psi_{L},\psi_{M}^{v},c)}{\mathfrak{c}_{n}\cdot\prod\limits_{k=1}^{n-1}(m_{i}\ell_{i})^{\frac{i(n-i)}{2}}\prod\limits_{k=1}^{n-1}c_{k}^{\alpha_{k}-\alpha_{k+1}+1}}\int\limits_{y_{1}=0}^{\infty}\cdots\int\limits_{y_{n-1}=0}^{\infty}\;\;\int\limits_{{U}_{w}(\mathbb{Z})\backslash{{U}}_{w}(\mathbb{R})}\;\int\limits_{\overline{{U}}_{w}(\mathbb{R})}
ψL(wuy)ψMv(u)¯pT,R(n)(Lcwuy)pT,R(n)(My)¯I(wuy,α)I(y,α)¯dudy1dyn1k=1n1ykk(nk)+1\displaystyle\quad\qquad\cdot\psi_{L}(wuy)\,\overline{\psi_{M}^{v}(u)}\;p_{T,R}^{(n)}(Lcwuy)\,\overline{p_{T,R}^{(n)}(My)}\;I(wuy,\alpha)\;\overline{I(y,\alpha)}\;d^{*}u\,\frac{dy_{1}\cdots dy_{n-1}}{\prod\limits_{k=1}^{n-1}y_{k}^{k(n-k)+1}}
=wWnvVnc1=1cn1=1Sw(ψL,ψMv,c)𝔠nk=1n1(mkk)k(nk)2y1=0yn1=0Uw()\Uw()U¯w()\displaystyle\quad=\;\sum_{w\in W_{n}}\sum_{v\in V_{n}}\sum_{c_{1}=1}^{\infty}\cdots\sum_{c_{n-1}=1}^{\infty}\frac{S_{w}(\psi_{L},\psi_{M}^{v},c)}{\mathfrak{c}_{n}\cdot\prod\limits_{k=1}^{n-1}(m_{k}\ell_{k})^{\frac{k(n-k)}{2}}}\int\limits_{y_{1}=0}^{\infty}\cdots\int\limits_{y_{n-1}=0}^{\infty}\;\;\int\limits_{{U}_{w}(\mathbb{Z})\backslash{{U}}_{w}(\mathbb{R})}\;\int\limits_{\overline{{U}}_{w}(\mathbb{R})}
ψL(wuy)ψMv(u)¯pT,R(n)(Lcwuy)pT,R(n)(My)¯dudy1dyn1k=1n1ykk(nk)+1\displaystyle\quad\qquad\cdot\psi_{L}(wuy)\,\overline{\psi_{M}^{v}(u)}\;p_{T,R}^{(n)}(Lcwuy)\,\overline{p_{T,R}^{(n)}(My)}\;du\,\frac{dy_{1}\cdots dy_{n-1}}{\prod\limits_{k=1}^{n-1}y_{k}^{k(n-k)+1}}
=wWnw,\displaystyle\quad=\sum_{w\in W_{n}}\mathcal{I}_{w},

as claimed. ∎

5. Asymptotic formula for the main term

Proposition 5.1 (Main term in the trace formula).

Let L=(1,,n1),M=(m1,,mn1)n1L=(\ell_{1},\ldots,\ell_{n-1}),\,M=(m_{1},\ldots,m_{n-1})\in\mathbb{Z}^{n-1} satisfy i=1n1i0\prod\limits_{i=1}^{n-1}\ell_{i}\neq 0 and i=1n1mi0\prod\limits_{i=1}^{n-1}m_{i}\neq 0. There exist fixed constants 𝔠1,,𝔠n1>0\mathfrak{c}_{1},\ldots,\mathfrak{c}_{n-1}>0 (depending only on RR and nn) such that the main term \mathcal{M} in the Kuznetsov trace formula (4.1) is given by

=δL,M((i=1n1𝔠iTR(2D(n)+n(n1))+ni)+𝒪(TR(2D(n)+n(n1)))),\mathcal{M}=\delta_{L,M}\cdot\left(\left(\sum_{i=1}^{n-1}\mathfrak{c}_{i}\cdot T^{R(2\cdot D(n)+n(n-1))+n-i}\right)+\mathcal{O}\left(T^{R(2\cdot D(n)+n(n-1))}\right)\right),

where D(n)=12(2nn)n(n1)22n1D(n)=\frac{1}{2}\binom{2n}{n}-\frac{n(n-1)}{2}-2^{n-1} and δL,M\delta_{L,M} is the Kronecker symbol (i.e., δL,M=0\delta_{L,M}=0 if LML\neq M and δL,L=1\delta_{L,L}=1).

Proof.

It follows from the definition =w1\mathcal{M}=\mathcal{I}_{w_{1}}, making the change of variables yM1yy\mapsto M^{-1}y, and noting that Uw1()=Un()U_{w_{1}}({\mathbb{Z}})=U_{n}({\mathbb{Z}}) and Uw1()=Un()U_{w_{1}}(\mathbb{R})=U_{n}(\mathbb{R}), that

\displaystyle\mathcal{M} =1𝔠ny1=0yn1=0(Un()\Un()ψL(u)ψM(u)¯du)pT,R(LM1y)pT,R(y)¯dy1dyn1i=1n1yii(ni)+1\displaystyle=\frac{1}{\mathfrak{c}_{n}}\cdot\int\limits_{y_{1}=0}^{\infty}\cdots\int\limits_{y_{n-1}=0}^{\infty}\;\left(\;\int\limits_{{U}_{n}(\mathbb{Z})\backslash{{U}}_{n}(\mathbb{R})}\hskip-5.0pt\psi_{L}(u)\overline{\psi_{M}(u)}\;d^{*}u\right)p_{T,R}(LM^{-1}y)\,\overline{p_{T,R}(y)}\;\frac{dy_{1}\cdots dy_{n-1}}{\prod\limits_{i=1}^{n-1}y_{i}^{i(n-i)+1}}
=δL,M𝔡ny1=0yn1=0|pT,R(n)(y)|2dy1dyn1i=1n1yii(ni)+1=δL,M𝔡npT,R,pT,R\displaystyle=\delta_{L,M}\cdot\mathfrak{d}_{n}\int\limits_{y_{1}=0}^{\infty}\cdots\int\limits_{y_{n-1}=0}^{\infty}|p_{T,R}^{(n)}(y)|^{2}\,\frac{dy_{1}\cdots dy_{n-1}}{\prod\limits_{i=1}^{n-1}y_{i}^{i(n-i)+1}}=\delta_{L,M}\cdot\mathfrak{d}_{n}\big{\langle}p_{T,R},p_{T,R}\big{\rangle}
=δL,M𝔡nα^n=0Re(α)=0|pT,Rn,#(α)|21jknΓ(αjαk2)𝑑α\displaystyle=\;\delta_{L,M}\cdot\mathfrak{d}_{n}\int\limits_{\begin{subarray}{c}\widehat{\alpha}_{n}=0\\ \operatorname{Re}(\alpha)=0\end{subarray}}\frac{\big{|}p_{T,R}^{n,\#}(\alpha)\big{|}^{2}}{\prod\limits_{1\leq j\neq k\leq n}\Gamma\left(\frac{\alpha_{j}-\alpha_{k}}{2}\right)}\;d\alpha
=δL,M𝔡npT,Rn,#,pT,Rn,#,\displaystyle=\delta_{L,M}\,\mathfrak{d}_{n}\cdot\big{\langle}p_{T,R}^{n,\#},\;p_{T,R}^{n,\#}\big{\rangle},

where the representation in terms of the norm of pT,Rn,#p_{T,R}^{n,\#} follows from the Plancherel formula in Corollary 1.9 of [GK12] and 𝔡n\mathfrak{d}_{n} is a nonzero constant depending only on nn. Hence the main term for GL(n)\operatorname{GL}(n) is thus

=δL,M𝔡nα^n=0Re(αj)=0|eα12++αn22T2R(n)(α2)1jknΓ(2R+1+αjαk4)|21jknΓ(αjαk2)𝑑α.\mathcal{M}=\delta_{L,M}\mathfrak{d}_{n}\cdot\int\limits_{\begin{subarray}{c}\widehat{\alpha}_{n}=0\\ \operatorname{Re}(\alpha_{j})=0\end{subarray}}\frac{\Big{|}e^{\frac{\alpha_{1}^{2}+\cdots+\alpha_{n}^{2}}{2T^{2}}}\mathcal{F}_{R}^{(n)}\big{(}\frac{\alpha}{2}\big{)}\prod\limits_{1\leq\,j\neq k\,\leq n}\Gamma\left(\textstyle{\frac{2R+1+\alpha_{j}-\alpha_{k}}{4}}\right)\Big{|}^{2}}{\prod\limits_{1\leq j\neq k\leq n}\Gamma\left(\frac{\alpha_{j}-\alpha_{k}}{2}\right)}\;d\alpha.

Let αj=iτj\alpha_{j}=i\tau_{j} with τj\tau_{j}\in\mathbb{R}. It then follows from Stirling’s asymptotic formula that

\displaystyle\mathcal{M} δL,M𝔡nτ^n=0eτ12τn2T2(R(n)(iτ2))21j<kn(1+|τjτk|)2Rdτ.\displaystyle\sim\delta_{L,M}\mathfrak{d}_{n}\cdot\int\limits_{\widehat{\tau}_{n}=0}e^{\frac{-\tau_{1}^{2}-\cdots-\tau_{n}^{2}}{T^{2}}}\Big{(}\mathcal{F}_{R}^{(n)}\big{(}\tfrac{i\tau}{2}\big{)}\Big{)}^{2}\prod\limits_{1\leq\,j<k\,\leq n}\big{(}1+\lvert\tau_{j}-\tau_{k}\rvert\big{)}^{2R}\ d\tau.

If we now make the change of variables τjτjT\tau_{j}\to\tau_{j}T for each j=1,,nj=1,\ldots,n, and we use the fact that the degree of 1(n)\mathcal{F}_{1}^{(n)} is D(n)D(n) (see Definition 1.1) it follows that, if L=ML=M, as TT\to\infty we have 𝔠TR(2D(n)+n(n1))+n1\mathcal{M}\sim\mathfrak{c}T^{R\cdot\big{(}2D(n)+n(n-1)\big{)}+n-1}, where

𝔠\displaystyle\mathfrak{c} =𝔡nτ^n=0eτ12τn2(R(n)(iτ2))21j<kn(1+|τjτk|)2Rdτ,\displaystyle=\mathfrak{d}_{n}\cdot\int\limits_{\widehat{\tau}_{n}=0}e^{-\tau_{1}^{2}-\cdots-\tau_{n}^{2}}\Big{(}\mathcal{F}_{R}^{(n)}\big{(}\tfrac{i\tau}{2}\big{)}\Big{)}^{2}\prod\limits_{1\leq\,j<k\,\leq n}\big{(}1+\lvert\tau_{j}-\tau_{k}\rvert\big{)}^{2R}\ d\tau,

and otherwise, the main term is zero. This gives the i=1i=1 term in the statement of the proposition. The method of proof can be extended by using additional terms in Stirling’s asymptotic expansion for the Gamma function to obtain the additional terms. ∎

Remark 5.2.

Note that this doesn’t agree with [GSW21] in the case of n=4n=4 because we have used a different normalization. Namely, the linear factors of R(n)\mathcal{F}_{R}^{(n)} agree with those defined previously, but we take a different power of each. Also, the gamma factors which appear in pT,Rn,#p_{T,R}^{n,\#} have a different RR: namely, what was 2+R2+R in each gamma factor previously has been replaced by 2R+12R+1 here.

6. Bounding the geometric side

The goal of this section is to use the bound given in Theorem 10.1 to prove the following, i.e., to bound 𝒦\mathcal{K} the geometric side of the Kuznetsov trace formula.

Proposition 6.1.

Let w\mathcal{I}_{w} be as above. Let M=(m1,,mn1),L=(1,,n1)(>0)n1M=(m_{1},\ldots,m_{n-1}),\;L=(\ell_{1},\ldots,\ell_{n-1})\in(\mathbb{Z}_{>0})^{n-1}. Let ρ12+\rho\in\frac{1}{2}+{\mathbb{Z}}. Let D(n)=12(2nn)n(n1)22n1D(n)=\frac{1}{2}\binom{2n}{n}-\frac{n(n-1)}{2}-2^{n-1} as in (1.2). Then for RR sufficiently large and any ε>0\varepsilon>0, we have

|w|ε,RTε+R(2D(n)+n(n1))+(n1)(n+4)2n12ρnΦ(w)i=1n1(imi)2ρ+n2+14,\big{|}\mathcal{I}_{w}\big{|}\ll_{\varepsilon,R}\;T^{\varepsilon+R\big{(}2D(n)+n(n-1)\big{)}+\frac{(n-1)(n+4)}{2}-\left\lfloor\frac{n-1}{2}\right\rfloor-\rho n-\Phi(w)}\cdot\prod_{i=1}^{n-1}(\ell_{i}m_{i})^{2\rho+\frac{n^{2}+1}{4}},

where if w=w(n1,,nr)w=w_{(n_{1},\ldots,n_{r})} with r2r\geq 2,

Φ(w):=Φ(n1,,nr):=12k=1r1(nk+nk+1)(nn^k)n^k.\Phi(w):=\Phi(n_{1},\ldots,n_{r}):=\frac{1}{2}\sum_{k=1}^{r-1}(n_{k}+n_{k+1})(n-\widehat{n}_{k})\widehat{n}_{k}.
Remark 6.2.

Assuming the lower bound conjecture for Rankin-Selberg L-functions, the resulting bound for the Eisenstein series contribution to the Kuznetsov trace formula (see Theorem 7.1) is of the magnitude TT to the power R(2D(n)+n(n1))+εR\big{(}2D(n)+n(n-1)\big{)}+\varepsilon. Therefore, given Proposition 6.1 and Lemma A.13 (which says that Φ(w)Φ(1,n1)=n(n1)2\Phi(w)\geq\Phi(1,n-1)=\frac{n(n-1)}{2}), in order for the bound from the geometric side of the trace formula to be less than the Eisenstein series contribution, it suffices that

(n1)(n+4)2n12ρnn(n1)20,\frac{(n-1)(n+4)}{2}-\left\lfloor\frac{n-1}{2}\right\rfloor-\rho n-\frac{n(n-1)}{2}\leq 0,

which simplifies to give

ρ{3232n if n is odd,321n if n is even.\rho\geq\begin{cases}\frac{3}{2}-\frac{3}{2n}&\mbox{ if $n$ is odd},\\ \frac{3}{2}-\frac{1}{n}&\mbox{ if $n$ is even}.\end{cases}

Since we require that ρ12+\rho\in\frac{1}{2}+{\mathbb{Z}}, we find that it suffices to take ρ=32\rho=\frac{3}{2} universally, meaning that the exponent of each term imi\ell_{i}m_{i} can be taken to be n2+134\frac{n^{2}+13}{4}. In particular, for the case of n=4n=4, we see that this exponent is 294\frac{29}{4} which is an improvement on the bound of 152\frac{15}{2} obtained in [GSW21].

As remarked above, the main result that we will need is Theorem 10.1 or, more specifically, Remark 10.5, which states that for any 0<ε<120<\varepsilon<\frac{1}{2}, and for a=(a1,a2,,an1)a=(a_{1},a_{2},\ldots,a_{n-1}) satisfying aj+ε<aj<ajε\lfloor a_{j}\rfloor+\varepsilon<a_{j}<\lceil a_{j}\rceil-\varepsilon for each j=1,,n1j=1,\ldots,n-1, that

(6.3) |pT,R(n)(y)|δ12(y)y2aTε+(n+4)(n1)4+R(D(n)+n(n1)2)j=1n1B(aj).\boxed{\big{|}p_{T,R}^{(n)}(y)\big{|}\ll\delta^{-\frac{1}{2}}(y)\cdot\big{\lVert}y\big{\rVert}^{2a}\cdot T^{\varepsilon+\frac{(n+4)(n-1)}{4}+R\cdot\big{(}D(n)+\frac{n(n-1)}{2}\big{)}-\sum\limits_{j=1}^{n-1}B(a_{j})}}.

(The terms δ12(y)\delta^{-\frac{1}{2}}(y) and y2a\lVert y\rVert^{2a} are defined in Section 6.1 below. The function BB is defined in Theorem 9.2.)

This bound for pT,R(n)(y)p_{T,R}^{(n)}(y) is obtained via an integral representation denoted pT,R(n)(y;b)p_{T,R}^{(n)}(y;b) (see (8.4)) over variables s=(s1,,sn)s=(s_{1},\ldots,s_{n}) valid for any b=(b1,,bn)b=(b_{1},\ldots,b_{n}) with bj>0b_{j}>0 for each j=1,,n1j=1,\ldots,n-1. The integral is taken over the lines Re(sj)=bj\operatorname{Re}(s_{j})=b_{j}. Essentially, the bound is then obtained by moving the lines of integration to Re(sj)=aj\operatorname{Re}(s_{j})=-a_{j} for some a=(a1,,an1)(>0)n1a=(a_{1},\ldots,a_{n_{1}})\in\big{(}\mathbb{R}_{>0}\big{)}^{n-1}.

The strategy for proving Proposition 6.1 will be to, first, introduce notation to rewrite w\mathcal{I}_{w} in a simplified form. We do this in Section 6.1. Then, in Section 6.2 we give bounds for w\mathcal{I}_{w} obtained by applying (6.3) to |pT,R(Lcwuy)|\lvert p_{T,R}(Lcwuy)\rvert (with a parameter a=(a1,,an1)a=(a_{1},\ldots,a_{n-1})) and to |pT,R(My)|\lvert p_{T,R}(My)\rvert (with a parameter b=(b1,,bn1)b=(b_{1},\ldots,b_{n-1})) for general a,b(>0)n1a,b\in(\mathbb{R}_{>0})^{n-1}. In particular, we establish (6.2), bounding |w|\lvert\mathcal{I}_{w}\rvert in terms of the product of three independent quantities K(c,w;a)K(c,w;a), X(u,w;a)X(u,w;a) and Y(y,w;a,b)Y(y,w;a,b). In Section 6.3, we will show that K(c,w;a)K(c,w;a) will converge provided that aa satisfies certain conditions (independent of ww), and that for this choice of aa, X(u,w;a)X(u,w;a) also converges. We then determine bb (dependent on by ww and aa) for which Y(y,w;a,b)Y(y,w;a,b) is also convergent. Finally, in Section 6.4, we complete the proof of Proposition 6.1 by simplifying the expression for the given choices of aa and bb.

6.1. Rewriting w\mathcal{I}_{w}

Let Tn()T_{n}(\mathbb{R}) and Un()U_{n}(\mathbb{R}) be the subgroups of GLn()\operatorname{GL}_{n}(\mathbb{R}) consisting of diagonal matrices (with positive terms) and upper triangular unipotent matrices, respectively. Recall that if t=diag(t1,,tn)Tn()t=\operatorname{diag}(t_{1},\ldots,t_{n})\in T_{n}(\mathbb{R}) and uUn()u\in U_{n}(\mathbb{R}), the modular character δ:Tn()\delta:T_{n}(\mathbb{R})\to\mathbb{R} is defined to satisfy d(t1ut)=δ(t)dud(t^{-1}ut)=\delta(t)\,du. Explicitly, it is given by

δ(t)=i=1nti2in1.{\delta(t)=\prod_{i=1}^{n}t_{i}^{2i-n-1}}.

More generally, if a=(a1,,an1)n1a=(a_{1},\ldots,a_{n-1})\in\mathbb{R}^{n-1}, for

y=(y1,,yn1):=diag(y1yn2yn1,,y1y2,y1,1)y=(y_{1},\ldots,y_{n-1}):=\operatorname{diag}(y_{1}\cdots y_{n-2}y_{n-1},\ldots,y_{1}y_{2},y_{1},1)

with y1,,yn1>0y_{1},\ldots,y_{n-1}>0 we define

ya:=k=1n1ykak.\lVert y\rVert^{a}:=\prod_{k=1}^{n-1}y_{k}^{a_{k}}.

One checks that in the special case of aj=j(nj)2a_{j}=\frac{j(n-j)}{2} for j=1,,n1j=1,\ldots,n-1,

(6.1) δ12(y)=ya.\delta^{-\frac{1}{2}}(y)=\lVert y\rVert^{a}.

Similarly, if Unt(){}^{t}U_{n}(\mathbb{R}) is the subgroup of GLn()\operatorname{GL}_{n}(\mathbb{R}) consisting of lower triangular unipotent matrices and

U¯w:=(w1Unt()w)Un(),\overline{U}_{w}:=\big{(}w^{-1}\;{}^{t}U_{n}(\mathbb{R})\;w\big{)}\cap U_{n}(\mathbb{R}),

then we can consider the character δw\delta_{w} on Tn()T_{n}(\mathbb{R}) which satisfies d(tut1)=δw(t)dud(tut^{-1})=\delta_{w}(t)du upon restricting the measure on Un()U_{n}(\mathbb{R}) to U¯w\overline{U}_{w}. It can be checked that

(6.2) δw(y)=δ12(y)δ12(wyw1).\delta_{w}(y)=\delta^{\frac{1}{2}}(y)\cdot\delta^{-\frac{1}{2}}(wyw^{-1}).

Recall from Theorem 4.1 that for L=(1,,n1),M=(m1,,mn1)(>0)n1L=(\ell_{1},\ldots,\ell_{n-1}),M=(m_{1},\ldots,m_{n-1})\in({\mathbb{Z}}_{>0})^{n-1} and

c=(1/cn1cn1/cn2c2/c1c1),c=\left(\begin{smallmatrix}1/c_{n-1}&&&&\\ &c_{n-1}/c_{n-2}&&&\\ &&\ddots&&\\ &&&c_{2}/c_{1}&\\ &&&&c_{1}\end{smallmatrix}\right),

where ci>0c_{i}\in\mathbb{{\mathbb{Z}}}_{>0} for i=1,,n1i=1,\ldots,n-1, the Kloosterman contribution to the Kuznetsov trace formula is given by

𝒦=wWnww1w,\mathcal{K}\;=\;\sum\limits_{\begin{subarray}{c}w\in W_{n}\\ w\neq w_{1}\end{subarray}}\mathcal{I}_{w},

where, using the notation defined above and letting dy×dy^{\times} denote the measure k=1n1dykyk\prod\limits_{k=1}^{n-1}\frac{dy_{k}}{y_{k}},

(6.3) w:=𝔠n1vVnc1=1cn1=1Sw(ψL,ψMv,c)y=(y1,,yn1)y1,,yn1>0Uw()\Uw()U¯w()δ12(LM)δ(y)ψL(wuy)ψMv(u)¯pT,R(n)(Lcwuy)pT,R(n)(My)¯dudy×.\mathcal{I}_{w}:=\;\mathfrak{c}_{n}^{-1}\sum_{v\in V_{n}}\sum_{c_{1}=1}^{\infty}\cdots\sum_{c_{n-1}=1}^{\infty}S_{w}(\psi_{L},\psi_{M}^{v},c)\;\int\limits_{\begin{subarray}{c}y=(y_{1},\ldots,y_{n-1})\\ y_{1},\ldots,y_{n-1}>0\end{subarray}}\;\int\limits_{{U}_{w}(\mathbb{Z})\backslash{U}_{w}(\mathbb{R})}\;\int\limits_{\overline{{U}}_{w}(\mathbb{R})}\\ \cdot\delta^{\frac{1}{2}}(LM)\cdot\delta(y)\cdot\psi_{L}(wuy)\,\overline{\psi_{M}^{v}(u)}\;p_{T,R}^{(n)}(Lcwuy)\,\overline{p_{T,R}^{(n)}(My)}\;d^{*}u\;dy^{\times}.

We recall that by Friedberg [Fri87], w\mathcal{I}_{w} is identically zero unless ww is relevant (see Definition 2.6).

6.2. Bounds for w\mathcal{I}_{w} in terms of aa and bb

Since pT,R(n)(g)p_{T,R}^{(n)}(g) is determined by the Iwasawa decomposition of gg, we first make the change of variables uy1uyu\mapsto y^{-1}uy. Then (6.3) implies that

(6.1) |w|vVnc1=1cn1=1|Sw(ψL,ψMv,c)|y=(y1,,yn1)y1,,yn1>0Uw()\Uw()U¯w()δ12(M)δ12(L)δw(y)δ(y)|pT,R(n)(Lcwyu)||pT,R(n)(My)|dudy×.\lvert\mathcal{I}_{w}\rvert\ll\;\sum_{v\in V_{n}}\sum_{c_{1}=1}^{\infty}\cdots\sum_{c_{n-1}=1}^{\infty}\lvert S_{w}(\psi_{L},\psi_{M}^{v},c)\rvert\int\limits_{\begin{subarray}{c}y=(y_{1},\ldots,y_{n-1})\\ y_{1},\ldots,y_{n-1}>0\end{subarray}}\;\int\limits_{{U}_{w}(\mathbb{Z})\backslash{U}_{w}(\mathbb{R})}\;\int\limits_{\overline{{U}}_{w}(\mathbb{R})}\\ \cdot\,\delta^{\frac{1}{2}}(M)\cdot\delta^{\frac{1}{2}}(L)\cdot\delta_{w}(y)\cdot\delta(y)\cdot\lvert p_{T,R}^{(n)}(Lcwyu)\rvert\,\lvert p_{T,R}^{(n)}(My)\rvert\;d^{*}u\;dy^{\times}.

For the purposes of our analysis, we break up the integral in the yy-variables. To this end, let

I0:=(0,1],I1=(1,).I_{0}:=(0,1],\qquad I_{1}=(1,\infty).

For τ=(τ1,,τn1){0,1}n1\tau=(\tau_{1},\ldots,\tau_{n-1})\in\{0,1\}^{n-1}, define

Iτ:=Iτ1××Iτn1.I_{\tau}:=I_{\tau_{1}}\times\cdots\times I_{\tau_{n-1}}.

Hence,

y=(y1,,yn1)y1,,yn1>0=τ{0,1}n1Iτ,\int\limits_{\begin{subarray}{c}y=(y_{1},\ldots,y_{n-1})\\ y_{1},\ldots,y_{n-1}>0\end{subarray}}=\sum_{\tau\in\{0,1\}^{n-1}}\;\int\limits_{I_{\tau}},

and (6.1) becomes

|w|τ|w(τ)|\lvert\mathcal{I}_{w}\rvert\ll\sum\limits_{\tau}\lvert\mathcal{I}_{w}(\tau)\rvert

where

(6.2) |w(τ)|:=vVnc1=1cn1=1|Sw(ψL,ψMv,c)|y1Iτ1y2Iτ2yn1Iτn1Uw()\Uw()U¯w()δ12(M)δ12(L)δw(y)δ(y)|pT,R(n)(Lcwyu)||pT,R(n)(My)|dudy×.\lvert\mathcal{I}_{w}(\tau)\rvert\;:=\;\sum_{v\in V_{n}}\sum_{c_{1}=1}^{\infty}\cdots\sum_{c_{n-1}=1}^{\infty}\lvert S_{w}(\psi_{L},\psi_{M}^{v},c)\rvert\int\limits_{y_{1}\in I_{\tau_{1}}}\int\limits_{y_{2}\in I_{\tau_{2}}}\cdots\int\limits_{y_{n-1}\in I_{\tau_{n-1}}}\;\\ \cdot\int\limits_{{U}_{w}(\mathbb{Z})\backslash{U}_{w}(\mathbb{R})}\;\int\limits_{\overline{{U}}_{w}(\mathbb{R})}\,\delta^{\frac{1}{2}}(M)\cdot\delta^{\frac{1}{2}}(L)\cdot\delta_{w}(y)\cdot\delta(y)\cdot\lvert p_{T,R}^{(n)}(Lcwyu)\rvert\,\lvert p_{T,R}^{(n)}(My)\rvert\;d^{*}u\;dy^{\times}.

Our strategy is now to, for each choice of τ\tau, replace the terms with pT,R(n)p_{T,R}^{(n)} with the bound from (6.3) (in the first instance using a choice of a=(a1,,an1)n1a=(a_{1},\ldots,a_{n-1})\in\mathbb{R}^{n-1}, and in the second instance using b=(b1,,bn1)n1b=(b_{1},\ldots,b_{n-1})\in\mathbb{R}^{n-1}). Then we need to find choices of aa and bb for which the corresponding integrals converge and give good bounds.

Recall that if g=utkg=utk is the Iwasawa decomposition of an element gGLn()g\in\operatorname{GL}_{n}(\mathbb{R}), then pT,R(n)(g)=pT,R(n)(t)p_{T,R}^{(n)}(g)=p_{T,R}^{(n)}(t). With this in mind, consider the Iwasawa decomposition wu=u0tkwu=u_{0}tk, where u0Un()u_{0}\in U_{n}(\mathbb{R}), tTn()t\in T_{n}(\mathbb{R}) and kO(n,)k\in O(n,\mathbb{R}). Then

Lcwyu=Lc(wyw1)u0tk=u1Lc(wyw1)tk(u1=(Lcwyw1)1u0(Lcwyw1))Lcwyu=Lc(wyw^{-1})u_{0}tk=u_{1}Lc(wyw^{-1})tk\qquad\mbox{($u_{1}=\big{(}Lcwyw^{-1}\big{)}^{-1}u_{0}\big{(}Lcwyw^{-1}\big{)}$)}

is the Iwasawa form of LcwyuLcwyu, hence |pT,R(n)(Lcwyu)|=|pT,R(n)(Lcwyw1t)|{\big{\lvert}p_{T,R}^{(n)}(Lcwyu)\big{\rvert}=\big{\lvert}p_{T,R}^{(n)}(Lcwyw^{-1}t)\big{\rvert}}. Recall that the Iwasawa form of wuwu is assumed to be u0tku_{0}tk, meaning wu=u0tkwu=u_{0}tk where u0Un()u_{0}\in U_{n}(\mathbb{R}), tTn()t\in T_{n}(\mathbb{R}) and kO(n,)k\in O(n,\mathbb{R}). It can be shown [Jac67] that

(6.3) t2=(1/ξn1ξn1/ξn2ξ2/ξ1ξ1),t^{2}=\left(\begin{smallmatrix}1/\xi_{n-1}&&&&\\ &\xi_{n-1}/\xi_{n-2}&&&\\ &&\ddots&&\\ &&&\xi_{2}/\xi_{1}&\\ &&&&\xi_{1}\end{smallmatrix}\right),

where ξi=ξi(wu)1\xi_{i}=\xi_{i}(wu)\geq 1 for any uU¯wu\in\overline{U}_{w}. For example, in the case n=4n=4 and w=wlong=w(1,1,1,1)w=w_{\mathrm{long}}=w_{(1,1,1,1)}, we find that U¯wlong=U4()\overline{U}_{w_{\mathrm{long}}}=U_{4}(\mathbb{R}) and, for

u=(1x12x13x1401x23x24001x340001),u=\left(\begin{matrix}1&x_{12}&x_{13}&x_{14}\\ 0&1&x_{23}&x_{24}\\ 0&0&1&x_{34}\\ 0&0&0&1\end{matrix}\right),

that

ξ1(wlongu)\displaystyle\xi_{1}(w_{\mathrm{long}}u) =1+x122+x132+x142,\displaystyle=1+x_{12}^{2}+x_{13}^{2}+x_{14}^{2},
ξ2(wlongu)\displaystyle\xi_{2}(w_{\mathrm{long}}u) =1+x232+x242+(x12x24x14)2+(x12x23x13)2+(x13x24x14x23)2,\displaystyle=1+x_{23}^{2}+x_{24}^{2}+\big{(}x_{12}x_{24}-x_{14}\big{)}^{2}+\big{(}x_{12}x_{23}-x_{13}\big{)}^{2}+\big{(}x_{13}x_{24}-x_{14}x_{23}\big{)}^{2},
ξ3(wlongu)\displaystyle\xi_{3}(w_{\mathrm{long}}u) =1+x342+(x23x34x24)2+(x12x23x34x13x34x12x24+x14)2.\displaystyle=1+x_{34}^{2}+\big{(}x_{23}x_{34}-x_{24}\big{)}^{2}+\big{(}x_{12}x_{23}x_{34}-x_{13}x_{34}-x_{12}x_{24}+x_{14}\big{)}^{2}.

In general, the values ξi\xi_{i} are always of the form 11 plus a sum of squares of functions consisting of the entries of uu.

From (6.3), replacing aa with bb, we see that |pT,R(n)(My)|\big{|}p_{T,R}^{(n)}(My)\big{|} is bounded by

δ(M)12M2bδ(y)12y2bTε+(n+4)(n1)4+R(D(n)+n(n1)2)j=1n1B(bj).\ll\delta(M)^{-\frac{1}{2}}\cdot\big{\lVert}M\big{\rVert}^{2b}\cdot\delta(y)^{-\frac{1}{2}}\cdot\big{\lVert}y\big{\rVert}^{2b}\cdot T^{\varepsilon+\frac{(n+4)(n-1)}{4}+R\cdot\big{(}D(n)+\frac{n(n-1)}{2}\big{)}-\sum\limits_{j=1}^{n-1}B(b_{j})}.

To similarly bound |pT,R(n)(Lcwyw1t)|\lvert p_{T,R}^{(n)}(Lcwyw^{-1}t)\rvert, we first remark that since

c=c1(d1d2dn1d1d2dn2d11)=:c1dwhere di=ci1ci+1ci2,c=c_{1}\left(\begin{smallmatrix}d_{1}d_{2}\cdots d_{n-1}&&&\\ &\hskip-30.0ptd_{1}d_{2}\cdots d_{n-2}&&\\ &\ddots&&\\ &&\hskip-5.0ptd_{1}&\\ &&&1\end{smallmatrix}\right)=:c_{1}d\qquad\mbox{where $d_{i}=\frac{c_{i-1}c_{i+1}}{c_{i}^{2}}$},

setting c0=cn:=1c_{0}=c_{n}:=1 (and a0=an:=0a_{0}=a_{n}:=0 as usual), we see that

δ12(c)c2a=δ(c)12i=1n1(ci1ci+1ci2)2ai=k=1n1ck1+2ai14ai+2ai+1.\delta^{-\frac{1}{2}}(c)\cdot\lVert c\rVert^{2a}=\delta(c)^{-\frac{1}{2}}\prod_{i=1}^{n-1}\left(\tfrac{c_{i-1}c_{i+1}}{c_{i}^{2}}\right)^{2a_{i}}=\prod_{k=1}^{n-1}c_{k}^{-1+2a_{i-1}-4a_{i}+2a_{i+1}}.

Therefore, it follows that

|pT,R(n)(Lcwyw1t)|δ(L)12L2aδ(t)12t2ak=1n1ck12ai1+4ai2ai+1δ(wyw1)12wyw12aTε+(n+4)(n1)4+R(D(n)+n(n1)2)j=1n1B(aj).\big{\lvert}p_{T,R}^{(n)}(Lcwyw^{-1}t)\big{\rvert}\;\ll\;\frac{\delta(L)^{-\frac{1}{2}}\cdot\lVert L\rVert^{2a}\cdot\delta(t)^{-\frac{1}{2}}\cdot\lVert t\rVert^{2a}}{\prod\limits_{k=1}^{n-1}c_{k}^{1-2a_{i-1}+4a_{i}-2a_{i+1}}}\cdot\delta(wyw^{-1})^{-\frac{1}{2}}\\ \cdot\lVert wyw^{-1}\rVert^{2a}\cdot T^{\varepsilon+\frac{(n+4)(n-1)}{4}+R\cdot\big{(}D(n)+\frac{n(n-1)}{2}\big{)}-\sum\limits_{j=1}^{n-1}B(a_{j})}.

Recall that if t=t(u)t=t(u) is as in (6.3), if we define, for a=(a1,,an1),b=(b1,,bn1)n1a=(a_{1},\ldots,a_{n-1}),b=(b_{1},\ldots,b_{n-1})\in\mathbb{R}^{n-1},

K(w;a):=\displaystyle K(w;a):= vVnc1=1cn1=1|Sw(ψL,ψMv,c)|i=1n1ci12ai1+4ai2ai+1,\displaystyle\sum_{v\in V_{n}}\sum_{c_{1}=1}^{\infty}\cdots\sum_{c_{n-1}=1}^{\infty}\frac{\lvert S_{w}(\psi_{L},\psi_{M}^{v},c)\rvert}{\prod\limits_{i=1}^{n-1}c_{i}^{1-2a_{i-1}+4a_{i}-2a_{i+1}}},
X(w;a):=\displaystyle X(w;a):= Uw()\Uw()U¯w()δ(t)12t2adu,\displaystyle\int\limits_{{U}_{w}(\mathbb{Z})\backslash{U}_{w}(\mathbb{R})}\;\int\limits_{\overline{{U}}_{w}(\mathbb{R})}\delta(t)^{-\frac{1}{2}}\cdot\lVert t\rVert^{2a}\;d^{*}u,

and for a given choice of τ=(τ1,,τn1){0,1}n1\tau=(\tau_{1},\ldots,\tau_{n-1})\in\{0,1\}^{n-1}

Y(τ,w;a,b):=\displaystyle Y(\tau,w;a,b):= y1Iτ1y2Iτ2yn1Iτn1y2bwyw12a𝑑y×,\displaystyle\int\limits_{y_{1}\in I_{\tau_{1}}}\int\limits_{y_{2}\in I_{\tau_{2}}}\cdots\int\limits_{y_{n-1}\in I_{\tau_{n-1}}}\lVert y\rVert^{2b}\cdot\lVert wyw^{-1}\rVert^{2a}\;dy^{\times},

then the bound on |w(τ)|\lvert\mathcal{I}_{w}{(\tau)}\rvert given in (6.2) can be replaced by

(6.4) |w(τ)|Tε+(n+4)(n1)2+R(2D(n)+n(n1))j=1n1(B(aj)+B(bj))K(w;a)X(w;a)Y(τ,w;a,b)L2aM2b.\lvert\mathcal{I}_{w}{(\tau)}\rvert\;\ll\;T^{\varepsilon+\frac{(n+4)(n-1)}{2}+R\cdot\big{(}2D(n)+n(n-1)\big{)}-\sum\limits_{j=1}^{n-1}\big{(}B(a_{j})+B(b_{j})\big{)}}\\ \cdot K(w;a)\cdot X(w;a)\cdot Y({\tau,}w;a,b)\cdot\lVert L\rVert^{2a}\cdot\lVert M\rVert^{2b}.

We remark that in simplifying/finding Y(τ,w;a,b)Y({\tau,w;a,b}), we have used (6.2). The basic strategy to prove Proposition 6.1 is now clear: we first find aa such that both K(w;a)K(w;a) and X(w;a)X(w;a) converge; then given this choice of aa, we determine a particular value of bb for which Y(τ,w;a,b)Y({\tau,}w;a,b) converges as well; finally, we work out the corresponding bounds on L2a\lVert L\rVert^{2a}, M2b\lVert M\rVert^{2b} and j=1n1(B(aj)+B(bj))\sum\limits_{j=1}^{n-1}\big{(}B(a_{j})+B(b_{j})\big{)}.

6.3. Restrictions on the parameters aa and bb

The trivial bound (see [DR98]) for the Kloosterman sum is given by

S(1,1,c)δ12(c)=c1c2cn1.S(1,1,c)\ll\delta^{\frac{1}{2}}(c)=c_{1}c_{2}\cdots c_{n-1}.

Hence K(w;a)K(w;a) is convergent whenever aa is chosen such that

c2a=k=1n1ck2ak14ak+2ak+1δ12ε(c),{\lVert c\rVert^{2a}=\prod_{k=1}^{n-1}c_{{k}}^{2a_{k-1}-4a_{k}+2a_{k+1}}\ll\delta^{-\frac{1}{2}-\varepsilon}(c)},

From (6.1), if we set aj=j(nj)2(1+ε)a_{j}=\frac{j(n-j)}{2}(1+\varepsilon), then c2a=δ1ε(c)δ12ε(c)\lVert c\rVert^{2a}=\delta^{-1-\varepsilon}(c)\ll\delta^{-\frac{1}{2}-\varepsilon}(c). More generally, K(w;a)K(w;a) converges in the following case:

(6.1) aj:=ρ+j(nj)2(1+ε),ρ>0,j=1,,n1.\boxed{a_{j}:=\rho+\frac{j(n-j)}{2}(1+\varepsilon),\quad{\rho>0},\ j=1,\ldots,n-1.}

That this choice of aa makes K(w;a)K(w;a) converge is a consequence of the easily verifiable fact that

c2a=(c1cn1)2ρδ1ε(c).{\lVert c\rVert^{2a}=(c_{1}c_{n-1})^{-2\rho}\cdot\delta^{-1-\varepsilon}(c).}

We assume henceforth that aa satisfies (6.1).

We next consider the convergence of X(w;a)X(w;a). Recall that the Iwasawa form of wuwu is assumed to be u0tku_{0}tk, meaning wu=u0tkwu=u_{0}tk where u0Un()u_{0}\in U_{n}(\mathbb{R}), tTn()t\in T_{n}(\mathbb{R}) and kO(n,)k\in O(n,\mathbb{R}). Indeed, tt is given by (6.3). Then

X(w;a)=Uw()\Uw()U¯w()δ(t)12t2aduUw()\Uw()U¯w()δ32ε(t)du.X(w;a)=\int\limits_{{U}_{w}(\mathbb{Z})\backslash{U}_{w}(\mathbb{R})}\;\int\limits_{\overline{{U}}_{w}(\mathbb{R})}\delta(t)^{-\frac{1}{2}}\cdot\lVert t\rVert^{2a}\;d^{*}u\ll\int\limits_{{U}_{w}(\mathbb{Z})\backslash{U}_{w}(\mathbb{R})}\;\int\limits_{\overline{{U}}_{w}(\mathbb{R})}\delta^{-\frac{3}{2}-\varepsilon}(t)\;d^{*}u.

The fact that the right hand side converges is a consequence of Jacquet [Jac67].

We now turn to the convergence of Y(τ,w;a,b)Y(\tau,w;a,b). Applying Lemma A.1 (which describes wyw12a\lVert wyw^{-1}\rVert^{2a}), we see that

Y(τ,w;a,b)\displaystyle Y({\tau,w;a,b}) =Iτ(i=1sj=1niynn^i+j2b(nn^i+j)2(an^i1a(n^i1+j)+an^i))𝑑y×=i=1sj=1niYnn^i+j(τ,w;a,b),\displaystyle=\int\limits_{I_{\tau}}\left(\prod_{i=1}^{s}\prod_{j=1}^{n_{i}}y_{n-\widehat{n}_{i}+j}^{2b_{(n-\widehat{n}_{i}+j)}-2\big{(}a_{\widehat{n}_{i-1}}-a_{(\widehat{n}_{i-1}+j)}+a_{\widehat{n}_{i}}\big{)}}\right)dy^{\times}=\prod_{i=1}^{s}\prod_{j=1}^{n_{i}}Y_{n-\widehat{n}_{i}+j}({\tau,w;a,b}),

where

Ynn^i+j(τ,w;a,b):=Iτnn^i+jynn^i+j2bnn^i+j2(an^i1an^i1+j+an^i)dynn^i+jynn^i+j.Y_{n-\widehat{n}_{i}+j}({\tau,w;a,b}):=\int\limits_{I_{\tau_{n-\widehat{n}_{i}+j}}}y_{n-\widehat{n}_{i}+j}^{2b_{n-\widehat{n}_{i}+j}-2(a_{\widehat{n}_{i-1}}-a_{\widehat{n}_{i-1}+j}+a_{\widehat{n}_{i}})}\;\frac{dy_{n-\widehat{n}_{i}+j}}{y_{n-\widehat{n}_{i}+j}}.

Hence, in order to bound Y(τ,w;a,b)Y({\tau,w;a,b}) (and thereby show that w(τ)\mathcal{I}_{w}{(\tau)} converges), we must choose b=(b1,,bn1)b=(b_{1},\ldots,b_{n-1}) such that Ynn^i+j(τ,w;a,b)Y_{n-\widehat{n}_{i}+j}({\tau,w;a,b}) converges. Clearly

(6.2) bnn^i+j=an^i1an^i1+j+an^i+(1)τnn^i+jε2,(i=1,,s,j=1,,ni)\boxed{b_{n-\widehat{n}_{i}+j}=a_{\widehat{n}_{i-1}}-a_{\widehat{n}_{i-1}+j}+a_{\widehat{n}_{i}}+(-1)^{\tau_{{n-\widehat{n}_{i}+j}}}\cdot\tfrac{\varepsilon}{2},\quad\quad\mbox{($i=1,\ldots,s,\ j=1,\ldots,n_{i}$)}}

suffices, since making this choice implies that, for each k=1,,n1k=1,\ldots,n-1,

Yk(τ,w;a,b)={01yεdyy if τk=0,1yεdyy if τk=1,Y_{k}({\tau,w;a,b})=\begin{cases}\displaystyle{\int_{0}^{1}}y^{\varepsilon}\;\frac{dy}{y}&\mbox{ if }\tau_{k}=0,\\ \\ \displaystyle{\int_{1}^{\infty}}y^{-\varepsilon}\;\frac{dy}{y}&\mbox{ if }\tau_{k}=1,\end{cases}

which converges (and gives the same value 1ε\frac{1}{\varepsilon}) in either case.

6.4. Proof of Proposition 6.1

We have now shown that if w=w(n1,,nr)w=w_{(n_{1},\ldots,n_{r})} and we choose aa as in (6.1) and bb via (6.2) accordingly, the right hand side of (6.4) converges, hence gives a bound for |w|\lvert\mathcal{I}_{w}\rvert. Therefore, in order to complete the proof of Proposition 6.1, we need to first show that

L2aM2bi=1n1(imi)2ρ+n2+14,\lVert L\rVert^{2a}\cdot\lVert M\rVert^{2b}\ll\prod_{i=1}^{n-1}\big{(}\ell_{i}m_{i}\big{)}^{2\rho+\frac{n^{2}+1}{4}},

and second that the given choice of aa and bb gives the claimed bound for the power of TT appearing in (6.4).

To complete the first of these tasks we note that, by (6.1) and the fact that j(nj)j(n-j) is maximized (in jj) when j=n/2j=n/2, we have

(6.1) aj=ρ+j(nj)2(1+ε)ρ+n28(1+ε)<ρ+n2+18a_{j}=\rho+\frac{j(n-j)}{2}(1+\varepsilon)\leq\rho+\frac{n^{2}}{8}(1+\varepsilon)<\rho+\frac{n^{2}+1}{8}

for ε<1/n2\varepsilon<1/n^{2} and 1jn11\leq j\leq n-1. Similarly, using (6.1) and (6.2) we compute that, for 1is1\leq i\leq s and 1jni1\leq j\leq n_{i},

(6.2) bnn^i+j=ρ+12(j2+j(2n^i1n)+n^i(nn^i))+εb_{n-\widehat{n}_{i}+j}=\rho+\frac{1}{2}\big{(}j^{2}+j(2\widehat{n}_{i-1}-n)+\widehat{n}_{i}(n-\widehat{n}_{i})\big{)}+\varepsilon

for ε\varepsilon sufficiently small. Note that the right hand side of (6.2)is a concave up parabola in jj, and therefore, on the interval 1jni1\leq j\leq n_{i}, can attain its maximum only at j=1j=1 or j=nij=n_{i}. So, if we can show that bnn^i+1b_{n-\widehat{n}_{i}+1} and bnn^i+nib_{n-\widehat{n}_{i}+ni} both satisfy a suitable upper bound, then the same bound will hold for all 1jni1\leq j\leq n_{i}.

We consider first the endpoint j=nij=n_{i}. Using (6.2) and the fact that n^ini=n^i1\widehat{n}_{i}-n_{i}=\widehat{n}_{i-1}, we find that

bnn^i+ni=ρ+12n^i1(nn^i1)+ε.b_{n-\widehat{n}_{i}+n_{i}}=\rho+\frac{1}{2}\widehat{n}_{i-1}(n-\widehat{n}_{i-1})+\varepsilon.

Again, j(nj)j(n-j) is maximized when j=n/2j=n/2, so we conclude that

(6.3) bnn^i+niρ+n28+ε<ρ+n2+18b_{n-\widehat{n}_{i}+n_{i}}\leq\rho+\frac{n^{2}}{8}+\varepsilon<\rho+\frac{n^{2}+1}{8}

for ε\varepsilon sufficiently small.

Next we consider the endpoint j=1j=1. From (6.2) we find that

(6.4) bnn^i+1\displaystyle b_{n-\widehat{n}_{i}+1} =ρ+12(1n+n^i(nn^i)+2n^i1)+ε\displaystyle=\rho+\frac{1}{2}\big{(}1-n+\widehat{n}_{i}(n-\widehat{n}_{i})+2\widehat{n}_{i-1}\big{)}+\varepsilon
ρ+12(1n+n^i(nn^i)+2n^i)+ε,\displaystyle\leq\rho+\frac{1}{2}\big{(}-1-n+\widehat{n}_{i}(n-\widehat{n}_{i})+2\widehat{n}_{i}\big{)}+\varepsilon,

the last step because n^i1=n^inin^i1\widehat{n}_{i-1}=\widehat{n}_{i}-n_{i}\leq\widehat{n}_{i}-1. We find using calculus that, as a function of n^i\widehat{n}_{i}, the right hand side of (6.4) is maximized when n^i=(n+2)/2\widehat{n}_{i}=(n+2)/2. So

(6.5) bnn^i+1\displaystyle b_{n-\widehat{n}_{i}+1} ρ+12(1n+n+22(nn+22)+n+2)+ε\displaystyle\leq\rho+\frac{1}{2}\bigg{(}-1-n+\frac{n+2}{2}\bigg{(}n-\frac{n+2}{2}\bigg{)}+n+2\bigg{)}+\varepsilon
=ρ+n28+ερ+n2+18\displaystyle=\rho+\frac{n^{2}}{8}+\varepsilon\leq\rho+\frac{n^{2}+1}{8}

for ε\varepsilon small enough. From (6.3) and (6.5) it follows, again, that

bnn^i+jρ+n2+18b_{n-\widehat{n}_{i}+j}\leq\rho+\frac{n^{2}+1}{8}

for all 1is1\leq i\leq s and 1jni1\leq j\leq n_{i}. This and (6.1) yield the desired bound on L2aM2b\lVert L\rVert^{2a}\cdot\lVert M\rVert^{2b}.

The second task is accomplished using Lemma A.9. ∎

7. Bounding the Eisenstein spectrum \mathcal{E}

Recall that if L=(1,,n1)L=(\ell_{1},\ldots,\ell_{n-1}), M=(m1,,mn1)n1M=(m_{1},\ldots,m_{n-1})\in{\mathbb{Z}}^{n-1} with i=1n1imi0\prod\limits_{i=1}^{n-1}\ell_{i}m_{i}\neq 0, then, by Theorem 4.1, the Eisenstein contribution to the Kuznetsov trace formula is given by

=𝒫Φ𝒫,Φ,\displaystyle\mathcal{E}=\sum_{\mathcal{P}}\sum_{\Phi}\mathcal{E}_{\mathcal{P},\Phi},

where

𝒫,Φ:=c𝒫n1s1++nrsr=0Re(sj)=0AE𝒫,Φ(L,s)AE𝒫,Φ(M,s)¯|pT,Rn,#(α(𝒫,Φ)(s))|2ds.\displaystyle\mathcal{E}_{\mathcal{P},\Phi}:=c_{\mathcal{P}}\hskip-4.0pt\underset{\text{\rm Re}(s_{j})=0}{\int\limits_{n_{1}s_{1}+\cdots+n_{r}s_{r}=0}}\hskip-11.0ptA_{E_{\mathcal{P},\Phi}}(L,s)\,\overline{A_{E_{\mathcal{P},\Phi}}(M,s)}\cdot\Big{|}p_{T,R}^{n,\#}\big{(}\alpha_{{}_{(\mathcal{P},\Phi)}}(s)\big{)}\Big{|}^{2}\;ds.

In this section we give bounds for \mathcal{E} in the case that L=(,1,,1)L=(\ell,1,\ldots,1) and M=(m,1,,1)M=(m,1,\ldots,1) with ,m0\ell,m\neq 0.

7.1. The Eisenstein contribution \mathcal{E} to the Kuznetsov trace formula

The main result of this section is the following.

Theorem 7.1 (Bounding the Eisenstein contribution \mathcal{E}).

Fix n2n\geq 2 and a sufficiently large integer R>0R>0. Let L=(,1,,1)L=(\ell,1,\ldots,1), M=(m,1,,1)n1M=(m,1,\ldots,1)\in{\mathbb{Z}}^{n-1} with ,m0\ell,m\neq 0. Then, assuming the Lower bound conjecture for Rankin-Selberg L-functions (see (1.2)), for TT\to\infty we have the bound

𝒫Φ|𝒫,Φ|(m)121n2+1+εTR((2nn)2n)+ε.\sum_{\mathcal{P}}\sum_{\Phi}\left|\mathcal{E}_{\mathcal{P},\Phi}\right|\;\ll\;(\ell m)^{\frac{1}{2}-\frac{1}{n^{2}+1}+\varepsilon}\cdot T^{R\cdot\left(\binom{2n}{n}-2^{n}\right)\;\;+\;\varepsilon}.

7.2. Proof of Theorem 7.1.1

Proof.

We proceed by induction on nn, beginning with the case n=2n=2. In this case, the only parabolic subgroup is the minimum parabolic, or Borel, subgroup =𝒫1,1\mathcal{B}=\mathcal{P}_{1,1}, and the only function Φ\Phi corresponding to \mathcal{B} (see Definition 3.2) is the constant function Φ=1\Phi=1. The Eisenstein contribution in this case, then, is simply the quantity ,1\mathcal{E}_{\mathcal{B},1}.

By Theorem 4.1 in the case n=2n=2, we have

,1=cRes1=0AE,1(,s)AE,1(m,s)¯|pT,R2,#(α(,1)(s))|2𝑑s1,\mathcal{E}_{\mathcal{B},1}=\;c_{\mathcal{B}}\int\limits_{\operatorname{Re}{s_{1}}=0}A_{E_{\mathcal{B},1}}(\ell,s)\,\overline{A_{E_{\mathcal{B},1}}(m,s)}\cdot\Big{|}p_{T,R}^{2,\#}\big{(}\alpha_{{}_{(\mathcal{B},1)}}(s)\big{)}\Big{|}^{2}\;ds_{1},

where s=(s1,s1)s=(s_{1},-s_{1}). Now note that, by (3.7), α(,1)(s)=s\alpha_{{}_{(\mathcal{B},1)}}(s)=s. Moreover, by Definition 1.1, we have R(2)1\mathcal{F}_{R}^{(2)}\equiv 1, so by Definition 1.4, we have

pT,R2,#(α(,1)(s))=es12/T2Γ(2R+1+2s14)Γ(2R+12s14).\displaystyle p_{T,R}^{2,\#}\big{(}\alpha_{{}_{(\mathcal{B},1)}}(s)\bigr{)}\;=\;e^{{s_{1}^{2}}/{T^{2}}}\Gamma\left(\textstyle{\frac{2R+1+2s_{1}}{4}}\right)\Gamma\left(\textstyle{\frac{2R+1-2s_{1}}{4}}\right).

Furthermore, we see from Proposition 4.5 that

|AE,1(,s)|\displaystyle\left|A_{E_{\mathcal{B},1}}\big{(}\ell,s\big{)}\right| |ζ(1+2s1)|1c1,c2>0c1c2=|c1α1c2α2|ε|Γ(1+2s12)ζ(1+2s1)|1.\displaystyle\ll\big{|}\zeta^{*}\big{(}1+2s_{1}\big{)}\big{|}^{-1}\underset{c_{1}c_{2}=\ell}{\sum_{c_{1},c_{2}\in\mathbb{Z}_{>0}}}\left|c_{1}^{\alpha_{1}}c_{2}^{\alpha_{2}}\right|\ll\;{\ell^{\varepsilon}\left|\Gamma\bigl{(}{\textstyle{\frac{1+2s_{1}}{2}}}\bigr{)}\zeta\big{(}1+2s_{1}\big{)}\right|^{-1}.}

Then

|,1|\displaystyle\big{|}\mathcal{E}_{\mathcal{B},1}\big{|} (m)εRe(s1)=0es12/T2|Γ(2R+1+2s14)Γ(2R+12s14)|2|Γ(1+2s12)ζ(1+2s1)|2|ds1|.\displaystyle\;\ll\;(\ell m)^{\varepsilon}\int\limits_{\operatorname{Re}(s_{1})=0}e^{{s_{1}^{2}}/{T^{2}}}\frac{\left|\Gamma\left(\textstyle{\frac{2R+1+2s_{1}}{4}}\right)\Gamma\left(\textstyle{\frac{2R+1-2s_{1}}{4}}\right)\right|^{2}}{\left|\Gamma\bigl{(}{\textstyle{\frac{1+2s_{1}}{2}}}\bigr{)}\zeta\big{(}1+2s_{1}\big{)}\right|^{2}}\;|ds_{1}|.

We may restrict our integration to the domain |Im(s)|T|\operatorname{Im}(s)|\leq T, since es12/T2e^{{s_{1}^{2}}/{T^{2}}} decays exponentially otherwise. On this domain, we use Stirling’s bound (9.1) for the Gamma function, as well as the Vinogradov bound

|ζ(1+it)|1(1+|t|)ε,(t).|\zeta(1+it)|^{-1}\ll(1+|t|)^{\varepsilon},\qquad(t\in\mathbb{R}).

We get

|,1|(m)εRe(s1)=0Im(s1)T|1+s1|2R1+ε|ds1|,\big{|}\mathcal{E}_{\mathcal{B},1}\big{|}\;\ll\;(\ell m)^{\varepsilon}\int\limits_{\begin{subarray}{c}\operatorname{Re}(s_{1})=0\\ \operatorname{Im}(s_{1})\leq T\end{subarray}}\big{|}1+s_{1}\big{|}^{2R-1+\varepsilon}\;|ds_{1}|,

from which it follows immediately that |,1|T2R+ε.\big{|}\mathcal{E}_{\mathcal{B},1}\big{|}\;\ll\;T^{2R+\varepsilon}. So our desired result holds in the case n=2n=2.

We now proceed to the general case. For n>2n>2, in order to establish bounds for 𝒫,Φ\mathcal{E}_{\mathcal{P},\Phi}, we need to know that our main theorem is true for all k<nk<n. The reason this is needed is because we have to bound Rankin-Selberg L-functions L(s,ϕk×ϕk)L(s,\phi_{k}\times\phi_{k^{\prime}}) with 2k,k<n2\leq k,k^{\prime}<n. This will require knowing the Weyl law with harmonic weights (Theorem 7.3) for 2k,k<n2\leq k,k^{\prime}<n. We may assume by induction, however, that this is indeed the case, i.e., the Weyl law with harmonic weights holds for all 2k<n2\leq k<n.

Now recall that, for the parabolic 𝒫\mathcal{P} associated to a partition n=n1++nrn=n_{1}+\cdots+n_{r}, we have

𝒫,Φ=n1s1++nrsr=0Re(sj)=0AE𝒫,Φ(L,s)AE𝒫,Φ(M,s)¯|pT,Rn,#(α(𝒫,Φ)(s))|2ds\mathcal{E}_{\mathcal{P},\Phi}=\underset{\text{\rm Re}(s_{j})=0}{\int\limits_{n_{1}s_{1}+\cdots+n_{r}s_{r}=0}}\hskip-6.0ptA_{E_{\mathcal{P},\Phi}}(L,s)\,\overline{A_{E_{\mathcal{P},\Phi}}(M,s)}\cdot\Big{|}p_{T,R}^{n,\#}\big{(}\alpha_{{}_{(\mathcal{P},\Phi)}}(s)\big{)}\Big{|}^{2}\;ds

where α𝒫,Φ(s)\alpha_{{}_{\mathcal{P},\Phi}}(s) is given by (see (3.7))

(α1,1+s1,,α1,n1+s1n1terms,α2,1+s2,,α2,n2+s2n2terms.,αr,1+sr,,αr,nr+srnrterms),\displaystyle\bigg{(}\overbrace{\alpha_{1,1}+s_{1},\;\ldots\;,\alpha_{1,n_{1}}+s_{1}}^{n_{1}\;\,\text{\rm terms}},\quad\overbrace{\alpha_{2,1}+s_{2},\;\ldots\;,\alpha_{2,n_{2}}+s_{2}}^{n_{2}\;\,\text{\rm terms}}.\quad\ldots\quad,\overbrace{\alpha_{r,1}+s_{r},\;\ldots\;,\alpha_{r,n_{r}}+s_{r}}^{n_{r}\;\,\text{\rm terms}}\bigg{)},

Since i=1nkαk,i=0\sum\limits_{i=1}^{n_{k}}\alpha_{k,i}=0 for all 1kr1\leq k\leq r we see that k=1ri=1nk(αk,i+sk)2=k=1ri=1nk(αk,i2+sk2).\sum\limits_{k=1}^{r}\sum\limits_{i=1}^{n_{k}}(\alpha_{k,i}+s_{k})^{2}=\sum\limits_{k=1}^{r}\sum\limits_{i=1}^{n_{k}}(\alpha_{k,i}^{2}+s_{k}^{2}).

Now, for any β=(β1,,βn)(i)n\beta=(\beta_{1},\ldots,\beta_{n})\in\left(i\mathbb{R}\right)^{n} where β^n=0\widehat{\beta}_{n}=0 we have

pT,Rn,#(β):=exp(β12+β22++βn22T2)R(n)(β2)1i<jn|Γ(2R+1+βiβj4)|2.\displaystyle p_{T,R}^{n,\#}\big{(}\beta\big{)}\;:=\;\text{\rm exp}\left(\frac{\beta_{1}^{2}+\beta_{2}^{2}+\cdots+\beta_{n}^{2}}{2T^{2}}\right)\cdot\mathcal{F}_{R}^{(n)}(\tfrac{\beta}{2})\prod_{1\leq\,i<j\,\leq n}\left|\Gamma\left(\textstyle{\frac{2R+1+\beta_{i}-\beta_{j}}{4}}\right)\right|^{2}.

It follows that

pT,Rn,#(α(𝒫,Φ)(s))=exp(k=1ri=1nk(αk,i2+sk2)2T2)R(n)(α(𝒫,Φ)(s)2)k=1rnk11i<jnk|Γ(2R+1+αk,iαk,j4)|21k<kri=1nkj=1nk|Γ(2R+1+sksk+αk,iαk,j4)|2.p_{T,R}^{n,\#}\big{(}\alpha_{{}_{(\mathcal{P},\Phi)}}(s)\big{)}\;=\text{\rm exp}\left(\frac{\sum\limits_{k=1}^{r}\sum\limits_{i=1}^{n_{k}}\left(\alpha_{k,i}^{2}+s_{k}^{2}\right)}{2T^{2}}\right)\mathcal{F}_{R}^{(n)}\left(\tfrac{\alpha_{{}_{(\mathcal{P},\Phi)}}(s)}{2}\right)\\ \cdot\underset{n_{k}\neq 1}{\prod_{k=1}^{r}}\prod_{1\leq\,i<j\,\leq n_{k}}\left|\Gamma\left(\textstyle{\frac{2R+1+\alpha_{k,i}-\alpha_{k,j}}{4}}\right)\right|^{2}\cdot\prod_{1\leq k<k^{\prime}\leq r}\;\prod_{i=1}^{n_{k}}\;\prod_{j=1}^{n_{k^{\prime}}}\left|\Gamma\left(\textstyle{\frac{2R+1+s_{k}-s_{k^{\prime}}+\alpha_{k,i}-\alpha_{k^{\prime},j}}{4}}\right)\right|^{2}.

By Proposition 4.5, the mthm^{th} coefficient of E𝒫,ΦE_{\mathcal{P},\Phi} is given by

AE𝒫,Φ((m,1,,1),s)\displaystyle A_{E_{\mathcal{P},\Phi}}\big{(}(m,1,\ldots,1),s\big{)} =k=1rnk1L(1,Adϕk)121i<jrL(1+sisj,ϕi×ϕj)1\displaystyle=\underset{n_{k}\neq 1}{\prod_{k=1}^{r}}L^{*}\big{(}1,\text{\rm Ad}\;\phi_{k}\big{)}^{-\frac{1}{2}}\prod_{1\leq i<j\leq r}L^{*}\big{(}1+s_{i}-s_{j},\;\phi_{i}\times\phi_{j}\big{)}^{-1}
1c1,c2,,crc1c2cr=mλϕ1(c1)λϕr(cr)c1s1crsr\displaystyle\hskip 50.0pt\cdot\underset{c_{1}c_{2}\cdots c_{r}=m}{\sum_{1\leq c_{1},c_{2},\ldots,c_{r}\,\in\,\mathbb{Z}}}\hskip-10.0pt\lambda_{\phi_{1}}(c_{1})\cdots\lambda_{\phi_{r}}(c_{r})\cdot c_{1}^{s_{1}}\cdots c_{r}^{s_{r}}

up to a non-zero constant factor with absolute value depending only on nn. To bound the divisor sum above we will use the bound of Luo-Rudnick-Sarnak [LRS99] for the mthm^{th} Hecke Fourier coefficient of a GL(κ)\operatorname{GL}(\kappa) (for κ2)\kappa\geq 2) Hecke-Maass cusp form ϕ\phi given by

|λϕ(m,1,,1)|m121κ2+1+ε.\left|\lambda_{\phi}(m,1,\ldots,1)\right|\leq m^{\frac{1}{2}-\frac{1}{\kappa^{2}+1}+\varepsilon}.

(A slightly stronger result has been obtained by Kim and Sarnak [Kim03]. However, the stated result above is sufficient for our purposes.) We immediately obtain the following bound for the divisor sum

1c1,c2,,crc1c2cr=m|λϕ1(c1)λϕr(cr)c1s1crsr|m121n2+1+ε.\underset{c_{1}c_{2}\cdots c_{r}=m}{\sum_{1\leq c_{1},c_{2},\ldots,c_{r}\,\in\,\mathbb{Z}}}\hskip-10.0pt\left|\lambda_{\phi_{1}}(c_{1})\cdots\lambda_{\phi_{r}}(c_{r})\cdot c_{1}^{s_{1}}\cdots c_{r}^{s_{r}}\right|\ll m^{\frac{1}{2}-\frac{1}{n^{2}+1}+\varepsilon}.

It follows that

|𝒫,Φ|\displaystyle\left|\mathcal{E}_{\mathcal{P},\Phi}\right|\; (m)121n2+1+εexp(k=1ri=1nkαk,i2T2)n1s1++nrsr=0Re(sj)=0,Im(sj)T|R(n)(α(𝒫,Φ)(s)2)|2\displaystyle\ll(m\ell)^{\frac{1}{2}-\frac{1}{n^{2}+1}+\varepsilon}\cdot\text{\rm exp}\left(\frac{\sum\limits_{k=1}^{r}\sum\limits_{i=1}^{n_{k}}\alpha_{k,i}^{2}}{T^{2}}\right)\underset{\text{\rm Re}(s_{j})=0,\;\text{\rm Im}(s_{j})\ll T}{\int\limits_{n_{1}s_{1}+\cdots+n_{r}s_{r}=0}}\left|\mathcal{F}_{R}^{(n)}\left(\tfrac{\alpha_{{}_{(\mathcal{P},\Phi)}}(s)}{2}\right)\right|^{2}
(k=1rnk11i<jnk|Γ(2R+1+αk,iαk,j4)|4)(1k<kri=1nkj=1nk|Γ(2R+1+sksk+αk,iαk,j4)|4)\displaystyle\hskip-30.0pt\cdot\left(\underset{n_{k}\neq 1}{\prod_{k=1}^{r}}\prod_{1\leq\,i<j\,\leq n_{k}}\left|\Gamma\left(\textstyle{\frac{2R+1+\alpha_{k,i}-\alpha_{k,j}}{4}}\right)\right|^{4}\right)\cdot\left(\prod_{1\leq k<k^{\prime}\leq r}\;\prod_{i=1}^{n_{k}}\;\prod_{j=1}^{n_{k^{\prime}}}\left|\Gamma\left(\textstyle{\frac{2R+1+s_{k}-s_{k^{\prime}}+\alpha_{k,i}-\alpha_{k^{\prime},j}}{4}}\right)\right|^{4}\right)
(k=1rnk1|L(1,Adϕk)|1)(1k<kr|L(1+sksk,ϕk×ϕk)|2|ds|)\displaystyle\hskip 17.0pt\cdot\left(\underset{n_{k}\neq 1}{\prod_{k=1}^{r}}\big{|}L^{*}\big{(}1,\text{\rm Ad}\;\phi_{k}\big{)}\big{|}^{-1}\right)\cdot\left(\prod_{1\leq k<k^{\prime}\leq r}\big{|}L^{*}\big{(}1+s_{k}-s_{k^{\prime}},\;\phi_{k}\times\phi_{k^{\prime}}\big{)}\big{|}^{-2}\;|ds|\right)
(m)121n2+1+εk=1rnk1exp(αk,12++αk,nk2T2)\displaystyle\ll\;(m\ell)^{\frac{1}{2}-\frac{1}{n^{2}+1}+\varepsilon}\;\underset{n_{k}\neq 1}{\prod_{k=1}^{r}}\text{\rm exp}\left(\frac{\alpha_{k,1}^{2}+\cdots+\alpha_{k,n_{k}}^{2}}{T^{2}}\right)
n1s1++nrsr=0Re(sj)=0,Im(sj)T|R(n)(α(𝒫,Φ)(s)2)|2k=1rnk11|L(1,Adϕk)|1i<jnk|Γ(2R+1+αk,iαk,j4)|4|Γ(1+αk,iαk,j2)|2\displaystyle\cdot\hskip-16.0pt\underset{\text{\rm Re}(s_{j})=0,\;\text{\rm Im}(s_{j})\ll T}{\int\limits_{n_{1}s_{1}+\cdots+n_{r}s_{r}=0}}\hskip-10.0pt\left|\mathcal{F}_{R}^{(n)}\left(\tfrac{\alpha_{{}_{(\mathcal{P},\Phi)}}(s)}{2}\right)\right|^{2}\;\underset{n_{k}\neq 1}{\prod_{k=1}^{r}}\frac{1}{\left|L(1,\,\text{\rm Ad}\;\phi_{k})\right|}\;\;\prod_{1\leq\,i<j\,\leq n_{k}}\frac{\left|\Gamma\left(\textstyle{\frac{2R+1+\alpha_{k,i}-\alpha_{k,j}}{4}}\right)\right|^{4}}{\left|\Gamma\left(\frac{1+\alpha_{k,i}-\alpha_{k,j}}{2}\right)\right|^{2}}
1k<kr1|L(1+sksk,ϕk×ϕk)|2i=1nkj=1nk|Γ(2R+1+sksk+αk,iαk,j4)|4|Γ(1+sksk+αk,iαk,j2)|2|ds|.\displaystyle\hskip 20.0pt\cdot\prod_{1\leq k<k^{\prime}\leq r}\frac{1}{\left|L\big{(}1+s_{k}-s_{k^{\prime}},\;\phi_{k}\times\phi_{k^{\prime}}\big{)}\right|^{2}}\;\prod_{i=1}^{n_{k}}\;\prod_{j=1}^{n_{k^{\prime}}}\frac{\left|\Gamma\left(\textstyle{\frac{2R+1+s_{k}-s_{k^{\prime}}+\alpha_{k,i}-\alpha_{k^{\prime},j}}{4}}\right)\right|^{4}}{\left|\Gamma\left(\textstyle{\frac{1+s_{k}-s_{k^{\prime}}+\alpha_{k,i}-\alpha_{k^{\prime},j}}{2}}\right)\right|^{2}}\;|ds|.
Lemma 7.1.

Assume |sk|T1+ε|s_{k}|\ll T^{1+\varepsilon} and |αk,j|T1+ε|\alpha_{k,j}|\ll T^{1+\varepsilon} for 1kr1\leq k\leq r and 1jnk1\leq j\leq n_{k}. Then for α:=α(𝒫,Φ)(s)\alpha:=\alpha_{{}_{(\mathcal{P},\Phi)}}(s) and α(k)\alpha^{(k)} as in Definition A.16, we have

|R(n)(α(𝒫,Φ)(s))|2(k=1rnk1|R(nk)(α(k))|2)TRB(n)+ε\left|\mathcal{F}_{R}^{(n)}\big{(}\alpha_{{}_{(\mathcal{P},\Phi)}}(s)\big{)}\right|^{2}\ll\Bigg{(}\underset{n_{k}\neq 1}{\prod_{k=1}^{r}}\left|\mathcal{F}_{R}^{(n_{k})}\big{(}\alpha^{(k)}\big{)}\right|^{2}\Bigg{)}\cdot T^{R\cdot B(n)+\varepsilon}

where B(n)=2D(n)2k=1rnk1D(nk).B(n)=2D(n)-2\underset{n_{k}\neq 1}{\sum\limits_{k=1}^{r}}D(n_{k}).

Proof.

This follows immediately from Lemma A.27. ∎

It follows from Lemma 7.1 that for |α(k)|2=αk,12++αk,nk2|\alpha^{(k)}|^{2}=\alpha_{k,1}^{2}+\cdots+\alpha_{k,n_{k}}^{2}, we have

|𝒫,Φ|\displaystyle\left|\mathcal{E}_{\mathcal{P},\Phi}\right| (m)121n2+1+εTRB(n)+εk=1rnk1exp(|α(k)|2T2)|R(nk)(α(k)2)|21i<jnk|Γ(2R+1+αk,iαk,j4)|4|Γ(1+αk,iαk,j2)|2|L(1,Adϕk)|\displaystyle\ll(m\ell)^{\frac{1}{2}-\frac{1}{n^{2}+1}+\varepsilon}\;T^{R\cdot B(n)+\varepsilon}\underset{n_{k}\neq 1}{\prod_{k=1}^{r}}\frac{\text{\rm exp}\left(\frac{|\alpha^{(k)}|^{2}}{T^{2}}\right)\left|\mathcal{F}_{R}^{(n_{k})}\left(\tfrac{\alpha^{(k)}}{2}\right)\right|^{2}\prod\limits_{1\leq\,i<j\,\leq n_{k}}\frac{\left|\Gamma\left(\textstyle{\frac{2R+1+\alpha_{k,i}-\alpha_{k,j}}{4}}\right)\right|^{4}}{\left|\Gamma\left(\frac{1+\alpha_{k,i}-\alpha_{k,j}}{2}\right)\right|^{2}}}{\left|L(1,\,\text{\rm Ad}\;\phi_{k})\right|}
n1s1++nrsr=0Re(sj)=0,|Im(sj)|T1k<kr1|L(1+sksk,ϕk×ϕk)|2i=1nkj=1nk|Γ(2R+1+sksk+αk,iαk,j4)|4|Γ(1+sksk+αk,iαk,j2)|2|ds|.\displaystyle\hskip-10.0pt\cdot\hskip-16.0pt\underset{\text{\rm Re}(s_{j})=0,\;|\text{\rm Im}(s_{j})|\ll T}{\int\limits_{n_{1}s_{1}+\cdots+n_{r}s_{r}=0}}\prod_{1\leq k<k^{\prime}\leq r}\frac{1}{\left|L\big{(}1+s_{k}-s_{k^{\prime}},\;\phi_{k}\times\phi_{k^{\prime}}\big{)}\right|^{2}}\;\prod_{i=1}^{n_{k}}\;\prod_{j=1}^{n_{k^{\prime}}}\frac{\left|\Gamma\left(\textstyle{\frac{2R+1+s_{k}-s_{k^{\prime}}+\alpha_{k,i}-\alpha_{k^{\prime},j}}{4}}\right)\right|^{4}}{\left|\Gamma\left(\textstyle{\frac{1+s_{k}-s_{k^{\prime}}+\alpha_{k,i}-\alpha_{k^{\prime},j}}{2}}\right)\right|^{2}}\;|ds|.
(m)121n2+1+εTRB(n)+εk=1rnk1|hT,R(nk)(α(k))||L(1,Adϕk)|\displaystyle\ll\;(m\ell)^{\frac{1}{2}-\frac{1}{n^{2}+1}+\varepsilon}\cdot T^{R\cdot B(n)+\varepsilon}\;\;\underset{n_{k}\neq 1}{\prod_{k=1}^{r}}\;\;\frac{\left|h_{T,R}^{(n_{k})}(\alpha^{(k)})\right|}{\left|L(1,\,\text{\rm Ad}\;\phi_{k})\right|}
n1s1++nrsr=0Re(sj)=0,|Im(sj)|T1k<kr1|L(1+sksk,ϕk×ϕk)|2i=1nkj=1nk|Γ(2R+1+sksk+αk,iαk,j4)|4|Γ(1+sksk+αk,iαk,j2)|2|ds|.\displaystyle\hskip-20.0pt\cdot\hskip-10.0pt\underset{\text{\rm Re}(s_{j})=0,\;|\text{\rm Im}(s_{j})|\ll T}{\int\limits_{n_{1}s_{1}+\cdots+n_{r}s_{r}=0}}\prod_{1\leq k<k^{\prime}\leq r}\frac{1}{\left|L\big{(}1+s_{k}-s_{k^{\prime}},\;\phi_{k}\times\phi_{k^{\prime}}\big{)}\right|^{2}}\;\,\prod_{i=1}^{n_{k}}\;\prod_{j=1}^{n_{k^{\prime}}}\frac{\left|\Gamma\left(\textstyle{\frac{2R+1+s_{k}-s_{k^{\prime}}+\alpha_{k,i}-\alpha_{k^{\prime},j}}{4}}\right)\right|^{4}}{\left|\Gamma\left(\textstyle{\frac{1+s_{k}-s_{k^{\prime}}+\alpha_{k,i}-\alpha_{k^{\prime},j}}{2}}\right)\right|^{2}}\;|ds|.

where

hT,R(nk)(α(k))=exp(|α(k)|2T2)R(nk)(α(k)2)21ijnkΓ(2R+1+αk,iαk,j4)2Γ(1+αk,iαk,j2).h_{T,R}^{(n_{k})}\left(\alpha^{(k)}\right)=\text{\rm exp}\left(\frac{|\alpha^{(k)}|^{2}}{T^{2}}\right)\mathcal{F}_{R}^{(n_{k})}\big{(}\tfrac{\alpha^{(k)}}{2}\big{)}^{2}\prod_{1\leq\,i\neq j\,\leq n_{k}}\frac{\Gamma\left(\textstyle{\frac{2R+1+\alpha_{k,i}-\alpha_{k,j}}{4}}\right)^{2}}{\Gamma\left(\frac{1+\alpha_{k,i}-\alpha_{k,j}}{2}\right)}.

Next

1k<kri=1nkj=1nk|Γ(2R+1+sksk+αk,iαk,j4)|4|Γ(1+sksk+αk,iαk,j2)|2T(2R1)1k<krnknk.\prod_{1\leq k<k^{\prime}\leq r}\;\prod_{i=1}^{n_{k}}\;\prod_{j=1}^{n_{k^{\prime}}}\frac{\left|\Gamma\left(\textstyle{\frac{2R+1+s_{k}-s_{k^{\prime}}+\alpha_{k,i}-\alpha_{k^{\prime},j}}{4}}\right)\right|^{4}}{\left|\Gamma\left(\textstyle{\frac{1+s_{k}-s_{k^{\prime}}+\alpha_{k,i}-\alpha_{k^{\prime},j}}{2}}\right)\right|^{2}}\;\ll\;T^{(2R-1)\hskip-5.0pt\sum\limits_{1\leq k<k^{\prime}\leq r}n_{k}\cdot n_{k^{\prime}}}.

We obtain the bound

|𝒫,Φ|\displaystyle\left|\mathcal{E}_{\mathcal{P},\Phi}\right| (m)121n2+1+εTRB(n)+ε+(2R1)1k<krnknkk=1r|hT,R(nk)(α(k))||L(1,Adϕk)|\displaystyle\ll(m\ell)^{\frac{1}{2}-\frac{1}{n^{2}+1}+\varepsilon}\cdot T^{R\cdot B(n)+\varepsilon+(2R-1)\;\cdot\hskip-5.0pt\sum\limits_{1\leq k<k^{\prime}\leq r}\hskip-5.0ptn_{k}\cdot n_{k^{\prime}}}\;\cdot\prod_{k=1}^{r}\frac{\left|h_{T,R}^{(n_{k})}\left(\alpha^{(k)}\right)\right|}{\left|L(1,\,\text{\rm Ad}\;\phi_{k})\right|}
n1s1++nrsr=0Re(sj)=0,|Im(sj)|T1k<kr|ds||L(1+sksk,ϕk×ϕk)|2.\displaystyle\hskip 130.0pt\cdot\hskip-16.0pt\underset{\text{\rm Re}(s_{j})=0,\;|\text{\rm Im}(s_{j})|\ll T}{\int\limits_{n_{1}s_{1}+\cdots+n_{r}s_{r}=0}}\prod_{1\leq k<k^{\prime}\leq r}\frac{|ds|}{\left|L\big{(}1+s_{k}-s_{k^{\prime}},\;\phi_{k}\times\phi_{k^{\prime}}\big{)}\right|^{2}}.

Next, we bound the ss-integral above. It follows from Langlands conjecture (see 1.2) that for |Im(sk)|,|Im(sk)|T|\text{\rm Im}(s_{k})|,|\text{\rm Im}(s_{k^{\prime}})|\ll T we have the bound

|L(1+sksk,ϕk×ϕk)|2Tε.\left|L\big{(}1+s_{k}-s_{k^{\prime}},\;\phi_{k}\times\phi_{k^{\prime}}\big{)}\right|^{-2}\ll T^{\varepsilon}.

This together with the bound

n1s1++nrsr=0Re(sj)=0,|Im(sj)|T|ds|Tr1,\int\limits_{\begin{subarray}{c}n_{1}s_{1}+\cdots+n_{r}s_{r}=0\\ \operatorname{Re}(s_{j})=0,\ \lvert\operatorname{Im}(s_{j})\rvert\ll T\end{subarray}}\hskip-14.0pt|ds|\ \ll\ T^{r-1},

implies that

(7.2) |𝒫,Φ|(m)121n2+1+εTRB(n)+(2R1)1k<krnknk+(r1)+ε(k=1r|hT,R(nk)(α(k))||L(1,Adϕk)|).\left|\mathcal{E}_{\mathcal{P},\Phi}\right|\ll(m\ell)^{\frac{1}{2}-\frac{1}{n^{2}+1}+\varepsilon}\,T^{R\cdot B(n)\,+\,(2R-1)\hskip-8.0pt\sum\limits_{1\leq k<k^{\prime}\leq r}\hskip-5.0ptn_{k}\cdot n_{k^{\prime}}\;+(r-1)\,+\,\varepsilon}\;\cdot\left(\prod_{k=1}^{r}\frac{\left|h_{T,R}^{(n_{k})}\left(\alpha^{(k)}\right)\right|}{\left|L(1,\,\text{\rm Ad}\;\phi_{k})\right|}\right).

Since each nk<nn_{k}<n (for k=1,2,,r)k=1,2,\ldots,r), we can apply our inductive procedure together with the following theorem to bound Φ|𝒫,Φ|.\sum_{\Phi}\left|\mathcal{E}_{\mathcal{P},\Phi}\right|.

Theorem 7.3 (Weyl law with harmonic weights for 𝐆𝐋(nk){\operatorname{GL}(n_{k})} with nk<n{n_{k}<n}).

Suppose nkn_{k}\in\mathbb{Z} with 2nk<n2\leq n_{k}<n. Let {ϕ1,ϕ2,}\{\phi_{1},\phi_{2},\ldots\} be an orthogonal basis of Hecke-Maass cusp forms for GL(nk)\operatorname{GL}(n_{k}) ordered by eigenvalue. If α(j)\alpha^{(j)} are the Langlands parameters of ϕj\phi_{j}, then

(7.4) j=1hT,R(nk)(α(j)¯)jnT2R(D(k)+nk(nk1)2)+nk1.\sum_{j=1}^{\infty}\frac{h_{T,R}^{(n_{k})}\left(\overline{\alpha^{(j)}}\right)}{\mathcal{L}_{j}}\ll_{n}\;T^{2R\cdot\big{(}D(k)+\frac{n_{k}(n_{k}-1)}{2}\big{)}+n_{k}-1}.
Proof.

In [GSW21], all that was needed to prove this statement for n=4n=4 was to have it be true for nk=2n_{k}=2 and nk=3n_{k}=3, which was already known. A similar induction argument works in general. ∎

It immediately follows from the bound (7.2) and (7.4) that

Φ|𝒫,Φ|(m)121n2+1+εTRB(n)+(2R1)1k<krnknk+(r1)+εTk=1r(2R(D(k)+nk(nk1)2)+nk1).\sum_{\Phi}\left|\mathcal{E}_{\mathcal{P},\Phi}\right|\ll(m\ell)^{\frac{1}{2}-\frac{1}{n^{2}+1}+\varepsilon}\,T^{R\cdot B(n)\,+\,(2R-1)\hskip-8.0pt\sum\limits_{1\leq k<k^{\prime}\leq r}\hskip-5.0ptn_{k}\cdot n_{k^{\prime}}\;+(r-1)\,+\,\varepsilon}\hskip-5.0pt\cdot\;T^{\sum\limits_{k=1}^{r}\left(2R\cdot\big{(}D(k)+\frac{n_{k}(n_{k}-1)}{2}\big{)}+n_{k}-1\right)}.

Recall that B(n)=2D(n)2k=1rD(nk),B(n)=2D(n)-2\sum\limits_{k=1}^{r}D(n_{k}), which implies that

Φ|𝒫,Φ|(m)121n2+1+εT2RD(n)+ 2R(1k<krnknk+k=1rnk(nk1)2)+k=1rnk1k<krnknk1+ε.\sum_{\Phi}\left|\mathcal{E}_{\mathcal{P},\Phi}\right|\ll(m\ell)^{\frac{1}{2}-\frac{1}{n^{2}+1}+\varepsilon}\,T^{2R\cdot D(n)\,+\,2R\left(\sum\limits_{1\leq k<k^{\prime}\leq r}\hskip-5.0ptn_{k}\cdot n_{k^{\prime}}+\sum\limits_{k=1}^{r}\frac{n_{k}(n_{k}-1)}{2}\right)+\sum\limits_{k=1}^{r}n_{k}\;\,-\hskip-4.0pt\sum\limits_{1\leq k<k^{\prime}\leq r}\hskip-5.0ptn_{k}\cdot n_{k^{\prime}}\;-1\,+\,\varepsilon}.

Next, 1k<krnknk+k=1rnk(nk1)2=n(n1)2\sum\limits_{1\leq k<k^{\prime}\leq r}\hskip-5.0ptn_{k}\cdot n_{k^{\prime}}\;+\sum\limits_{k=1}^{r}\frac{n_{k}(n_{k}-1)}{2}=\frac{n(n-1)}{2} by Lemma A.22 and k=1rnk=n.\sum\limits_{k=1}^{r}n_{k}=n. It follows that

Φ|𝒫,Φ|(m)121n2+1+εT2R(D(n)+n(n1)2)+n11k<krnknk+ε.\sum_{\Phi}\left|\mathcal{E}_{\mathcal{P},\Phi}\right|\ll(m\ell)^{\frac{1}{2}-\frac{1}{n^{2}+1}+\varepsilon}\,T^{2R\cdot\left(D(n)\,+\,\frac{n(n-1)}{2}\right)+n-1\;-\hskip-5.0pt\sum\limits_{1\leq k<k^{\prime}\leq r}\hskip-5.0ptn_{k}\cdot n_{k^{\prime}}\,+\,\varepsilon}.

To complete the proof, we need to sum over all parabolics 𝒫\mathcal{P}. It suffices, therefore, to consider the “worst case scenario” among the possible partitions n=n1++nrn=n_{1}+\cdots+n_{r} for which the expression

1k<krnknk\sum\limits_{1\leq k<k^{\prime}\leq r}n_{k}n_{k^{\prime}}

is minimized. It is easy to see that this occurs when r=2r=2 and {n1,n2}={n1,1}\{n_{1},n_{2}\}=\{n-1,1\}, giving the bound n1n-1. It follows that

𝒫Φ|𝒫,Φ|(m)121n2+1+εT2R(D(n)+n(n1)2)+ε.\sum_{\mathcal{P}}\sum_{\Phi}\left|\mathcal{E}_{\mathcal{P},\Phi}\right|\ll(m\ell)^{\frac{1}{2}-\frac{1}{n^{2}+1}+\varepsilon}\,T^{2R\cdot\left(D(n)\,+\,\frac{n(n-1)}{2}\right)\,+\,\varepsilon}.

Using (1.2), this immediately implies the desired result. ∎

Remark 7.5.

In [JN19] Jana and Nelson prove the bound

(7.6) c(ϕj)Tn1jTn2n,\sum_{c\left(\phi_{j}\right)\leq T^{n}}\frac{1}{\mathcal{L}_{j}}\;\ll\;T^{n^{2}-n},

where c(ϕ)c(\phi) is the analytic conductor given in (1.1). This is an unsmoothed version of Theorem 7.3. Our result is a smoothed version, and it doesn’t seem possible to derive a bound as in (7.6) with a sharp cutoff without using a different approach.

8. An integral representation of pT,R(n)(y)p_{T,R}^{(n)}(y)

Recall (see (1.4)) that

pT,Rn,#(α):=eα12+α22++αn22T2R(n)(α2)1jknΓ(1+2R+αjαk4).p_{T,R}^{n,\#}(\alpha)\;:=\;e^{\frac{\alpha_{1}^{2}+\alpha_{2}^{2}+\cdots+\alpha_{n}^{2}}{2T^{2}}}\cdot\mathcal{F}_{R}^{(n)}(\tfrac{\alpha}{2})\prod_{1\leq\,j\neq k\,\leq n}\Gamma\left(\textstyle{\frac{1+2R+\alpha_{j}-\alpha_{k}}{4}}\right).

Using the formula for the inverse Lebedev-Whittaker transform given in [GK12], it follows that

pT,R(n)(y)\displaystyle p_{T,R}^{(n)}(y) :=1πn1α^n=0Re(α)=0pT,Rn,#(α)Wn,α(y)¯1jknΓ(αjαk2)𝑑α\displaystyle:=\frac{1}{\pi^{n-1}}\int\limits_{\begin{subarray}{c}\widehat{\alpha}_{n}=0\\ \operatorname{Re}(\alpha)=0\end{subarray}}\frac{p^{n,\#}_{T,R}(\alpha)\;\overline{W_{n,\alpha}(y)}}{\prod\limits_{1\leq\,j\neq k\,\leq n}\Gamma\left(\frac{\alpha_{j}-\alpha_{k}}{2}\right)}\;d\alpha
=1πn1α^n=0Re(α)=0eα12+α22++αn22T2R(n)(α2)1jknΓR(αjαk2)Wn,α(y)¯dα,\displaystyle=\frac{1}{\pi^{n-1}}\int\limits_{\begin{subarray}{c}\widehat{\alpha}_{n}=0\\ \operatorname{Re}(\alpha)=0\end{subarray}}e^{\frac{\alpha_{1}^{2}+\alpha_{2}^{2}+\cdots+\alpha_{n}^{2}}{2T^{2}}}\cdot\mathcal{F}_{R}^{(n)}(\tfrac{\alpha}{2})\prod_{1\leq\,j\neq k\,\leq n}\Gamma_{R}\big{(}\tfrac{\alpha_{j}-\alpha_{k}}{2}\big{)}\;\overline{W_{n,\alpha}(y)}\ d\alpha,

where ΓR(z):=Γ(12+R+z2)Γ(z)\Gamma_{R}(z):=\frac{\Gamma\left(\frac{\frac{1}{2}+R+z}{2}\right)}{\Gamma(z)}.

The strategy in this section for giving a representation of pT,R(n)(y)p_{T,R}^{(n)}(y) follows the same general outline as was used to obtain the results for GL(3)\operatorname{GL}(3) and GL(4)\operatorname{GL}(4) given in the papers [GK13] and [GSW21], respectively. As in the prior works, we express the Whittaker function as the inverse Mellin transform of its Mellin transform. (See Section 8.1.) Plugging this into the above formula gives an integral representation of pT,R(n)(y)p_{T,R}^{(n)}(y) in terms of an additional variable s=(s1,,sn1)s=(s_{1},\ldots,s_{n-1}).

8.1. Normalized Mellin transform of Whittaker function

We introduce (as in [IS07]) the following Mellin transform and its inverse.

Definition 8.1 (Normalized Mellin transform of Whittaker function).

Let n+n\in{\mathbb{Z}}_{+} and α=(α1,,αn)n\alpha=(\alpha_{1},\ldots,\alpha_{n})\in{\mathbb{C}}^{n} such that α^n=0\widehat{\alpha}_{n}=0. Let Wn,α(y)W_{n,\alpha}(y) be the Whittaker function of Definition 2.3. The Mellin transform is

(8.2) W~n,α(s):=2n100Wn,2α(y)j=1n1(πyj)2sjdyjyj1+j(nj)2,\widetilde{W}_{n,\alpha}(s):=2^{n-1}\int_{0}^{\infty}\cdots\int_{0}^{\infty}W_{n,2\alpha}(y)\prod_{j=1}^{n-1}(\pi y_{j})^{2s_{j}}\frac{dy_{j}}{y_{j}^{1+\frac{j(n-j)}{2}}},

and the inverse Mellin transform is given by

(8.3) Wn,α(y)=12n1s=(s1,,sn1)Re(s)=2b(j=1n1yjj(nj)2(πyj)sj)W~n,α2(s2)𝑑s.W_{n,\alpha}(y)=\frac{1}{2^{n-1}}\int\limits_{\begin{subarray}{c}s=(s_{1},\ldots,s_{n-1})\\ \operatorname{Re}(s)=2b\end{subarray}}\bigg{(}\prod_{j=1}^{n-1}y_{j}^{\frac{j(n-j)}{2}}(\pi y_{j})^{-s_{j}}\bigg{)}\widetilde{W}_{n,\frac{\alpha}{2}}(\tfrac{s}{2})\,ds.

As a consequence of this definition, we have

(8.4) pT,R(n)(y)=\displaystyle p_{T,R}^{(n)}(y)= 1(2π)n1α^n=0Re(α)=0eα12++αn2T2/2R(n)(α)(1jknΓR(αjαk))\displaystyle\frac{1}{(2\pi)^{n-1}}\int\limits_{\begin{subarray}{c}\widehat{\alpha}_{n}=0\\ \operatorname{Re}(\alpha)=0\end{subarray}}e^{\frac{\alpha_{1}^{2}+\cdots+\alpha_{n}^{2}}{T^{2}/2}}\cdot\mathcal{F}_{R}^{(n)}(\alpha)\bigg{(}\prod_{1\leq j\neq k\leq n}\hskip-4.0pt\Gamma_{R}\big{(}\alpha_{j}-\alpha_{k}\big{)}\bigg{)}
s=(s1,,sn1)Re(s)=b(j=1n1yjj(nj)2(πyj)2sj)W~n,α(s)dsdα,\displaystyle\hskip 96.0pt\cdot\hskip-8.0pt\int\limits_{\begin{subarray}{c}s=(s_{1},\ldots,s_{n-1})\\ \operatorname{Re}(s)=b\end{subarray}}\hskip-8.0pt\bigg{(}\prod_{j=1}^{n-1}y_{j}^{\frac{j(n-j)}{2}}(\pi y_{j})^{-2s_{j}}\bigg{)}\widetilde{W}_{n,\alpha}(s)\,ds\,d\alpha,

where b=(b1,,bn1)b=(b_{1},\ldots,b_{n-1}) with each bj>0b_{j}>0.

We use the following theorem to make (8.4) explicit and to begin setting up an inductive method to bound pT,R(n)(y)p_{T,R}^{(n)}(y) for all n2n\geq 2.

Theorem 8.5 (Ishii-Stade).

Let m2m\geq 2 and ε>0\varepsilon>0. Fix a Langlands parameter αm\alpha\in{\mathbb{C}}^{m}. Let sm1s\in{\mathbb{C}}^{m-1} with Re(s)>ε\operatorname{Re}(s)>\varepsilon. Then

(8.6) W~m,α(s)=z=(z1,,zm2)Re(z)=ε(j=1m1Γ(sjzj1+(mj)αmm1)Γ(sjzjjαmm1))W~m1,β(z)(2πi)m2𝑑z,\widetilde{W}_{m,\alpha}(s)=\int\limits_{\begin{subarray}{c}z=(z_{1},\ldots,z_{m-2})\\ \operatorname{Re}(z)=\varepsilon\end{subarray}}\left(\prod_{j=1}^{m-1}\Gamma\Big{(}s_{j}-z_{j-1}+\frac{(m-j)\alpha_{m}}{m-1}\Big{)}\Gamma\Big{(}s_{j}-z_{j}-\frac{j\alpha_{m}}{m-1}\Big{)}\right)\cdot\frac{\widetilde{W}_{m-1,\beta}\left(z\right)}{(2\pi i)^{m-2}}\,dz,

where

z0:=0+0αmm1=0,zm1:=αm(m1)αmm1=0,z_{0}:=-0+\frac{0\cdot\alpha_{m}}{m-1}=0,\quad z_{m-1}:=\alpha_{m}-\frac{(m-1)\alpha_{m}}{m-1}=0,

and

β=(β1,,βm1):=(α1+αmm1,,αm1+αmm1).\beta=\big{(}\beta_{1},\ldots,\beta_{m-1}\big{)}:=\left(\alpha_{1}+\frac{\alpha_{m}}{m-1},\ldots,\alpha_{m-1}+\frac{\alpha_{m}}{m-1}\right).

8.2. A shifted pT,R(n)p_{T,R}^{(n)} term and the Ishii-Stade Conjecture

Our goal is to insert (8.6) into (8.4) and then shift the lines of integration in ss to Re(s)=a\operatorname{Re}(s)=-a, to the left of some of the poles of W~n,α(s)\widetilde{W}_{n,\alpha}(s), which (see Theorem 10.1) occur at Re(si)=δ\operatorname{Re}(s_{i})=-\delta for every 1in11\leq i\leq n-1 and δ0\delta\in{\mathbb{Z}}_{\geq 0}. By Cauchy’s residue formula, this allows us to describe pT,R(n)(y)p_{T,R}^{(n)}(y) in terms of a the sum of a shifted pT,R(n)p_{T,R}^{(n)} term and finitely many shifted residue terms.

Definition 8.1 (shifted pT,R(n)p_{T,R}^{(n)} term).

Let n2n\geq 2 be an integer and a=(a1,,an1)n1a=(a_{1},\ldots,a_{n-1})\in\mathbb{R}^{n-1}. The shifted pT,R(n)p_{T,R}^{(n)} term is given by the same formula as (8.4) but with bb replaced by a-a:

(8.2) pT,R(n)(y;a):=\displaystyle p_{T,R}^{(n)}(y;-a):= 1(2π)n1α^n=0Re(α)=0eα12++αn2T2/2R(n)(α)(1jknΓR(αjαk))\displaystyle\frac{1}{(2\pi)^{n-1}}\int\limits_{\begin{subarray}{c}\widehat{\alpha}_{n}=0\\ \operatorname{Re}(\alpha)=0\end{subarray}}e^{\frac{\alpha_{1}^{2}+\cdots+\alpha_{n}^{2}}{T^{2}/2}}\cdot\mathcal{F}_{R}^{(n)}(\alpha)\bigg{(}\prod_{1\leq j\neq k\leq n}\hskip-4.0pt\Gamma_{R}\big{(}\alpha_{j}-\alpha_{k}\big{)}\bigg{)}
s=(s1,,sn1)Re(s)=a(j=1n1yjj(nj)2(πyj)2sj)W~n,α(s)dsdα.\displaystyle\hskip 96.0pt\cdot\hskip-8.0pt\int\limits_{\begin{subarray}{c}s=(s_{1},\ldots,s_{n-1})\\ \operatorname{Re}(s)=-a\end{subarray}}\hskip-8.0pt\bigg{(}\prod_{j=1}^{n-1}y_{j}^{\frac{j(n-j)}{2}}(\pi y_{j})^{-2s_{j}}\bigg{)}\widetilde{W}_{n,\alpha}(s)\,ds\,d\alpha.

One might be tempted to insert (8.6) into (8.2), but this is invalid if n>3n>3, because Theorem 8.5 requires that Re(si)>ε\operatorname{Re}(s_{i})>\varepsilon for each i=1,,n1i=1,\ldots,n-1. To overcome this difficulty, we use shift equations as given in the following conjecture. This allows us to evaluate W~n,α(s)\widetilde{W}_{n,\alpha}(s) for Re(s)<0\operatorname{Re}(s)<0.

Conjecture 8.3 (Ishii-Stade).

Let m,nm,n\in{\mathbb{Z}} with 1mn11\leq m\leq n-1; let δ0\delta\in{\mathbb{Z}}_{\geq 0}. Let (x)n:=Γ(x+n)Γ(x)=x(x+1)(x+n1)(x)_{n}:=\frac{\Gamma(x+n)}{\Gamma(x)}=x(x+1)\cdots(x+n-1). Then there exists a positive integer rr and, for each ii with 1ir1\leq i\leq r, a polynomial Pi(s,α)P_{i}(s,\alpha) and an (n1)(n-1)-tuple Σi(0)n1\Sigma_{i}\in({\mathbb{Z}}_{\geq 0})^{n-1}, such that

(8.4) W~n,α(s)=[1j1<j2<<jmn(sm+αj1+αj2++αjm)δ]1i=1rPi(s,α)W~n,α(s+Σi),\displaystyle\widetilde{W}_{n,\alpha}(s)=\biggl{[}\prod_{1\leq j_{1}<j_{2}<\ldots<j_{m}\leq n}(s_{m}+\alpha_{j_{1}}+\alpha_{j_{2}}+\cdots+\alpha_{j_{m}})_{{}_{\scriptstyle{\delta}}}\biggr{]}^{-1}\sum_{i=1}^{r}P_{i}(s,\alpha)\widetilde{W}_{n,\alpha}(s+\Sigma_{i}),

where the mmth coordinate of each Σi\Sigma_{i} is at least δ\delta. Moreover, for each ii, we have

deg(Pi(s,α))+2|Σi|=δ(nm).\deg(P_{i}(s,\alpha))+2\big{|}\Sigma_{i}\big{|}=\delta\binom{n}{m}.
Proof of conjecture for 2n52\leq n\leq 5.

Note that the case δ=0\delta=0 of the conjecture is trivial. Moreover, for a given mm and nn with 1mn11\leq m\leq n-1, it’s enough to prove the conjecture for δ=1\delta=1. The case δ>1\delta>1 then follows by applying the case δ=1\delta=1 to itself iteratively.

For δ=1\delta=1 and n=2n=2 or n=3n=3, the conjecture follows immediately from the explicit formulae

W~2,α(s)\displaystyle\widetilde{W}_{2,\alpha}(s) =Γ(s+α)Γ(sα);\displaystyle=\Gamma(s+\alpha)\Gamma(s-\alpha);
W~3,α(s)\displaystyle\widetilde{W}_{3,\alpha}(s) =Γ(s1+α1)Γ(s1+α2)Γ(s1+α3)Γ(s2α1)Γ(s2α2)Γ(s2α3)Γ(s1+s2),\displaystyle=\frac{\Gamma(s_{1}+\alpha_{1})\Gamma(s_{1}+\alpha_{2})\Gamma(s_{1}+\alpha_{3})\Gamma(s_{2}-\alpha_{1})\Gamma(s_{2}-\alpha_{2})\Gamma(s_{2}-\alpha_{3})}{\Gamma(s_{1}+s_{2})},

respectively, together with the functional equation Γ(s+1)=sΓ(s).\Gamma(s+1)=s\Gamma(s). The case δ=1\delta=1 and n=4n=4 is a consequence of [ST21, equations (21), (29), and (31)].

We now consider the case δ=1\delta=1 and n=5n=5. Note that it suffices to derive the appropriate recurrence relations for m=1m=1 and m=2m=2 (that is, for the variables s1s_{1} and s2s_{2}); the cases m=3m=3 and m=4m=4 then follow from the invariance of W~5,α(s)\widetilde{W}_{5,\alpha}(s) under the involution

(s1,s2,s3,s4,α1,α2,α3,α4,α5)(s4,s3,s2,s1,α1,α2,α3,α4,α5).(s_{1},s_{2},s_{3},s_{4},\alpha_{1},\alpha_{2},\alpha_{3},\alpha_{4},\alpha_{5})\to(s_{4},s_{3},s_{2},s_{1},-\alpha_{1},-\alpha_{2},-\alpha_{3},-\alpha_{4},-\alpha_{5}).

We follow an approach developed by Taku Ishii (personal correspondence). First, consider the case m=1m=1: we wish to show that

(8.5) [i=15(s1+αi)]W~5,α(s)\displaystyle\biggl{[}\prod_{i=1}^{5}(s_{1}+\alpha_{i})\biggr{]}\widetilde{W}_{5,\alpha}(s)

is equal to a finite sum of terms Pi(s,α)W~n,α(s+Σi),P_{i}(s,\alpha)\widetilde{W}_{n,\alpha}(s+\Sigma_{i}), where the first coordinate of each Σi(0)4\Sigma_{i}\in({\mathbb{Z}}_{\geq 0})^{4} is at least one, and deg(Pi(s,α))+2|Σi|=5\deg(P_{i}(s,\alpha))+2\big{|}\Sigma_{i}\big{|}=5 for each ii. To this end, let

(8.6) σ=(σ1,σ2,σ3,σ4,σ5)=(s1,s1s2,s2s3,s3s4,s4);\sigma=(\sigma_{1},\sigma_{2},\sigma_{3},\sigma_{4},\sigma_{5})=(-s_{1},s_{1}-s_{2},s_{2}-s_{3},s_{3}-s_{4},s_{4});

note that iσi=0\sum_{i}\sigma_{i}=0. Since s1+σ1=0s_{1}+\sigma_{1}=0, we have

(8.7) [i=15(s1+αi)]W~5,α(s)=[i=15(s1+αi)i=15(s1+σi)]W~5,α(s).\biggl{[}\prod_{i=1}^{5}(s_{1}+\alpha_{i})\biggr{]}\widetilde{W}_{5,\alpha}(s)=\biggl{[}\prod_{i=1}^{5}(s_{1}+\alpha_{i})-\prod_{i=1}^{5}(s_{1}+\sigma_{i})\biggr{]}\widetilde{W}_{5,\alpha}(s).

But for indeterminates T,x1,x2,x3,x4,x5T,x_{1},x_{2},x_{3},x_{4},x_{5}, we have

(8.8) i=15(T+xi)=T5+T4P1(x)+T3P2(x)+T2P3(x)+TP4(x)+P5(x),\displaystyle\prod_{i=1}^{5}(T+x_{i})=T^{5}+T^{4}P_{1}(x)+T^{3}P_{2}(x)+T^{2}P_{3}(x)+TP_{4}(x)+P_{5}(x),

where Pk(x)P_{k}(x) is the elementary symmetric polynomial of degree kk in x1,x2,x3,x4,x5x_{1},x_{2},x_{3},x_{4},x_{5}. So by equation (8.7) above, we have

(8.9) [i=15(s1+αi)]W~5,α(s)\displaystyle\biggl{[}\prod_{i=1}^{5}(s_{1}+\alpha_{i})\biggr{]}\widetilde{W}_{5,\alpha}(s) =[s15+s14P1(α)+s13P2(α)+s12P3(α)+sP4(α)+P5(α)]W~5,α(s)\displaystyle=\bigl{[}s_{1}^{5}+s_{1}^{4}P_{1}(\alpha)+s_{1}^{3}P_{2}(\alpha)+s_{1}^{2}P_{3}(\alpha)+sP_{4}(\alpha)+P_{5}(\alpha)\bigr{]}\widetilde{W}_{5,\alpha}(s)
[s15+s14P1(σ)+s13P2(σ)+s12P3(σ)+sP4(σ)+P5(σ)]W~5,α(s)\displaystyle-\bigl{[}s_{1}^{5}+s_{1}^{4}P_{1}(\sigma)+s_{1}^{3}P_{2}(\sigma)+s_{1}^{2}P_{3}(\sigma)+sP_{4}(\sigma)+P_{5}(\sigma)\bigr{]}\widetilde{W}_{5,\alpha}(s)
=[s13{P2(α)P2(σ)}+s12{P3(α)P3(σ)}\displaystyle=\bigl{[}s_{1}^{3}\{P_{2}(\alpha)-P_{2}(\sigma)\}+s_{1}^{2}\{P_{3}(\alpha)-P_{3}(\sigma)\}
+s1{P4(α)P4(σ)}+{P5(α)P5(σ)}]W~5,α(s),\displaystyle\hskip 17.0pt+s_{1}\{P_{4}(\alpha)-P_{4}(\sigma)\}+\{P_{5}(\alpha)-P_{5}(\sigma)\}\bigr{]}\widetilde{W}_{5,\alpha}(s),

since P1(α)=P1(σ)=0P_{1}(\alpha)=P_{1}(\sigma)=0.

Now let eke_{k}, for 1k41\leq k\leq 4, be the four-tuple with a one in the kkth place and zeroes elsewhere. By [IO14, Proposition 3.6], we have

Pk(α)Pk(σ)=ZkCkP_{k}(\alpha)-P_{k}(\sigma)=Z_{k}-C_{k}

(as operators acting on functions in the variable s=(s1,s2,s3,s4)s=(s_{1},s_{2},s_{3},s_{4})), where the “Capelli elements” CkC_{k} annihilate W~5,α(s)\widetilde{W}_{5,\alpha}(s), and

Z2f(s)\displaystyle Z_{2}f(s) =f(s+e1)+f(s+e2)+f(s+e3)+f(s+e4);\displaystyle=f(s+e_{1})+f(s+e_{2})+f(s+e_{3})+f(s+e_{4});
Z3f(s)\displaystyle Z_{3}f(s) =P1(σ3,σ4,σ5)f(s+e1)+P1(σ1,σ4,σ5)f(s+e2)+P1(σ1,σ2,σ5)f(s+e3)\displaystyle=P_{1}(\sigma_{3},\sigma_{4},\sigma_{5})f(s+e_{1})+P_{1}(\sigma_{1},\sigma_{4},\sigma_{5})f(s+e_{2})+P_{1}(\sigma_{1},\sigma_{2},\sigma_{5})f(s+e_{3})
+P1(σ1,σ2,σ3)f(s+e4);\displaystyle+P_{1}(\sigma_{1},\sigma_{2},\sigma_{3})f(s+e_{4});
Z4f(s)\displaystyle Z_{4}f(s) =P2(σ3,σ4,σ5)f(s+e1)+P2(σ1,σ4,σ5)f(s+e2)+P2(σ1,σ2,σ5)f(s+e3)\displaystyle=P_{2}(\sigma_{3},\sigma_{4},\sigma_{5})f(s+e_{1})+P_{2}(\sigma_{1},\sigma_{4},\sigma_{5})f(s+e_{2})+P_{2}(\sigma_{1},\sigma_{2},\sigma_{5})f(s+e_{3})
+P2(σ1,σ2,σ3)f(s+e4)+f(s+e1+e3)+f(s+e1+e4)+f(s+e2+e4)\displaystyle+P_{2}(\sigma_{1},\sigma_{2},\sigma_{3})f(s+e_{4})+f(s+e_{1}+e_{3})+f(s+e_{1}+e_{4})+f(s+e_{2}+e_{4})
Z5f(s)\displaystyle Z_{5}f(s) =P3(σ3,σ4,σ5)f(s+e1)+P3(σ1,σ4,σ5)f(s+e2)+P3(σ1,σ2,σ5)f(s+e3)\displaystyle=P_{3}(\sigma_{3},\sigma_{4},\sigma_{5})f(s+e_{1})+P_{3}(\sigma_{1},\sigma_{4},\sigma_{5})f(s+e_{2})+P_{3}(\sigma_{1},\sigma_{2},\sigma_{5})f(s+e_{3})
+P3(σ1,σ2,σ3)f(s+e4)+P1(σ5)f(s+e1+e3)+P1(σ3)f(s+e1+e4)\displaystyle+P_{3}(\sigma_{1},\sigma_{2},\sigma_{3})f(s+e_{4})+P_{1}(\sigma_{5})f(s+e_{1}+e_{3})+P_{1}(\sigma_{3})f(s+e_{1}+e_{4})
+P1(σ1)f(s+e2+e4);\displaystyle+P_{1}(\sigma_{1})f(s+e_{2}+e_{4});
.

So by (8.9),

(8.10) [i=15(s1+αi)]W~5,α(s)\displaystyle\biggl{[}\prod_{i=1}^{5}(s_{1}+\alpha_{i})\biggr{]}\widetilde{W}_{5,\alpha}(s) =[s13Z2+s12Z3+s1Z4+Z5]W~5,α(s)\displaystyle=\bigl{[}s_{1}^{3}Z_{2}+s_{1}^{2}Z_{3}+s_{1}Z_{4}+Z_{5}\bigr{]}\widetilde{W}_{5,\alpha}(s)
=[s13+s12P1(σ3,σ4,σ5)+s1P2(σ3,σ4,σ5)+P3(σ3,σ4,σ5)]W~5,α(s+e1)\displaystyle=\bigl{[}s_{1}^{3}+s_{1}^{2}P_{1}(\sigma_{3},\sigma_{4},\sigma_{5})+s_{1}P_{2}(\sigma_{3},\sigma_{4},\sigma_{5})+P_{3}(\sigma_{3},\sigma_{4},\sigma_{5})\bigr{]}\widetilde{W}_{5,\alpha}(s+e_{1})
+[s13+s12P1(σ1,σ4,σ5)+s1P2(σ1,σ4,σ5)+P3(σ1,σ4,σ5)]W~5,α(s+e2)\displaystyle+\bigl{[}s_{1}^{3}+s_{1}^{2}P_{1}(\sigma_{1},\sigma_{4},\sigma_{5})+s_{1}P_{2}(\sigma_{1},\sigma_{4},\sigma_{5})+P_{3}(\sigma_{1},\sigma_{4},\sigma_{5})\bigr{]}\widetilde{W}_{5,\alpha}(s+e_{2})
+[s13+s12P1(σ1,σ2,σ5)+s1P2(σ1,σ2,σ5)+P3(σ1,σ2,σ5)]W~5,α(s+e3)\displaystyle+\bigl{[}s_{1}^{3}+s_{1}^{2}P_{1}(\sigma_{1},\sigma_{2},\sigma_{5})+s_{1}P_{2}(\sigma_{1},\sigma_{2},\sigma_{5})+P_{3}(\sigma_{1},\sigma_{2},\sigma_{5})\bigr{]}\widetilde{W}_{5,\alpha}(s+e_{3})
+[s13+s12P1(σ1,σ2,σ3)+s1P2(σ1,σ2,σ3)+P3(σ1,σ2,σ3)]W~5,α(s+e4)\displaystyle+\bigl{[}s_{1}^{3}+s_{1}^{2}P_{1}(\sigma_{1},\sigma_{2},\sigma_{3})+s_{1}P_{2}(\sigma_{1},\sigma_{2},\sigma_{3})+P_{3}(\sigma_{1},\sigma_{2},\sigma_{3})\bigr{]}\widetilde{W}_{5,\alpha}(s+e_{4})
+[s1+P1(σ5)]W~5,α(s+e1+e3)\displaystyle+\bigl{[}s_{1}+P_{1}(\sigma_{5})\bigr{]}\widetilde{W}_{5,\alpha}(s+e_{1}+e_{3})
+[s1+P1(σ3)]W~5,α(s+e1+e4)\displaystyle+\bigl{[}s_{1}+P_{1}(\sigma_{3})\bigr{]}\widetilde{W}_{5,\alpha}(s+e_{1}+e_{4})
+[s1+P1(σ1)]W~5,α(s+e2+e4).\displaystyle+\bigl{[}s_{1}+P_{1}(\sigma_{1})\bigr{]}\widetilde{W}_{5,\alpha}(s+e_{2}+e_{4}).

Recalling that the PkP_{k}’s are the elementary symmetric polynomials of degree kk in their arguments, we see that

s13+s12P1(a,b,c)+s1P2(a,b,c)+P3(a,b,c)=(s1+a)(s1+b)(s1+c),s_{1}^{3}+s_{1}^{2}P_{1}(a,b,c)+s_{1}P_{2}(a,b,c)+P_{3}(a,b,c)=(s_{1}+a)(s_{1}+b)(s_{1}+c),

for indeterminates a,b,ca,b,c. So (8.10) gives

[i=15(s1+αi)]W~5,α(s)\displaystyle\biggl{[}\prod_{i=1}^{5}(s_{1}+\alpha_{i})\biggr{]}\widetilde{W}_{5,\alpha}(s)
=(s1+σ3)(s1+σ4)(s1+σ5)W~5,α(s+e1)+(s1+σ1)(s1+σ4)(s1+σ5)W~5,α(s+e2)\displaystyle=(s_{1}+\sigma_{3})(s_{1}+\sigma_{4})(s_{1}+\sigma_{5})\widetilde{W}_{5,\alpha}(s+e_{1})+(s_{1}+\sigma_{1})(s_{1}+\sigma_{4})(s_{1}+\sigma_{5})\widetilde{W}_{5,\alpha}(s+e_{2})
+(s1+σ1)(s1+σ2)(s1+σ5)W~5,α(s+e3)\displaystyle+(s_{1}+\sigma_{1})(s_{1}+\sigma_{2})(s_{1}+\sigma_{5})\widetilde{W}_{5,\alpha}(s+e_{3})
+(s1+σ1)(s1+σ2)(s1+σ3)W~5,α(s+e4)\displaystyle+(s_{1}+\sigma_{1})(s_{1}+\sigma_{2})(s_{1}+\sigma_{3})\widetilde{W}_{5,\alpha}(s+e_{4})
+(s1+σ5)W~4,α(s+e1+e3)+(s1+σ3)W~5,α(s+e1+e4)+(s1+σ1)W~5,α(s+e2+e4)\displaystyle+(s_{1}+\sigma_{5})\widetilde{W}_{4,\alpha}(s+e_{1}+e_{3})+(s_{1}+\sigma_{3})\widetilde{W}_{5,\alpha}(s+e_{1}+e_{4})+(s_{1}+\sigma_{1})\widetilde{W}_{5,\alpha}(s+e_{2}+e_{4})
=(s1+s2s3)(s1+s3s4)(s1+s4)W~5,α(s+e1)\displaystyle=(s_{1}+s_{2}-s_{3})(s_{1}+s_{3}-s_{4})(s_{1}+s_{4})\widetilde{W}_{5,\alpha}(s+e_{1})
+(s1+s4)W~5,α(s+e1+e3)+(s1+s2s3)W~5,α(s+e1+e4),\displaystyle+(s_{1}+s_{4})\widetilde{W}_{5,\alpha}(s+e_{1}+e_{3})+(s_{1}+s_{2}-s_{3})\widetilde{W}_{5,\alpha}(s+e_{1}+e_{4}),

the last step by the definition (8.6) of the σi\sigma_{i}’s. This is our desired shift equation in s1s_{1}.

The shift equation in s2s_{2} is derived analogously. A fundamental difference in this derivation is that, in place of (8.8), we use the following expression involving Schur polynomials sμs_{\mu} (see [Mac79, §I.3], especially Exercise 10 of that section):

1i<j5(T+xi+xj)=μ=(μ1,μ2,,μ5)S(T2)10(μ1+μ2++μ5)dμsμ(x1,x2,,x5).\displaystyle\prod_{1\leq i<j\leq 5}(T+x_{i}+x_{j})=\sum_{\mu=(\mu_{1},\mu_{2},\ldots,\mu_{5})\in S}\biggl{(}\frac{T}{2}\biggr{)}^{10-(\mu_{1}+\mu_{2}+\cdots+\mu_{5})}d_{\mu}\,s_{\mu}(x_{1},x_{2},\ldots,x_{5}).

Here,

S={(μ1,μ2,,μ5)(0)5:μi5i(1i5) and μ1μ2μ5},S=\bigl{\{}(\mu_{1},\mu_{2},\ldots,\mu_{5})\in({\mathbb{Z}}_{\geq 0})^{5}:\mu_{i}\leq 5-i\ (1\leq i\leq 5)\hbox{ and }\mu_{1}\geq\mu_{2}\geq\cdots\geq\mu_{5}\bigr{\}},

and dμd_{\mu} is the determinant of the matrix

((2(5i)μj+5j))1i,j5.\biggl{(}\binom{2(5-i)}{\mu_{j}+5-j}\biggr{)}_{1\leq i,j\leq 5}.

The Schur polynomials are symmetric polynomials in the xkx_{k}’s, and are therefore expressible in terms of the elementary symmetric polynomials in the xkx_{k}’s. Techniques like those employed above, in the case m=1m=1, therefore apply. We omit the details. ∎

Remark 8.11.

The above proof, in the case m=1m=1 (that is, for the variable s1s_{1}—and therefore also for the variable sn1s_{n-1}), generalizes to the case of GL(n,)\operatorname{GL}(n,\mathbb{R}), for any n2n\geq 2. For 2mn22\leq m\leq n-2, we do not yet have a proof that works for all n2n\geq 2, though we expect that the above ideas and techniques should prove relevant. Indeed, using the above methods, and applying Mathematica to help with the more arduous calculations, we have been able to verify Conjecture 8.3 in full generality for n7n\leq 7.

We further note that, alternatively, one might continue W~n,α(s)\widetilde{W}_{n,\alpha}(s) in the sjs_{j}’s by shifting or deforming the lines of integration in (8.6). Unfortunately such an approach has, thus far, failed to yield suitable results. In particular, the residues that one obtains in moving these lines of integration past poles of the integrand are quite complicated, and do not seem to lend themselves to bounds of the type required to estimate pT,R(n)(y)p_{T,R}^{(n)}(y).

8.3. pT,R(n)(y){p_{T,R}^{(n)}(y)} is a sum of a shifted term and residues

Besides the shifted pT,R(n)p_{T,R}^{(n)} term (because we cross poles of W~n,α(s)\widetilde{W}_{n,\alpha}(s) upon shifting the lines of integration) there are also many residue terms. The residue terms will be parameterized by compositions of nn. Recall that a composition of length rr of a positive integer nn is a way of writing n=n1++nrn=n_{1}+\cdots+n_{r} as a sum of strictly positive integers. Two sums that differ in the order define different compositions. Compare this, on the other hand with partitions which are compositions of nn for which the order doesn’t matter.

Definition 8.1.

(a\boldsymbol{a}-admissible compositions) Let a=(a1,,an1)n1a=(a_{1},\ldots,a_{n-1})\in\mathbb{R}^{n-1}. A composition n=n1++nrn=n_{1}+\cdots+n_{r} is termed aa-admissible if

an^i>0 for all i=1,,r1.a_{\widehat{n}_{i}}>0\mbox{ for all }i=1,\ldots,r-1.

The set of aa-admissible compositions of length greater than one is

𝒞a:={compositionsn=n1++nr|2rnan^i>0 for all i=1,,r1}.\mathcal{C}_{a}:=\left\{\left.\begin{array}[]{c}\mbox{compositions}\\ n=n_{1}+\cdots+n_{r}\end{array}\ \right|\begin{array}[]{c}2\leq r\leq n\\ a_{\widehat{n}_{i}}>0\mbox{ for all }i=1,\ldots,r-1\end{array}\right\}.
Remark 8.2.

At times we may also notate a composition n=n1++nrn=n_{1}+\cdots+n_{r} as an ordered list C=(n1,,nr)C=(n_{1},\ldots,n_{r}).

Definition 8.3 ((𝒓𝟏)\boldsymbol{(r-1)}-fold residue term).

Suppose that r2r\geq 2 and C𝒞aC\in\mathcal{C}_{a} is given by n=n1++nrn=n_{1}+\cdots+n_{r}. Let

δC:=(δ1,δ2,,δr1)(0)r1\delta_{C}:=(\delta_{1},\delta_{2},\ldots,\delta_{r-1})\in\big{(}{\mathbb{Z}}_{\geq 0}\big{)}^{r-1}

with 0δian^i0\leq\delta_{i}\leq\lfloor a_{\widehat{n}_{i}}\rfloor for each i=1,,r1i=1,\ldots,r-1. If CC has length two, we write δC=δ\delta_{C}=\delta. We define the (r1)(r-1)-fold residue term

(8.4) pT,R(n)(y;a,δC)\displaystyle p_{T,R}^{(n)}(y;-a,\delta_{C}) :=α^n=0Re(α)=0eα12++αn2T2/2R(n)(α)(1jknΓR(αjαk))\displaystyle:=\int\limits_{\begin{subarray}{c}\widehat{\alpha}_{n}=0\\ \operatorname{Re}(\alpha)=0\end{subarray}}e^{\frac{\alpha_{1}^{2}+\cdots+\alpha_{n}^{2}}{T^{2}/2}}\cdot\mathcal{F}_{R}^{(n)}(\alpha)\bigg{(}\prod_{1\leq j\neq k\leq n}\hskip-4.0pt\Gamma_{R}\big{(}\alpha_{j}-\alpha_{k}\big{)}\bigg{)}\hskip-3.0pt
(i=1r1yin^i(nn^i)2+2(α^n^i+δi))Re(sj)=ajj{n^1,,n^r1}(j{n^1,,n^r1}yjj(nj)22sj)\displaystyle\hskip 12.0pt\cdot\bigg{(}\prod_{i=1}^{r-1}y_{i}^{\frac{\widehat{n}_{i}(n-\widehat{n}_{i})}{2}+2(\widehat{\alpha}_{\widehat{n}_{i}}+\delta_{i})}\bigg{)}\cdot\int\limits_{\begin{subarray}{c}\operatorname{Re}(s_{j})=-a_{j}\\ j\notin\{\widehat{n}_{1},\ldots,\widehat{n}_{r-1}\}\end{subarray}}\bigg{(}\prod_{j\notin\{\widehat{n}_{1},\ldots,\widehat{n}_{r-1}\}}y_{j}^{\frac{j(n-j)}{2}-2s_{j}}\bigg{)}
Ressn^1=α^n^1δ1(Ressn^2=α^n^2δ2((Ressn^r1=α^n^r1δr1W~n,α(s))))dsdα.\displaystyle\hskip-24.0pt\cdot\underset{s_{\widehat{n}_{1}}=-\widehat{\alpha}_{\widehat{n}_{1}}-\delta_{1}}{\operatorname{Res}}\left(\underset{s_{\widehat{n}_{2}}=-\widehat{\alpha}_{\widehat{n}_{2}}-\delta_{2}}{\operatorname{Res}}\left(\cdots\left(\underset{s_{\widehat{n}_{r-1}}=-\widehat{\alpha}_{\widehat{n}_{r-1}}-\delta_{r-1}}{\operatorname{Res}}\widetilde{W}_{n,\alpha}(s)\right)\cdots\right)\right)\,ds\,d\alpha.
Remark 8.5.

In the shifted integral (8.4), if ai>0-a_{i}>0 for some ii, there will be no residues coming from the integral in sis_{i} because we are not shifting past any poles. For this reason, one only obtains residue terms pT,R(n)(y;a,δC)p_{T,R}^{(n)}(y;-a,\delta_{C}) in the case that CC is aa-admissible. That said, equation (8.4) makes perfect sense even if CC is not aa-admissible. In this case, pT,R(n)(y;a,δC)p_{T,R}^{(n)}(y;-a,\delta_{C}) is identically zero.

Proposition 8.6.

Suppose that a=(a1,,an1)n1a=(a_{1},\ldots,a_{n-1})\in\mathbb{R}^{n-1}. Then there exists constants κ(C)\kappa(C) such that

pT,R(n)(y)=pT,R(n)(y;a)+C𝒞aκ(C)δC=(δ1,,δr1)0δian^ipT,R(n)(y;a,δC).p_{T,R}^{(n)}(y)=p_{T,R}^{(n)}(y;-a)\ +\ \sum_{C\in\mathcal{C}_{a}}\kappa(C)\sum_{\begin{subarray}{c}\delta_{C}=(\delta_{1},\ldots,\delta_{r-1})\\ 0\leq\delta_{i}\leq\lfloor a_{\widehat{n}_{i}}\rfloor\end{subarray}}p_{T,R}^{(n)}(y;-a,\delta_{C}).

Before giving the proof, we make some preliminary remarks and observations.

Remark 8.7.

Notice that an element σ\sigma of the symmetric group SnS_{n} (i.e., the group of permutations of a set of nn elements) acts on α=(α1,,αn)\alpha=(\alpha_{1},\ldots,\alpha_{n}) and, by extension, on α^k\widehat{\alpha}_{k} via

σα^k:=ασ(1)+ασ(2)++ασ(k).\sigma\cdot\widehat{\alpha}_{k}:=\alpha_{\sigma(1)}+\alpha_{\sigma(2)}+\cdots+\alpha_{\sigma(k)}.

We can consider the analog to (8.4) obtained by replacing each instance of α^m\widehat{\alpha}_{m} with σα^m\sigma\cdot\widehat{\alpha}_{m}:

α^n=0Re(α)=0eα12++αn2T2/2R(n)(α)(1jknΓR(αjαk))(i{n^1,,n^r1}yii(ni)2+2(σα^i+δi))Re(sj)=ajj{n^1,,n^r1}(j{n^1,,n^r1}yjj(nj)22sj)Ressi1=σα^i1δi1(Ressi2=σα^i2δi2Ressik=σα^ikδikW~n,α(s))dsdα\int\limits_{\begin{subarray}{c}\widehat{\alpha}_{n}=0\\ \operatorname{Re}(\alpha)=0\end{subarray}}e^{\frac{\alpha_{1}^{2}+\cdots+\alpha_{n}^{2}}{T^{2}/2}}\cdot\mathcal{F}_{R}^{(n)}(\alpha)\cdot\bigg{(}\prod_{1\leq j\neq k\leq n}\hskip-4.0pt\Gamma_{R}\big{(}\alpha_{j}-\alpha_{k}\big{)}\bigg{)}\hskip-3.0pt\bigg{(}\prod_{i\in\{\widehat{n}_{1},\ldots,\widehat{n}_{r-1}\}}y_{i}^{\frac{i(n-i)}{2}+2(\sigma\cdot\widehat{\alpha}_{i}+\delta_{i})}\bigg{)}\\ \cdot\int\limits_{\begin{subarray}{c}\operatorname{Re}(s_{j})=-a_{j}\\ j\in\{\widehat{n}_{1},\ldots,\widehat{n}_{r-1}\}\end{subarray}}\hskip-5.0pt\bigg{(}\prod_{j\notin\{\widehat{n}_{1},\ldots,\widehat{n}_{r-1}\}}y_{j}^{\frac{j(n-j)}{2}-2s_{j}}\bigg{)}\underset{s_{i_{1}}=-\sigma\cdot\widehat{\alpha}_{i_{1}}-\delta_{i_{1}}}{\operatorname{Res}}\left(\underset{s_{i_{2}}=-\sigma\cdot\widehat{\alpha}_{i_{2}}-\delta_{i_{2}}}{\operatorname{Res}}\cdots\underset{s_{i_{k}}=-\sigma\cdot\widehat{\alpha}_{i_{k}}-\delta_{i_{k}}}{\operatorname{Res}}\widetilde{W}_{n,\alpha}(s)\right)\\ \cdot ds\,d\alpha

We make two observations:

  • As CC varies over all compositions of length rr and σ\sigma varies over all possible permutations and δC\delta_{C} varies over all (0)r1\big{(}{\mathbb{Z}}_{\geq 0}\big{)}^{r-1}, one obtains all possible (r1)(r-1)-fold residues coming from shifting the lines of integration in pT,R(n)(y)p_{T,R}^{(n)}(y). This is a consequence of Theorem 10.1 below.

  • The action of SnS_{n} on ordered subsets of {α1,α2,,αn}\{\alpha_{1},\alpha_{2},\ldots,\alpha_{n}\} given by permuting the indices is trivial on W~n,α(s)\widetilde{W}_{n,\alpha}(s), i.e., W~n,σ(α)(s)=W~n,α(s)\widetilde{W}_{n,\sigma(\alpha)}(s)=\widetilde{W}_{n,\alpha}(s), and on the function

    eα12++αn2T2/2R(n)(α)(1jknΓR(αjαk)).e^{\frac{\alpha_{1}^{2}+\cdots+\alpha_{n}^{2}}{T^{2}/2}}\cdot\mathcal{F}_{R}^{(n)}(\alpha)\cdot\bigg{(}\prod_{1\leq j\neq k\leq n}\hskip-4.0pt\Gamma_{R}\big{(}\alpha_{j}-\alpha_{k}\big{)}\bigg{)}.

    This implies that relabeling the variables α1,α2,αn\alpha_{1},\alpha_{2},\ldots\alpha_{n} by ασ1(1),ασ1(2),,ασ1(n)\alpha_{\sigma^{-1}(1)},\alpha_{\sigma^{-1}(2)},\ldots,\alpha_{\sigma^{-1}(n)} everywhere (1) doesn’t change the value of the integral, and (2) recovers the original integral given in (8.4).

Remark 8.8.

The constant κ(C)\kappa(C) is the size of the (generic) orbit of the action of SnS_{n} on the set

A={α^n^1,,α^n^r1}.A=\{\widehat{\alpha}_{\widehat{n}_{1}},\ldots,\widehat{\alpha}_{\widehat{n}_{r-1}}\}.

Hence, defining the stabilizer of AA to be

Stab(A):={σSnσα^m=α^m for each m=n^1,,n^r1},\operatorname{Stab}(A):=\{\sigma\in S_{n}\mid\sigma\cdot\widehat{\alpha}_{m}=\widehat{\alpha}_{m}\mbox{ for each }m=\widehat{n}_{1},\ldots,\widehat{n}_{r-1}\},

we see that

κ(C)=#Sn#Stab(A)=n!i=1r1(ni!).\kappa(C)=\frac{\#S_{n}}{\#\operatorname{Stab}(A)}=\frac{n!}{\prod\limits_{i=1}^{r-1}(n_{i}!)}.

Since the exact value of κ(C)\kappa(C) is irrelevant to our application, we omit its proof below and leave it instead to the interested reader.

Proof of Proposition 8.6.

Beginning with (8.4), we see that pT,R(n)(y)=pT,R(n)(y;b)p_{T,R}^{(n)}(y)=p_{T,R}^{(n)}(y;b) for any b=(b1,,bn1)b=(b_{1},\ldots,b_{n-1}) with bi>0b_{i}>0 for each i=1,,n1i=1,\ldots,n-1. In order to compare this with pT,R(n)(y;a)p_{T,R}^{(n)}(y;-a), we successively shift the lines of integration in the variables sks_{k} for each kk such that ak<0-a_{k}<0 (in descending order). If ak>0-a_{k}>0 then shifting the line of integration from Re(s)=bk\operatorname{Re}(s)=b_{k} to Re(sk)=ak\operatorname{Re}(s_{k})=-a_{k} doesn’t change the value of the integral in sks_{k}. In other words, there is a residue term if and only if the composition CC is admissible.

Beginning with the fact that

pT,R(n)(y)=pT,R(n)(y;b)for any b=(b1,,bn1) for which bj>0 for all j,p_{T,R}^{(n)}(y)=p_{T,R}^{(n)}(y;b)\quad\mbox{for any $b=(b_{1},\ldots,b_{n-1})$ for which $b_{j}>0$ for all $j$},

we may shift the line of integration in sn1s_{n-1} to Re(sn1)=an1\operatorname{Re}(s_{n-1})=-a_{n-1}. In doing so, provided that an1>0a_{n-1}>0, we pass poles at sn1=σα^1δ1s_{n-1}=-\sigma\cdot\widehat{\alpha}_{1}-\delta_{1} for each 0δa10\leq\delta\leq\lfloor a_{1}\rfloor. Hence, taking into account Remark 8.7, and considering n=(n1)+1n=(n-1)+1 (denoted (n1,1)(n-1,1)), it follows that

(8.9) pT,R(n)(y)\displaystyle p_{T,R}^{(n)}(y) =pT,R(n)(y;(b1,b2,b3,,an1))\displaystyle=p_{T,R}^{(n)}(y;(b_{1},b_{2},b_{3},\ldots,-a_{n-1}))
+κ((n1,1))δ(n1,1)pT,R(n)(y;(b1,b2,,an1),δ(n1,1)),\displaystyle\hskip 36.0pt+\kappa((n-1,1))\cdot\sum_{\delta_{(n-1,1)}}p_{T,R}^{(n)}(y;(b_{1},b_{2},\ldots,-a_{n-1}),\delta_{(n-1,1)}),

where κ((n1,1))\kappa((n-1,1)) is a constant (which can be verified to agree with the description given in Remark 8.8.)

We now shift the line of integration in sn2s_{n-2} to Re(sn2)=an2\operatorname{Re}(s_{n-2})=-a_{n-2}. As before, provided that an2>0a_{n-2}>0, the Cauchy residue theorem and Remark 8.7 give

(8.10) pT,R(n)(y)\displaystyle p_{T,R}^{(n)}(y) =pT,R(n)(y;(b1,,bn3,an2,an1))\displaystyle=p_{T,R}^{(n)}(y;(b_{1},\ldots,b_{n-3},-a_{n-2},-a_{n-1}))
+κ((n2,2))δ(n2,2)pT,R(n)(y;(b1,,bn3,an2,an1),δ(n2,2))\displaystyle\qquad+\kappa((n-2,2))\hskip-6.0pt\sum_{\delta_{(n-2,2)}}p_{T,R}^{(n)}(y;(b_{1},\ldots,b_{n-3},-a_{n-2},-a_{n-1}),\delta_{(n-2,2)})\hskip-6.0pt
+κ((n1,1))δ(n1)pT,R(n)(y;(b1,,bn3,an2,an1),δ(n1,1))\displaystyle\qquad+\kappa((n-1,1))\sum_{\delta_{(n-1)}}p_{T,R}^{(n)}(y;(b_{1},\ldots,b_{n-3},-a_{n-2},-a_{n-1}),\delta_{(n-1,1)})
+κ((n2,1,1))δ(n2,1,1)pT,R(n)(y;(b1,,bn3,an2,an1),δ(n2,1,1)),\displaystyle\qquad+\kappa((n-2,1,1))\sum_{\delta_{(n-2,1,1)}}p_{T,R}^{(n)}(y;(b_{1},\ldots,b_{n-3},-a_{n-2},-a_{n-1}),\delta_{(n-2,1,1)}),

for constants κ(C)\kappa(C) for each of C=(n1,1),(n2,2),(n2,1,1)C=(n-1,1),(n-2,2),(n-2,1,1) as claimed.

We next repeat this process shifting the integrals in sn3s_{n-3} for each of the terms on the right of (8.10), and then again for sn4s_{n-4} and so forth (skipping those sms_{m} for which am<0a_{m}<0) until all of the lines of integration have been moved to Re(sm)=am\operatorname{Re}(s_{m})=-a_{m} for every possible integral. The claimed formula is now evident. ∎

8.4. Example: 𝐆𝐋(𝟒)\operatorname{GL}(4)

We now consider the special case of W~4,α(s)\widetilde{W}_{4,\alpha}(s) where

α=(α1,α2,α3,α4)(i)4,α^4=0.\alpha=(\alpha_{1},\alpha_{2},\alpha_{3},\alpha_{4})\in\big{(}i\mathbb{R}\big{)}^{4},\quad\widehat{\alpha}_{4}=0.

Fix ε>0\varepsilon>0. Recall that pT,R(4)(y)=pT,R(4)(y;(ε,ε,ε))p_{T,R}^{(4)}(y)=p_{T,R}^{(4)}(y;(\varepsilon,\varepsilon,\varepsilon)). If we now shift the lines of integration to Re(s)=(a)\operatorname{Re}(s)=(-a) where a=(a1,a2,a3)3a=(a_{1},a_{2},a_{3})\in\mathbb{R}^{3}, then we get additional residue terms corresponding to each composition 4=n1++nr4=n_{1}+\cdots+n_{r} and each δC(0)r\delta_{C}\in\big{(}{\mathbb{Z}}_{\geq 0}\big{)}^{r} as follows.

In general the composition n=n1++nrn=n_{1}+\cdots+n_{r} (by abuse of notation, we also think of this as a vector (n1,,nr)(n_{1},\ldots,n_{r}) so that n^k=n1++nk\widehat{n}_{k}=n_{1}+\cdots+n_{k}) corresponds to taking an (r1)(r-1)-fold residue in the variables sn^1,sn^2,,sn^r1s_{\widehat{n}_{1}},s_{\widehat{n}_{2}},\ldots,s_{\widehat{n}_{r-1}}. Here is a table of the residues corresponding to the different compositions.

composition Cresidues in s-variablesδC1+3s1=α1δ1(δ1)2+2s2=α1α2δ2(δ2)3+1s3=α1α2α3δ3(δ3)1+1+2s1=α1δ1,s2=α1α2δ2(δ1,δ2)1+2+1s1=α1δ1,s3=α1α2α3δ3(δ1,δ3)2+1+1s2=α1α2δ2,s3=α1α2α3δ3(δ2,δ3)\begin{array}[]{|c|c|c|}\hline\cr\mbox{composition $C$}&\mbox{residues in $s$-variables}&\delta_{C}\\ \hline\cr 1+3&s_{1}=-\alpha_{1}-\delta_{1}&(\delta_{1})\\ 2+2&s_{2}=-\alpha_{1}-\alpha_{2}-\delta_{2}&(\delta_{2})\\ 3+1&s_{3}=-\alpha_{1}-\alpha_{2}-\alpha_{3}-\delta_{3}&(\delta_{3})\\ \hline\cr 1+1+2&s_{1}=-\alpha_{1}-\delta_{1},\ s_{2}=-\alpha_{1}-\alpha_{2}-\delta_{2}&(\delta_{1},\delta_{2})\\ 1+2+1&s_{1}=-\alpha_{1}-\delta_{1},\ s_{3}=-\alpha_{1}-\alpha_{2}-\alpha_{3}-\delta_{3}&(\delta_{1},\delta_{3})\\ 2+1+1&s_{2}=-\alpha_{1}-\alpha_{2}-\delta_{2},\ s_{3}=-\alpha_{1}-\alpha_{2}-\alpha_{3}-\delta_{3}&(\delta_{2},\delta_{3})\\ \hline\cr\end{array}

In each case 0δiai0\leq\delta_{i}\leq\lfloor a_{i}\rfloor. Not included in the table are the triple residues in si=α^iδis_{i}=-\widehat{\alpha}_{i}-\delta_{i} for each i=1,2,3i=1,2,3. These correspond to the composition 4=1+1+1+14=1+1+1+1 and δC=(δ1,δ2,δ3)\delta_{C}=(\delta_{1},\delta_{2},\delta_{3}).

8.5. The integral T,R(m)(a){\mathcal{I}_{T,R}^{(m)}(-a)} in terms of an explicit recursive formula for W~m,α(s)\widetilde{W}_{m,\alpha}(s)

At first glance, the following definition appears to be relevant only for the shifted pT,R(n)p_{T,R}^{(n)}-term, as it is essentially equal to pT,R(n)((1,,1);a)p_{T,R}^{(n)}((1,\ldots,1);-a), and not for the shifted residue terms. However, it will turn out to be pivotal to bounding the residue terms as well.

Definition 8.1 (The integral T,R(m)\mathcal{I}_{T,R}^{(m)}).

Let m2m\geq 2 be an integer and a=(a1,,am1)m1a=(a_{1},\ldots,a_{m-1})\in\mathbb{R}^{m-1}. Then we define

(8.2) T,R(m)(a):=α^m=0Re(α)=0eα12++αm2T2/2R(n)(α)(1jkmΓR(αjαk))s=(s1,,sm1)Re(s)=a|W~m,α(s)|𝑑s𝑑α.\mathcal{I}_{T,R}^{(m)}(-a):=\int\limits_{\begin{subarray}{c}\widehat{\alpha}_{m}=0\\ \operatorname{Re}(\alpha)=0\end{subarray}}e^{\frac{\alpha_{1}^{2}+\cdots+\alpha_{m}^{2}}{T^{2}/2}}\cdot\mathcal{F}_{R}^{(n)}(\alpha)\bigg{(}\prod_{1\leq j\neq k\leq m}\hskip-4.0pt\Gamma_{R}\big{(}\alpha_{j}-\alpha_{k}\big{)}\bigg{)}\int\limits_{\begin{subarray}{c}s=(s_{1},\ldots,s_{m-1})\\ \operatorname{Re}(s)=-a\end{subarray}}\hskip-8.0pt\left|\widetilde{W}_{m,\alpha}(s)\right|\,ds\,d\alpha.

As alluded to above, inserting the result of Theorem 8.5 into (8.2), we find that

|pT,R(n)(y,a)|\displaystyle\big{|}p_{T,R}^{(n)}(y,-a)\big{|} (j=1n1yjj(nj)22aj)T,R(n)(a).\displaystyle\ll\bigg{(}\prod_{j=1}^{n-1}y_{j}^{\frac{j(n-j)}{2}-2a_{j}}\bigg{)}\mathcal{I}_{T,R}^{(n)}(-a).

Hence, giving a bound for pT,R(n)(y)p_{T,R}^{(n)}(y) requires only that we bound T,R(m)(a)\mathcal{I}_{T,R}^{(m)}(-a) in the case of m=nm=n. However, much more is true: we will show that if CC is the composition n=n1++nrn=n_{1}+\cdots+n_{r}, then pT,R(n)(y;a,δC)p_{T,R}^{(n)}(y;-a,\delta_{C}) can be bounded by the same product of yiy_{i}’s as above times a certain power of TT and a product of the form

=1r1T,R(n)(a()),\prod_{\ell=1}^{r-1}\mathcal{I}_{T,R}^{(n_{\ell})}(-a^{(\ell)}),

for certain values a()=(a1(),,an1())a^{(\ell)}=(a_{1}^{(\ell)},\ldots,a_{n_{\ell}-1}^{(\ell)}) which depend on the value of a=(a1,,an1)n1a=(a_{1},\ldots,a_{n-1})\in\mathbb{R}^{n-1}.

The significance of this fact should not be understated. Without it, we would be required to treat nearly every possible composition CC (hence each possible residue term) individually. Indeed, returning to the case of n=4n=4, as noted in Section 8.4 above, there were seven residue terms. The only symmetries that we were able to exploit in [GSW21] to help were that the (1,3)(1,3) and (3,1)(3,1) residues were equivalent, and the (1,1,2)(1,1,2) and (2,1,1)(2,1,1) residues were equivalent as well. This left five individual distinct cases, each of which required several pages of work to bound. So, although the method of this paper does require dealing with some tricky notation and combinatorics, it eliminates the need to treat each residue on its own terms.

9. Bounding T,R(m)\mathcal{I}_{T,R}^{(m)}

Recall that for α=(α1,,αm)m\alpha=(\alpha_{1},\ldots,\alpha_{m})\in{\mathbb{C}}^{m} satisfying α^m=0\widehat{\alpha}_{m}=0 and a=(a1,a2,,an1)n1a=(a_{1},a_{2},\ldots,a_{n-1})\in\mathbb{R}^{n-1},

(9.1) T,R(m)(a):=α^m=0Re(α)=0eα12++αm2T2/2R(m)(α)1jkm|ΓR(αjαk)|Re(s)=a|W~m,α(s)|𝑑s𝑑α.\mathcal{I}_{T,R}^{(m)}(-a):=\int\limits_{\begin{subarray}{c}\widehat{\alpha}_{m}=0\\ \operatorname{Re}(\alpha)=0\end{subarray}}e^{\frac{\alpha_{1}^{2}+\cdots+\alpha_{m}^{2}}{T^{2}/2}}\cdot\mathcal{F}_{R}^{(m)}(\alpha)\hskip-3.0pt\prod_{1\leq j\neq k\leq m}\hskip-4.0pt\big{|}\Gamma_{R}(\alpha_{j}-\alpha_{k})\big{|}\int\limits_{\operatorname{Re}(s)=-a}\hskip-5.0pt\left|\widetilde{W}_{m,\alpha}(s)\right|\,ds\,d\alpha.
Theorem 9.2.

Let T,R(m)(a)\mathcal{I}_{T,R}^{(m)}(-a) be as above and set D(m)=deg(1(m)(α))D(m)=\deg(\mathcal{F}_{1}^{(m)}(\alpha)). Then for any 0<ε<120<\varepsilon<\frac{1}{2},

T,R(m)(a)Tε+(m+4)(m1)4+R(D(m)+m(m1)2)j=1m1B(aj),\mathcal{I}_{T,R}^{(m)}(-a)\ll T^{\varepsilon+\frac{(m+4)(m-1)}{4}+R\cdot\big{(}D(m)+\frac{m(m-1)}{2}\big{)}-\sum\limits_{j=1}^{m-1}B(a_{j})},

where

B(c)={0 if c<0c+2(cc) if 0<c+ε<cc+12,c if 12<c12c<cε.B(c)=\begin{cases}0&\mbox{ if }c<0\\ \lfloor c\rfloor+2(c-\lfloor c\rfloor)&\mbox{ if }0<\lfloor c\rfloor+\varepsilon<c\leq\lfloor c\rfloor+\frac{1}{2},\\ \lceil c\rceil&\mbox{ if }\frac{1}{2}<\lceil c\rceil-\frac{1}{2}\leq c<\lceil c\rceil-\varepsilon.\end{cases}

The implicit constant depends on ε\varepsilon, RR and mm.

Theorem 8.5 allows us to write W~m,α(s)\widetilde{W}_{m,\alpha}(s) in terms of an integral of the product of several gamma functions and the lower rank Mellin transform W~m1,β(z)\widetilde{W}_{m-1,\beta}(z) where

β=(β1,,βm1):=(α1+αmm1,,αm1+αmm1).\beta=\big{(}\beta_{1},\ldots,\beta_{m-1}\big{)}:=\left(\alpha_{1}+\frac{\alpha_{m}}{m-1},\ldots,\alpha_{m-1}+\frac{\alpha_{m}}{m-1}\right).

Using this, we are able siphon off the contribution to the integrand of 9.1 that is independent of the variable β\beta. This in turn allows us to relate T,R(m)\mathcal{I}_{T,R}^{(m)} to T,R(m1)\mathcal{I}_{T,R}^{(m-1)} and prove the result inductively.

9.1. Symmetry of integration in α\alpha

Since the integrand of (9.1) is invariant under the action of σSm\sigma\in S_{m} acting on α=(α1,,αm)\alpha=(\alpha_{1},\ldots,\alpha_{m}), we may restrict the integration to a fundamental domain. A choice of such a fundamental domain is

(9.1) Im(α1)Im(α2)Im(αm).\operatorname{Im}(\alpha_{1})\geq\operatorname{Im}(\alpha_{2})\geq\cdots\geq\operatorname{Im}(\alpha_{m}).

Hence, (9.1) is equal, up to a constant, to the same integral but restricted to α\alpha satisfying (9.1). In the sequel we will always assume that (9.1) holds.

9.2. Extended exponential zero set

Recall that Stirling’s asymptotic formula (for σ\sigma\in\mathbb{R} fixed and tt\in\mathbb{R} with |t|\lvert t\rvert\to\infty) is given by

(9.1) Γ(σ+it)2π|t|σ12eπ2|t|.\Gamma(\sigma+it)\sim\sqrt{2\pi}\cdot\lvert t\rvert^{\sigma-\frac{1}{2}}\,e^{-\frac{\pi}{2}\lvert t\rvert}.
Definition 9.2 (Exponential and Polynomial Factors of a Ratio of Gamma Functions).

We call |t|σ12\lvert t\rvert^{\sigma-\frac{1}{2}} the polynomial factor of Γ(σ+it)\Gamma(\sigma+it), and eπ2|t|e^{-\frac{\pi}{2}\lvert t\rvert} is called the exponential factor. For a ratio of gamma functions, the polynomial (respectively, exponential) factor is composed of the polynomial (respectively, exponential) factors of each individual Gamma function.

Lemma 9.3 (Extended Exponential Zero Set).

Assume that αm\alpha\in{\mathbb{C}}^{m} is a Langlands parameter satisfying

Im(α1)Im(α2)Im(αm).\operatorname{Im}(\alpha_{1})\geq\operatorname{Im}(\alpha_{2})\geq\ldots\geq\operatorname{Im}(\alpha_{m}).

Then the integrand of T,R(m)\mathcal{I}_{T,R}^{(m)} (as a function of ss) has exponential decay outside of the set I=I1×I2××Im1I=I_{1}\times I_{2}\times\cdots\times I_{m-1}, where

Ij:={sj|k=1jIm(αk)Im(sj)k=1jIm(αmk+1)}.I_{j}:=\left\{s_{j}\left\lvert-\sum_{k=1}^{j}\operatorname{Im}(\alpha_{k})\leq\operatorname{Im}(s_{j})\leq-\sum_{k=1}^{j}\operatorname{Im}(\alpha_{m-k+1})\right.\right\}.
Remark 9.4.

See [GSW21] for the definition of the exponential zero set of an integral. The extended exponential zero set given in Lemma 9.3 contains the exponential zero set for T,R(m)\mathcal{I}_{T,R}^{(m)}.

Proof.

We first prove Lemma 9.3 in the case that m=2m=2. In the formula (9.1) for T,R(n)\mathcal{I}_{T,R}^{(n)}, replace W~2,α(s1)\widetilde{W}_{2,\alpha}(s_{1}) with Γ(s1+α1)Γ(s1+α2)\Gamma(s_{1}+\alpha_{1})\Gamma(s_{1}+\alpha_{2}). Then assuming (9.1), the exponential factor is eπ2(s,α)e^{\frac{\pi}{2}\mathcal{E}(s,\alpha)} where

(s,α)=|Im(s1)+Im(α1)|+|Im(s1)+Im(α2)|2Im(α1).\mathcal{E}(s,\alpha)=\big{\lvert}\operatorname{Im}(s_{1})+\operatorname{Im}(\alpha_{1})\big{\rvert}+\big{\lvert}\operatorname{Im}(s_{1})+\operatorname{Im}(\alpha_{2})\big{\rvert}-2\operatorname{Im}(\alpha_{1}).

We see, therefore, that the exponential factor (s,α)\mathcal{E}(s,\alpha) is negative unless

Im(s1)+Im(α1)0andIm(s1)+Im(α2)0Im(α1)Im(s1)Im(α2),\operatorname{Im}(s_{1})+\operatorname{Im}(\alpha_{1})\geq 0\quad\mbox{and}\quad\operatorname{Im}(s_{1})+\operatorname{Im}(\alpha_{2})\leq 0\quad\Longleftrightarrow\quad-\operatorname{Im}(\alpha_{1})\leq\operatorname{Im}(s_{1})\leq-\operatorname{Im}(\alpha_{2}),

as claimed.

Let us suppose that m3m\geq 3 and c=(c1,c2,,cm1)c=(c_{1},c_{2},\ldots,c_{m-1}) with cj>0c_{j}>0 (j=1,2,,m1j=1,2,\ldots,m-1). In order to prove Lemma 9.3 using induction on mm, we make use of the change of variables

βj=αj+αmm1,j=1,,m1.\beta_{j}=\alpha_{j}+\frac{\alpha_{m}}{m-1},\quad j=1,\ldots,m-1.

Observe that β1+βm1=0\beta_{1}\cdots+\beta_{m-1}=0. By Lemma A.19 in the case that k=m1k=m-1,

α12++αm2=β12+βm12+mm1αm2.\alpha_{1}^{2}+\cdots+\alpha_{m}^{2}=\beta_{1}^{2}+\cdots\beta_{m-1}^{2}+\frac{m}{m-1}\alpha_{m}^{2}.

Then in the integrand for T,R(m)(c)\mathcal{I}_{T,R}^{(m)}(c) we may substitute the formula for W~m,α(s)\widetilde{W}_{m,\alpha}(s) given in Theorem 8.5. We also use the fact (see Lemma A.26) that

1jkmΓ(αjαk)=(1jkm1Γ(βjβk))(i=1m1Γ(αmαi)Γ(αiαm)),\prod_{1\leq j\neq k\leq m}\Gamma(\alpha_{j}-\alpha_{k})=\left(\prod_{1\leq j\neq k\leq m-1}\Gamma(\beta_{j}-\beta_{k})\right)\cdot\left(\prod_{i=1}^{m-1}\Gamma(\alpha_{m}-\alpha_{i})\Gamma(\alpha_{i}-\alpha_{m})\right),

and, via Stirling,

i=1m1Γ(αmαi)Γ(αiαm)eπIm(αm).\prod_{i=1}^{m-1}\Gamma(\alpha_{m}-\alpha_{i})\Gamma(\alpha_{i}-\alpha_{m})\ll e^{\pi\operatorname{Im}(\alpha_{m})}.

Note that (9.1) implies that Im(αm)0\operatorname{Im}(\alpha_{m})\leq 0, hence,

T,R(m)(c)Re(αm)=0emm1αm2T2/2β^m1=0Re(β)=0eβ12++βm12T2/2|𝒫(D(m)D(m1))R(αm,β)|R(m1)(β)1jkm1|ΓR(βjβk)|Re(zj)=bj1jm2|W~m1,β(z)|j=1m1Re(sj)=cj|Γ(sjzj1+(mj)αmm1)Γ(sjzjjαmm1)ΓR(mm1αmβj)ΓR(βj+mm1αm)|dsjdzdα.\mathcal{I}_{T,R}^{(m)}(c)\ll\int\limits_{\operatorname{Re}(\alpha_{m})=0}e^{\frac{m}{m-1}\frac{\alpha_{m}^{2}}{T^{2}/2}}\int\limits_{\begin{subarray}{c}\widehat{\beta}_{m-1}=0\\ \operatorname{Re}(\beta)=0\end{subarray}}e^{\frac{\beta_{1}^{2}+\cdots+\beta_{m-1}^{2}}{T^{2}/2}}\cdot\left|\mathcal{P}_{(D(m)-D(m-1))R}(\alpha_{m},\beta)\right|\\ \cdot\mathcal{F}_{R}^{(m-1)}(\beta)\hskip-3.0pt\prod_{1\leq j\neq k\leq m-1}\hskip-4.0pt\big{|}\Gamma_{R}(\beta_{j}-\beta_{k})\big{|}\int\limits_{\begin{subarray}{c}\operatorname{Re}(z_{j})=b_{j}\\ 1\leq j\leq m-2\end{subarray}}\left|\widetilde{W}_{m-1,\beta}\left(z\right)\right|\\ \cdot\prod_{j=1}^{m-1}\int\limits_{\operatorname{Re}(s_{j})=c_{j}}\hskip-6.0pt\left|\Gamma\Big{(}s_{j}-z_{j-1}+\tfrac{(m-j)\alpha_{m}}{m-1}\Big{)}\Gamma\Big{(}s_{j}-z_{j}-\tfrac{j\alpha_{m}}{m-1}\Big{)}\Gamma_{R}\Big{(}\tfrac{-m}{m-1}\alpha_{m}-\beta_{j}\Big{)}\Gamma_{R}\Big{(}\beta_{j}+\tfrac{m}{m-1}\alpha_{m}\Big{)}\right|\\ \;ds_{j}\,dz\,d\alpha.

By the induction hypothesis, the second row of this expression has exponential decay outside of the set

(9.5) {z=(z1,,zm2)|k=1jβkIm(zj)j=1kβmj},\left\{z=(z_{1},\ldots,z_{m-2})\,\left|\,-\sum_{k=1}^{j}\beta_{k}\leq\operatorname{Im}(z_{j})\leq-\sum_{j=1}^{k}\beta_{m-j}\right.\right\},

for each k=1,2,,m2k=1,2,\ldots,m-2. (Recall that z0=zm1=0z_{0}=z_{m-1}=0.)

The assumption Im(αj)Im(αm)\operatorname{Im}(\alpha_{j})\geq\operatorname{Im}(\alpha_{m}) and the definition of βj\beta_{j} above imply that

Im(αjαm)=Im(βj+mm1αn)0(j=1,2,,m1).\operatorname{Im}(\alpha_{j}-\alpha_{m})=\operatorname{Im}\Big{(}\beta_{j}+\tfrac{m}{m-1}\alpha_{n}\Big{)}\geq 0\quad\mbox{($j=1,2,\ldots,m-1$).}

Thus, the exponential factor coming from the final line in the expression above is eπ2(s,z,β,αm)e^{\frac{\pi}{2}\mathcal{E}(s,z,\beta,\alpha_{m})} where

(s,z,β,αn)\displaystyle\mathcal{E}(s,z,\beta,\alpha_{n}) =j=1n1(|Im(sjzj1+njn1αn)|+|Im(sjzjjn1αn)|Im(nn1αn+βj))\displaystyle=\sum_{j=1}^{n-1}\Big{(}\big{|}\operatorname{Im}(s_{j}-z_{j-1}+\tfrac{n-j}{n-1}\alpha_{n})\big{|}+\big{|}\operatorname{Im}(s_{j}-z_{j}-\tfrac{j}{n-1}\alpha_{n})\big{|}-\operatorname{Im}\big{(}\tfrac{n}{n-1}\alpha_{n}+\beta_{j}\big{)}\Big{)}
=j=1n1(|Im(sjzj1+njn1αn)|+|Im(sjzjjn1αn)|)nIm(αn).\displaystyle=\sum_{j=1}^{n-1}\Big{(}\big{|}\operatorname{Im}(s_{j}-z_{j-1}+\tfrac{n-j}{n-1}\alpha_{n})\big{|}+\big{|}\operatorname{Im}(s_{j}-z_{j}-\tfrac{j}{n-1}\alpha_{n})\big{|}\Big{)}-n\operatorname{Im}(\alpha_{n}).

We know that the integral defining T,R(m)(a)\mathcal{I}_{T,R}^{(m)}(-a) is convergent. Therefore, it must be the case that (s,z,β,αm)0\mathcal{E}(s,z,\beta,\alpha_{m})\leq 0. In order to find where =0\mathcal{E}=0, i.e., where there is not exponential decay, we seek for values ϵ1,1,ϵ2,1,,ϵ1,m1,ϵ2,m1{±1}\epsilon_{1,1},\epsilon_{2,1},\ldots,\epsilon_{1,m-1},\epsilon_{2,m-1}\in\{\pm 1\} for which

(9.6) j=1m1(ϵ1,jIm(sjzj1+mjm1αm)+ϵ2,jIm(sjzjjm1αm))=mIm(αm).\sum_{j=1}^{m-1}\left(\epsilon_{1,j}\operatorname{Im}(s_{j}-z_{j-1}+\tfrac{m-j}{m-1}\alpha_{m})+\epsilon_{2,j}\operatorname{Im}(s_{j}-z_{j}-\tfrac{j}{m-1}\alpha_{m})\right)=m\operatorname{Im}(\alpha_{m}).

In order for the ss-variables to cancel it is clear that for each j=1,2,,m1j=1,2,\ldots,m-1 it need be true that ϵj:=ϵ1,j=ϵ2,j\epsilon_{j}:=\epsilon_{1,j}=-\epsilon_{2,j}. With this assumption, equation 9.6 simplifies:

j=1m1(ϵjIm(zjzj1+mm1αm))=mIm(αm).\displaystyle\sum_{j=1}^{m-1}\Big{(}\epsilon_{j}\operatorname{Im}\big{(}z_{j}-z_{j-1}+\tfrac{m}{m-1}\alpha_{m}\big{)}\Big{)}=m\operatorname{Im}(\alpha_{m}).

In order for this to hold true, it is necessary that ϵj=1\epsilon_{j}=1 for all jj, since otherwise, the coefficients of αm\alpha_{m} on each side of the inequality wouldn’t match. On the other hand, ϵj=1\epsilon_{j}=1 for all jj is sufficient as well since

j=1m1Im(zj1zj)=Im(z0zm1)=0.\sum_{j=1}^{m-1}\operatorname{Im}(z_{j-1}-z_{j})=\operatorname{Im}(z_{0}-z_{m-1})=0.

This unique solution to (9.6) implies, therefore, that there is exponential decay in the integrand of T,R(m)\mathcal{I}_{T,R}^{(m)} above unless Im(zj1mjm1αm)Im(sj)Im(zj+jm1αm).\operatorname{Im}(z_{j-1}-\tfrac{m-j}{m-1}\alpha_{m})\leq\operatorname{Im}(s_{j})\leq\operatorname{Im}(z_{j}+\tfrac{j}{m-1}\alpha_{m}). The inductive assumption (9.5) implies that

Im(zj1mjm1αm)k=1j1(βkαmm1)αm=k=1jαj,\operatorname{Im}(z_{j-1}-\tfrac{m-j}{m-1}\alpha_{m})\geq-\sum_{k=1}^{j-1}\Big{(}\beta_{k}-\tfrac{\alpha_{m}}{m-1}\Big{)}-\alpha_{m}=-\sum_{k=1}^{j}\alpha_{j},

and

Im(zj+jm1αm)k=1j(βkαmm1)=k=1mαk,\operatorname{Im}(z_{j}+\tfrac{j}{m-1}\alpha_{m})\leq-\sum_{k=1}^{j}\big{(}\beta_{k}-\tfrac{\alpha_{m}}{m-1}\big{)}=-\sum_{k=1}^{m}\alpha_{k},

thus yielding the desired bounds on Im(sj)\operatorname{Im}(s_{j}).

To complete the proof, we remark that if a<0-a<0, in order to use the result of Theorem 8.5, we need to first apply the shift equations given in Corollary 9.8 below. This will allow us to rewrite T,R(m)(a)\mathcal{I}_{T,R}^{(m)}(-a) as a sum over terms all of which have the same basic form as that for T,R(m)(c)\mathcal{I}_{T,R}^{(m)}(c) with c>0c>0. Each of these terms has precisely the same exponential factor since this depends only on the imaginary parts of the arguments of the Gamma functions, hence the same exponential zero set is determined in general. ∎

For each j=1,,nj=1,\ldots,n, we define

(9.7) j(sj,α):=K{1,,n}#K=j(sj+kKαk).\mathcal{B}_{j}(s_{j},\alpha):=\prod_{\begin{subarray}{c}K\subseteq\{1,\ldots,n\}\\ \#K=j\end{subarray}}\Big{(}s_{j}+\sum_{k\in K}\alpha_{k}\Big{)}.

Using this, the following corollary is easily deduced. (See [GSW21] for the case of n=4n=4.)

Corollary 9.8.

Let r=(r1,,rn1)0n1r=(r_{1},\ldots,r_{n-1})\in{\mathbb{Z}}_{\geq 0}^{n-1}. There exists a sequence of shifts σ=(σ1,,σn1)0n1\sigma=(\sigma_{1},\ldots,\sigma_{n-1})\in{\mathbb{Z}}_{\geq 0}^{n-1} and polynomials Qσ,r(s,α)Q_{\sigma,r}(s,\alpha) such that

|W~n,α(s)|σ|Qσ,r(s,α)|j=1n1|j(sj,α)|rj|W~n,α(s+r+σ)|,\left|\widetilde{W}_{n,\alpha}(s)\right|\ll\sum_{\sigma}\frac{\lvert Q_{\sigma,r}(s,\alpha)\rvert}{\prod\limits_{j=1}^{n-1}\lvert\mathcal{B}_{j}(s_{j},\alpha)\rvert^{r_{j}}}\left|\widetilde{W}_{n,\alpha}(s+r+\sigma)\right|,

where

Qσ,r(s,α)=j=1n1Pσj,rj(s,α),deg(Pσj,rj(s,α))=rj((nj)2)2σj.Q_{\sigma,r}(s,\alpha)=\prod_{j=1}^{n-1}P_{\sigma_{j},r_{j}}(s,\alpha),\qquad\deg(P_{\sigma_{j},r_{j}}(s,\alpha))=r_{j}\left(\binom{n}{j}-2\right)-2\sigma_{j}.

9.3. Proof of Theorem 9.2 in the case m=2m=2

Proof.

As in the proof of Lemma 9.3, we can replace W~2,(α,α)(s)\widetilde{W}_{2,(\alpha,-\alpha)}(s) with Γ(s+α)Γ(sα)\Gamma(s+\alpha)\Gamma(s-\alpha) and estimate using Stirling’s bound. We may, moreover, restrict ss to the exponential zero set Im(α)Im(s)Im(α)-\operatorname{Im}(\alpha)\leq\operatorname{Im}(s)\leq\operatorname{Im}(\alpha) to see that

T,R(2)(a)\displaystyle\mathcal{I}_{T,R}^{(2)}(-a) =α^n=0Re(α)=0eα2T2|ΓR(2α)ΓR(2α)|s=(s1,,sn1)Re(s)=a|W~2,(α,α)(s)|𝑑s𝑑α\displaystyle=\int\limits_{\begin{subarray}{c}\widehat{\alpha}_{n}=0\\ \operatorname{Re}(\alpha)=0\end{subarray}}e^{\frac{\alpha^{2}}{T^{2}}}\cdot\hskip-3.0pt\big{|}\Gamma_{R}(2\alpha)\Gamma_{R}(-2\alpha)\big{|}\int\limits_{\begin{subarray}{c}s=(s_{1},\ldots,s_{n-1})\\ \operatorname{Re}(s)=-a\end{subarray}}\hskip-5.0pt\left|\widetilde{W}_{2,(\alpha,-\alpha)}(s)\right|\,ds\,d\alpha
α^n=0Re(α)=0eα2T2(1+|2Im(α)|)R+12Re(s)=aIm(α)Im(s)Im(α)(1+|Im(s)Im(α)|)a12\displaystyle\ll\int\limits_{\begin{subarray}{c}\widehat{\alpha}_{n}=0\\ \operatorname{Re}(\alpha)=0\end{subarray}}e^{\frac{\alpha^{2}}{T^{2}}}\cdot\big{(}1+\left|2\operatorname{Im}(\alpha)\right|\big{)}^{R+\frac{1}{2}}\hskip-3.0pt{\int\limits_{\begin{subarray}{c}\operatorname{Re}(s)=-a\\ -\operatorname{Im}(\alpha)\leq\operatorname{Im}(s)\leq\operatorname{Im}(\alpha)\end{subarray}}}\hskip-5.0pt\big{(}1+\lvert\operatorname{Im}(s)-\operatorname{Im}(\alpha)\rvert\big{)}^{-a-\frac{1}{2}}
(1+|Im(s)+Im(α)|)a12dsdα.\displaystyle\hskip 120.0pt\cdot\big{(}1+\lvert\operatorname{Im}(s)+\operatorname{Im}(\alpha)\rvert\big{)}^{-a-\frac{1}{2}}\,ds\,d\alpha.

Due to the presence of the term eα2T2e^{\frac{\alpha^{2}}{T^{2}}}, we may assume moreover that Im(α)T1+ε\operatorname{Im}(\alpha)\leq T^{1+\varepsilon}. Thus, we have the bound

T,R(2)(a)\displaystyle\mathcal{I}_{T,R}^{(2)}(-a) Re(α)=00Im(α)Tε+1(1+2|α|)R+12Re(s)=aIm(α)Im(s)Im(α)(1+αs)a12(1+αs)a12𝑑s𝑑α\displaystyle\ll\ \int\limits_{\begin{subarray}{c}\operatorname{Re}(\alpha)=0\\ 0\leq\operatorname{Im}(\alpha)\leq T^{\varepsilon+1}\end{subarray}}\big{(}1+2\lvert\alpha\rvert\big{)}^{R+\frac{1}{2}}\hskip-12.0pt\int\limits_{\begin{subarray}{c}\operatorname{Re}(s)=-a\\ -\operatorname{Im}(\alpha)\leq\operatorname{Im}(s)\leq\operatorname{Im}(\alpha)\end{subarray}}\hskip-12.0pt\big{(}1+\alpha-s\big{)}^{-a-\frac{1}{2}}\big{(}1+\alpha-s\big{)}^{-a-\frac{1}{2}}\,ds\;d\alpha
Re(α)=00Im(α)Tε+1(1+2|α|)R+12min{a+12,2a}𝑑αTε+R+32min{a+12,2a},\displaystyle\ll\int\limits_{\begin{subarray}{c}\operatorname{Re}(\alpha)=0\\ 0\leq\operatorname{Im}(\alpha)\leq T^{\varepsilon+1}\end{subarray}}\big{(}1+2\lvert\alpha\rvert\big{)}^{R+\frac{1}{2}-\min\big{\{}a+\frac{1}{2},2a\big{\}}}\;d\alpha\ll T^{\varepsilon+R+\frac{3}{2}-\min\big{\{}a+\frac{1}{2},2a\big{\}}},

In the statement of Theorem 9.2, the claimed bound is T,R(2)(a)Tε+R+32B(a)\mathcal{I}_{T,R}^{(2)}(-a)\ll T^{\varepsilon+R+\frac{3}{2}-B(a)}, where B(a)B(a) is as defined in Theorem 9.2. We have in fact proved that T,R(2)(a)Tε+R+32B(a)\mathcal{I}_{T,R}^{(2)}(-a)\ll T^{\varepsilon+R+\frac{3}{2}-B^{\prime}(a)}, where

B(a)=max{a+12,2a}={2a if ε<a12,a+12 if a12.B^{\prime}(a)=\max\big{\{}a+\tfrac{1}{2},2a\big{\}}=\begin{cases}2a&\mbox{ if }\varepsilon<a\leq\frac{1}{2},\\ a+\frac{1}{2}&\mbox{ if }a\geq\frac{1}{2}.\end{cases}

If, a<0a<0, then we may shift the integral over Re(s)=a\operatorname{Re}(s)=-a to be as close to Re(s)=0\operatorname{Re}(s)=0 as desired; indeed, we may make the shift to the point that the error can be absorbed into the ε\varepsilon term in the power of TT. Therefore, since B(a)B(a)B(a)\leq B^{\prime}(a) for all a>0a>0, the Theorem follows. ∎

9.4. Proof of Theorem 9.2 for general mm

Proof.

Let m3m\geq 3 and assume that Theorem 9.2 has been shown to be true for all integers 2k<m2\leq k<m. It follows from Corollary 9.8 with rj=ajr_{j}=\lceil a_{j}\rceil that

T,R(m)(a)\displaystyle\mathcal{I}_{T,R}^{(m)}(-a) σα^m=0Re(α)=0eα12++αm2T2/2R(m)(α)1jkm|ΓR(αjαk)|\displaystyle\ll\sum_{\sigma}\int\limits_{\begin{subarray}{c}\widehat{\alpha}_{m}=0\\ \operatorname{Re}(\alpha)=0\end{subarray}}e^{\frac{\alpha_{1}^{2}+\cdots+\alpha_{m}^{2}}{T^{2}/2}}\cdot\mathcal{F}_{R}^{(m)}(\alpha)\hskip-3.0pt\prod_{1\leq j\neq k\leq m}\hskip-4.0pt\big{|}\Gamma_{R}(\alpha_{j}-\alpha_{k})\big{|}
s=(s1,,sm1)Re(s)=a|𝒫d(m)2|σ|(s,α)|j=1m1|j(sj,α)|aj|W~m,α(s+r+σ)|𝑑s𝑑α\displaystyle\hskip 48.0pt\int\limits_{\begin{subarray}{c}s=(s_{1},\ldots,s_{m-1})\\ \operatorname{Re}(s)=-a\end{subarray}}\hskip-5.0pt\frac{\lvert\mathcal{P}_{d(m)-2\lvert\sigma\rvert}(s,\alpha)\rvert}{\prod\limits_{j=1}^{m-1}\lvert\mathcal{B}_{j}(s_{j},\alpha)\rvert^{\lceil a_{j}\rceil}}\left|\widetilde{W}_{m,\alpha}(s+r+\sigma)\right|\,ds\,d\alpha

By Theorem 8.5,

T,R(m)(a)\displaystyle\mathcal{I}_{T,R}^{(m)}(-a) σα^m=0Re(α)=0eα12++αm2T2/2R(m)(α)1jkm|ΓR(αjαk)|Re(s)=a|𝒫d(m)2|σ|(s,α)|j=1m1|j(sj,α)|aj\displaystyle\ll\sum_{\sigma}\int\limits_{\begin{subarray}{c}\widehat{\alpha}_{m}=0\\ \operatorname{Re}(\alpha)=0\end{subarray}}\hskip-4.0pte^{\frac{\alpha_{1}^{2}+\cdots+\alpha_{m}^{2}}{T^{2}/2}}\cdot\mathcal{F}_{R}^{(m)}(\alpha)\hskip-3.0pt\prod_{1\leq j\neq k\leq m}\hskip-4.0pt\big{|}\Gamma_{R}(\alpha_{j}-\alpha_{k})\big{|}\int\limits_{\operatorname{Re}(s)=-a}\hskip-5.0pt\frac{\lvert\mathcal{P}_{d(m)-2\lvert\sigma\rvert}(s,\alpha)\rvert}{\prod\limits_{j=1}^{m-1}\lvert\mathcal{B}_{j}(s_{j},\alpha)\rvert^{\lceil a_{j}\rceil}}
z=(z1,,zm2)Re(z)=b(j=1m1|Γ(sj+aj+σjzj1+(mj)αmm1)|\displaystyle\hskip 48.0pt\int\limits_{\begin{subarray}{c}z=(z_{1},\ldots,z_{m-2})\\ \operatorname{Re}(z)=b\end{subarray}}\Bigg{(}\prod_{j=1}^{m-1}\left|\Gamma\Big{(}s_{j}+\lceil a_{j}\rceil+\sigma_{j}-z_{j-1}+\frac{(m-j)\alpha_{m}}{m-1}\Big{)}\right|
|Γ(sj+aj+σjzjjαmm1))||W~m1,β(z)|dzdsdα.\displaystyle\hskip 96.0pt\left|\Gamma\Big{(}s_{j}+\lceil a_{j}\rceil+\sigma_{j}-z_{j}-\frac{j\alpha_{m}}{m-1}\Big{)}\Bigg{)}\right|\cdot\left|\widetilde{W}_{m-1,\beta}\left(z\right)\right|\,dz\,ds\,d\alpha.

Next, we use the functional equation for the gamma function to rewrite

Γ(sj+aj+σjzj1+(mj)αmm1)Γ(sj+aj+σjzjjαmm1)=𝒫2σj(s,z,α)Γ(sj+ajzj1+(mj)αmm1)Γ(sj+ajzjjαmm1).\Gamma\Big{(}s_{j}+\lceil a_{j}\rceil+\sigma_{j}-z_{j-1}+\frac{(m-j)\alpha_{m}}{m-1}\Big{)}\Gamma\Big{(}s_{j}+\lceil a_{j}\rceil+\sigma_{j}-z_{j}-\frac{j\alpha_{m}}{m-1}\Big{)}\\ =\mathcal{P}_{2\sigma_{j}}(s,z,\alpha)\Gamma\Big{(}s_{j}+\lceil a_{j}\rceil-z_{j-1}+\frac{(m-j)\alpha_{m}}{m-1}\Big{)}\Gamma\Big{(}s_{j}+\lceil a_{j}\rceil-z_{j}-\frac{j\alpha_{m}}{m-1}\Big{)}.

Additionally, we use the fact that the integrand has exponential decay unless |α1|,,|αm|T1+ε\lvert\alpha_{1}\rvert,\ldots,\lvert\alpha_{m}\rvert\leq T^{1+\varepsilon}, and by Lemma 9.3, each of the variables sjs_{j} are bounded in terms of α\alpha. This means that we may replace the polynomials 𝒫2σj\mathcal{P}_{2\sigma_{j}} with the bound Tε+2σjT^{\varepsilon+2\sigma_{j}}. Note that in doing so, the dependence on σ\sigma is removed:

T,R(m)(a)\displaystyle\mathcal{I}_{T,R}^{(m)}(-a) Tε+j=1m1aj((mj)2)α^m=0Re(α)=0eα12++αm2T2/2R(m)(α)1jkm|ΓR(αjαk)|s=(s1,,sm1)Re(s)=a\displaystyle\ll T^{\varepsilon+\sum\limits_{j=1}^{m-1}\lceil a_{j}\rceil\left(\binom{m}{j}-2\right)}\int\limits_{\begin{subarray}{c}\widehat{\alpha}_{m}=0\\ \operatorname{Re}(\alpha)=0\end{subarray}}e^{\frac{\alpha_{1}^{2}+\cdots+\alpha_{m}^{2}}{T^{2}/2}}\cdot\mathcal{F}_{R}^{(m)}(\alpha)\hskip-3.0pt\prod_{1\leq j\neq k\leq m}\hskip-4.0pt\big{|}\Gamma_{R}(\alpha_{j}-\alpha_{k})\big{|}\int\limits_{\begin{subarray}{c}s=(s_{1},\ldots,s_{m-1})\\ \operatorname{Re}(s)=-a\end{subarray}}
z=(z1,,zm2)Re(z)=b(j=1m1|Γ(sj+ajzj1+(mj)αmm1)Γ(sj+ajzjjαmm1)||j(sj,α)|aj)\displaystyle\hskip 24.0pt\cdot\int\limits_{\begin{subarray}{c}z=(z_{1},\ldots,z_{m-2})\\ \operatorname{Re}(z)=b\end{subarray}}\left(\prod_{j=1}^{m-1}\frac{\left|\Gamma\Big{(}s_{j}+\lceil a_{j}\rceil-z_{j-1}+\frac{(m-j)\alpha_{m}}{m-1}\Big{)}\Gamma\Big{(}s_{j}+\lceil a_{j}\rceil-z_{j}-\frac{j\alpha_{m}}{m-1}\Big{)}\right|}{\lvert\mathcal{B}_{j}(s_{j},\alpha)\rvert^{\lceil a_{j}\rceil}}\right)
|W~m1,β(z)|dzdsdα.\displaystyle\hskip 192.0pt\cdot\left|\widetilde{W}_{m-1,\beta}\left(z\right)\right|\,dz\,ds\,d\alpha.

Notice that the conclusion of Proposition 9.2 follows from the last several steps by simply replacing ss by s+Ls+L in the integrand (or, equivalently, replace Re(s)=a\operatorname{Re}(s)=-a by Re(s)=a+L\operatorname{Re}(s)=-a+L in the domain of integration), and then at the step where the functional equation of Gamma is used to remove σ\sigma from the gamma functions, we remove LL in the exact same fashion.

We deduce that

T,R(m)(a)\displaystyle\mathcal{I}_{T,R}^{(m)}(-a) Tε+j=1m1aj((mj)2)α^m=0Re(α)=0eα12++αm2T2/2R(m)(α)1jkm1|ΓR(αjαk)|z=(z1,,zm2)Re(z)=b\displaystyle\ll T^{\varepsilon+\sum\limits_{j=1}^{m-1}\lceil a_{j}\rceil\left(\binom{m}{j}-2\right)}\cdot\int\limits_{\begin{subarray}{c}\widehat{\alpha}_{m}=0\\ \operatorname{Re}(\alpha)=0\end{subarray}}e^{\frac{\alpha_{1}^{2}+\cdots+\alpha_{m}^{2}}{T^{2}/2}}\cdot\mathcal{F}_{R}^{(m)}(\alpha)\hskip-6.0pt\prod_{1\leq j\neq k\leq m-1}\hskip-4.0pt\big{|}\Gamma_{R}(\alpha_{j}-\alpha_{k})\big{|}\int\limits_{\begin{subarray}{c}z=(z_{1},\ldots,z_{m-2})\\ \operatorname{Re}(z)=b\end{subarray}}
j=1m1Re(sj)=ajaj|Γ(sjzj1(mj)α^m1)Γ(sjzj+jα^m1)||j(sj,α)|aj|ΓR(nn1α^βj)ΓR(βjmm1α^)|\displaystyle\hskip-30.0pt\prod_{j=1}^{m-1}\int\limits_{\operatorname{Re}(s_{j})=\lceil a_{j}\rceil-a_{j}}\hskip-10.0pt\frac{\left|\Gamma\Big{(}s_{j}-z_{j-1}-\frac{(m-j)\widehat{\alpha}}{m-1}\Big{)}\Gamma\Big{(}s_{j}-z_{j}+\frac{j\widehat{\alpha}}{m-1}\Big{)}\right|}{\lvert\mathcal{B}_{j}(s_{j},\alpha)\rvert^{\lceil a_{j}\rceil}}\Big{|}\Gamma_{R}\big{(}\tfrac{n}{n-1}\widehat{\alpha}-\beta_{j}\big{)}\Gamma_{R}\big{(}\beta_{j}-\tfrac{m}{m-1}\widehat{\alpha}\big{)}\Big{|}
|W~m1,β(z)|dsjdzdα.\displaystyle\hskip 192.0pt\cdot\left|\widetilde{W}_{m-1,\beta}\left(z\right)\right|ds_{j}\,dz\,d\alpha.

Note that we have also made the change of variable ssjajs\mapsto s_{j}-\lceil a_{j}\rceil for each j=1,2,,m1j=1,2,\ldots,m-1, and we are using the notation α^:=αm\widehat{\alpha}:=-\alpha_{m}. (Using the terminology of Lemma A.19 in the case of k=m1k=m-1, we have α^=α^m1\widehat{\alpha}=\widehat{\alpha}_{m-1}.) As in the case of n=2n=2, due to the presence of the exponential terms, we see that the integral has exponential decay unless |αj|T1+ε\lvert\alpha_{j}\rvert\ll T^{1+\varepsilon}.

Lemma 9.1.

Let α=(α1,,αm)\alpha=(\alpha_{1},\ldots,\alpha_{m}) and βj=αjα^m1\beta_{j}=\alpha_{j}-\frac{\widehat{\alpha}}{m-1} be as above. In particular, they are purely imaginary with |βk|,|α^j|<T1+ε\lvert\beta_{k}\rvert,\lvert\widehat{\alpha}_{j}\rvert<T^{1+\varepsilon}. Suppose, moreover, that α\alpha is in jj-general position. Then

Re(sj)=ajaj|Γ(sjzj1(mj)α^m1)Γ(sjzj+jα^m1)||j(sj,α)|aj|ΓR(mm1α^βj)ΓR(βjmm1α^)|𝑑sjTε+R+12+max{0,2(ajaj)1}L{1,,m}#L=jK{1,,m}#K=jKL(1+|LαkKαk|)[aj].\int\limits_{\operatorname{Re}(s_{j})=\lceil a_{j}\rceil-a_{j}}\hskip-8.0pt\frac{\left|\Gamma\Big{(}s_{j}-z_{j-1}-\frac{(m-j)\widehat{\alpha}}{m-1}\Big{)}\Gamma\Big{(}s_{j}-z_{j}+\frac{j\widehat{\alpha}}{m-1}\Big{)}\right|}{\lvert\mathcal{B}_{j}(s_{j},\alpha)\rvert^{\lceil a_{j}\rceil}}\Big{|}\Gamma_{R}\big{(}\tfrac{m}{m-1}\widehat{\alpha}-\beta_{j}\big{)}\Gamma_{R}\big{(}\beta_{j}-\frac{m}{m-1}\widehat{\alpha}\big{)}\Big{|}\;ds_{j}\\ \ll T^{\varepsilon+R+\frac{1}{2}+\max\{0,2(\lceil a_{j}\rceil-a_{j})-1\}}\sum_{\begin{subarray}{c}L\subseteq\{1,\ldots,m\}\\ \#L=j\end{subarray}}\prod_{\begin{subarray}{c}K\subseteq\{1,\ldots,m\}\\ \#K=j\\ K\neq L\end{subarray}}\left(1+\Big{|}\sum_{\ell\in L}\alpha_{\ell}-\sum_{k\in K}\alpha_{k}\Big{|}\right)^{-[a_{j}]}.
Proof.

Let j\mathcal{I}_{j} denote the integral we are seeking to bound.

The polynomial part (see Definition 9.2) of the Gamma functions in j\mathcal{I}_{j} is

|𝒬j(s,z,α)|(1+Im(βjnn1α^))ε+R+12(1+|Im(sjzj)|)ajajRe(zj)12(1+|Im(sjzj1)|)ajajRe(zj1)12,\lvert\mathcal{Q}_{j}(s,z,\alpha)\rvert\ll\big{(}1+\operatorname{Im}(\beta_{j}-\tfrac{n}{n-1}\widehat{\alpha})\big{)}^{\varepsilon+R+\frac{1}{2}}\big{(}1+\lvert\operatorname{Im}(s_{j}-z_{j})\rvert\big{)}^{\lceil a_{j}\rceil-a_{j}-\operatorname{Re}(z_{j})-\frac{1}{2}}\\ \cdot\big{(}1+\lvert\operatorname{Im}(s_{j}-z_{j-1})\rvert\big{)}^{\lceil a_{j}\rceil-a_{j}-\operatorname{Re}(z_{j-1})-\frac{1}{2}},

and the exponential factor (when taking all j\mathcal{I}_{j} in unison) is negative for any sjs_{j} outside of the interval IjI_{j} defined in Lemma 9.3. That lemma together with the presence of the other exponential terms in our integral allow us to take trivial bounds for the polynomial part, namely that 𝒬j(s,z,α)Tε+R+12+max{0,2(ajaj)1}\mathcal{Q}_{j}(s,z,\alpha)\ll T^{\varepsilon+R+\frac{1}{2}+\max\{0,2(\lceil a_{j}\rceil-a_{j})-1\}}. (Recall that 0Re(zj)0\leq\operatorname{Re}(z_{j}).) Thus we see that

j\displaystyle\mathcal{I}_{j} Tε+R+12+max{0,2(ajaj)1}Re(sj)=ajajIm(sj)IjJ{1,,n}#J=j|sj+kJαk|ajdsj,\displaystyle\ll T^{\varepsilon+R+\frac{1}{2}+\max\{0,2(\lceil a_{j}\rceil-a_{j})-1\}}\hskip-6.0pt\int\limits_{\begin{subarray}{c}\operatorname{Re}(s_{j})=\lceil a_{j}\rceil-a_{j}\\ \operatorname{Im}(s_{j})\in I_{j}\end{subarray}}\prod\limits_{\begin{subarray}{c}J\subseteq\{1,\ldots,n\}\\ \#J=j\end{subarray}}\Big{|}s_{j}+\sum\limits_{k\in J}\alpha_{k}\Big{|}^{-\lceil a_{j}\rceil}\,ds_{j},

The desired result now follows easily from this and the statement of Lemma A.3. ∎

Combining Lemma 9.1 with the bound for T,R(n)(a)\mathcal{I}_{T,R}^{(n)}(-a) given immediately before the statement of the lemma, and applying Lemma A.19, Lemma A.26 and Lemma A.27 (in the case that k=n1k=n-1 and γ1=0\gamma_{1}=0), we now have the bound

T,R(m)(a)\displaystyle\mathcal{I}_{T,R}^{(m)}(-a) L{1,,m}#L=jTε+(R+12)(m1)+j=1m1(max{0,2(ajaj)1}+aj((mj)2))Re(α^)=0emm1α^22T2\displaystyle\ll\sum_{\begin{subarray}{c}L\subseteq\{1,\ldots,m\}\\ \#L=j\end{subarray}}T^{\varepsilon+(R+\frac{1}{2})(m-1)+\sum\limits_{j=1}^{m-1}\Big{(}\max\{0,2(\lceil a_{j}\rceil-a_{j})-1\}+\lceil a_{j}\rceil\left(\binom{m}{j}-2\right)\Big{)}}\cdot\hskip-6.0pt\int\limits_{\operatorname{Re}(\widehat{\alpha})=0}e^{\frac{m}{m-1}\frac{\widehat{\alpha}^{2}}{2T^{2}}}
β^m1=0Re(β)=0eβ12++βm12T2/2𝒫D(m)D(m1)R(α^,β)R(m1)(β)1jkm1|ΓR(βjβk)|\displaystyle\hskip 24.0pt\cdot\int\limits_{\begin{subarray}{c}\widehat{\beta}_{m-1}=0\\ \operatorname{Re}(\beta)=0\end{subarray}}e^{\frac{\beta_{1}^{2}+\cdots+\beta_{m-1}^{2}}{T^{2}/2}}\hskip-3.0pt\cdot\mathcal{P}_{D(m)-D(m-1)}^{R}(\widehat{\alpha},\beta)\cdot\mathcal{F}_{R}^{(m-1)}(\beta)\hskip-3.0pt\prod_{1\leq j\neq k\leq m-1}\hskip-4.0pt\big{|}\Gamma_{R}(\beta_{j}-\beta_{k})\big{|}
j=1m1K{1,,m}#K=jKL(1+|LαkKαk|)[aj]z=(z1,,zm2)Re(z)=b|W~m1,β(z)|dzdβdα^.\displaystyle\hskip 24.0pt\cdot\prod_{j=1}^{m-1}\prod_{\begin{subarray}{c}K\subseteq\{1,\ldots,m\}\\ \#K=j\\ K\neq L\end{subarray}}\left(1+\Big{|}\sum_{\ell\in L}\alpha_{\ell}-\sum_{k\in K}\alpha_{k}\Big{|}\right)^{-[a_{j}]}\int\limits_{\begin{subarray}{c}z=(z_{1},\ldots,z_{m-2})\\ \operatorname{Re}(z)=b\end{subarray}}\left|\widetilde{W}_{m-1,\beta}\left(z\right)\right|\;dz\,d\beta\,d\widehat{\alpha}.

To be more explicit, the polynomial 𝒫D(m)D(m1)R(α^,β)\mathcal{P}_{D(m)-D(m-1)}^{R}(\widehat{\alpha},\beta) is the portion of R(m)(α)\mathcal{F}_{R}^{(m)}(\alpha) which involves the terms αm\alpha_{m}.

At this point, we combine each of the terms in the final row with the corresponding term in R(m)(α)\mathcal{F}_{R}^{(m)}(\alpha). Strictly speaking, what is actually happening here is that this has the effect of reducing the power of each factor of R(m)(α)\mathcal{F}_{R}^{(m)}(\alpha) by at most

max{a1,,am1}.\max\{\lceil a_{1}\rceil,\ldots,\lceil a_{m-1}\rceil\}.

Since each of the corresponding exponents remains positive, the net result is to reduce the overall power of TT by

ε+j=1m1aj((mj)1).\varepsilon+\sum_{j=1}^{m-1}\lceil a_{j}\rceil\left(\binom{m}{j}-1\right).

Using this, and accounting for the integration in α^\widehat{\alpha} (which may be assumed to take place only for |Im(α^)|T1+ε\lvert\operatorname{Im}(\widehat{\alpha})\rvert\leq T^{1+\varepsilon}), we now may write

T,R(m)(a)\displaystyle\mathcal{I}_{T,R}^{(m)}(-a)\ll Tε+(R+12)(n1)+R(D(m)D(m1))+1+j=1m1(max{0,2(ajaj)1}aj)β^m1=0Re(β)=0\displaystyle\ T^{\varepsilon+(R+\frac{1}{2})(n-1)+R(D(m)-D(m-1))+1+\sum\limits_{j=1}^{m-1}\big{(}\max\{0,2(\lceil a_{j}\rceil-a_{j})-1\}-\lceil a_{j}\rceil\big{)}}\int\limits_{\begin{subarray}{c}\widehat{\beta}_{m-1}=0\\ \operatorname{Re}(\beta)=0\end{subarray}}
eβ12++βm122T2R(m1)(β)1jkm1|ΓR(βjβk)|z=(z1,,zm2)Re(z)=b|W~m1,β(z)|𝑑z𝑑β.\displaystyle\hskip 6.0pt\cdot e^{\frac{\beta_{1}^{2}+\cdots+\beta_{m-1}^{2}}{2T^{2}}}\hskip-3.0pt\cdot\mathcal{F}_{R}^{(m-1)}(\beta)\hskip-3.0pt\prod_{1\leq j\neq k\leq m-1}\hskip-4.0pt\big{|}\Gamma_{R}(\beta_{j}-\beta_{k})\big{|}\int\limits_{\begin{subarray}{c}z=(z_{1},\ldots,z_{m-2})\\ \operatorname{Re}(z)=b\end{subarray}}\left|\widetilde{W}_{m-1,\beta}\left(z\right)\right|\;dz\,d\beta.

Obviously, at this point we want to apply the inductive hypothesis. Since at this point we only need to do so in the case that bj>0b_{j}>0 (i.e., aj<0-a_{j}<0) for all j=1,,m2j=1,\ldots,m-2, the reduction in the powers of the exponents of any one of the factors of R(m)(α)\mathcal{F}_{R}^{(m)}(\alpha), as occurred above, leaves the overall power positive. Therefore, there is no issue, and we can assert (additionally applying Lemma A.5) the bound

T,R(m)(a)\displaystyle\mathcal{I}_{T,R}^{(m)}(-a)\ll Tε+(R+12)(m1)+R(D(m)D(m1))+1+A(m1)TR(D(m1)+(m1)(m2)2)j=1m1B(aj)\displaystyle\ T^{\varepsilon+(R+\frac{1}{2})(m-1)+R(D(m)-D(m-1))+1+A(m-1)}\cdot T^{R\big{(}D(m-1)+\frac{(m-1)(m-2)}{2}\big{)}-\sum\limits_{j=1}^{m-1}B(a_{j})}
=\displaystyle= Tε+R(D(m)+m(m1)2)+n+12+A(m1)j=1m1B(aj).\displaystyle\ T^{\varepsilon+R\big{(}D(m)+\frac{m(m-1)}{2}\big{)}+\frac{n+1}{2}+A(m-1)-\sum\limits_{j=1}^{m-1}B(a_{j})}.

Taking A(m)=m+12+A(m1)A(m)=\tfrac{m+1}{2}+A(m-1) gives the claimed bound. Since A(2)=32A(2)=\frac{3}{2}, it follows that

A(3)=42+A(2)=12(4+3),,A(m)=12((m+1)+m++3)=(m+4)(m1)4,A(3)=\tfrac{4}{2}+A(2)=\tfrac{1}{2}(4+3),\ldots,A(m)=\tfrac{1}{2}\big{(}(m+1)+m+\cdots+3\big{)}=\tfrac{(m+4)(m-1)}{4},

as claimed. ∎

In the course of proving Theorem 9.2 we also established the following result that we record here since it will be useful in its own right.

Proposition 9.2.

Suppose that L=(1,2,,m1)(0)m1L=(\ell_{1},\ell_{2},\ldots,\ell_{m-1})\in\big{(}{\mathbb{Z}}_{\geq 0}\big{)}^{m-1}. Then

α^m=0Re(α)=0eα12++αm22T2R(m)(α)1jkm|ΓR(αjαk)|s=(s1,,sm1)Re(s)=a|W~m,α(s+L)|𝑑s𝑑αTε+2|L|α^m=0Re(α)=0eα12++αm22T2R(m)(α)1jkm|ΓR(αjαk)|s=(s1,,sm1)Re(s)=a|W~m,α(s)|𝑑s𝑑α.\int\limits_{\begin{subarray}{c}\widehat{\alpha}_{m}=0\\ \operatorname{Re}(\alpha)=0\end{subarray}}e^{\frac{\alpha_{1}^{2}+\cdots+\alpha_{m}^{2}}{2T^{2}}}\cdot\mathcal{F}_{R}^{(m)}(\alpha)\hskip-3.0pt\prod_{1\leq j\neq k\leq m}\hskip-4.0pt\big{|}\Gamma_{R}(\alpha_{j}-\alpha_{k})\big{|}\int\limits_{\begin{subarray}{c}s=(s_{1},\ldots,s_{m-1})\\ \operatorname{Re}(s)=-a\end{subarray}}\hskip-5.0pt\left|\widetilde{W}_{m,\alpha}(s+L)\right|\,ds\,d\alpha\\ \ll T^{\varepsilon+2\lvert L\rvert}\cdot\int\limits_{\begin{subarray}{c}\widehat{\alpha}_{m}=0\\ \operatorname{Re}(\alpha)=0\end{subarray}}e^{\frac{\alpha_{1}^{2}+\cdots+\alpha_{m}^{2}}{2T^{2}}}\cdot\mathcal{F}_{R}^{(m)}(\alpha)\hskip-3.0pt\prod_{1\leq j\neq k\leq m}\hskip-4.0pt\big{|}\Gamma_{R}(\alpha_{j}-\alpha_{k})\big{|}\int\limits_{\begin{subarray}{c}s=(s_{1},\ldots,s_{m-1})\\ \operatorname{Re}(s)=-a\end{subarray}}\hskip-5.0pt\left|\widetilde{W}_{m,\alpha}(s)\right|\,ds\,d\alpha.

As a shorthand for this result, we write T,R(m)(a+L)Tε+2|L|T,R(m)(a)\mathcal{I}_{T,R}^{(m)}(-a+L)\ll T^{\varepsilon+2\lvert L\rvert}\cdot\mathcal{I}_{T,R}^{(m)}(-a).

10. Bounding pT,R(n)(y)p_{T,R}^{(n)}(y)

In this section we prove the following.

Theorem 10.1.

Let n2n\geq 2 and ε(0,14)\varepsilon\in(0,\frac{1}{4}). Suppose that a=(a1,a2,,an1)a=(a_{1},a_{2},\ldots,a_{n-1}) satisfies aj+ε<aj<ajε\lfloor a_{j}\rfloor+\varepsilon<a_{j}<\lceil a_{j}\rceil-\varepsilon for each j=1,,n1j=1,\ldots,n-1. Let 𝒞\mathcal{C} be the set of compositions n=n1++nrn=n_{1}+\cdots+n_{r} with r2r\geq 2. Then, for

Δa(C):={δC=(δ1,,δr1)r1| 0δj<an^j(j=1,,r1)},\Delta_{a}(C):=\Big{\{}\delta_{C}=(\delta_{1},\ldots,\delta_{r-1})\in{\mathbb{Z}}^{r-1}\ \Big{|}\ 0\leq\delta_{j}<a_{\widehat{n}_{j}}\ (j=1,\ldots,r-1)\Big{\}},

and B(c)B(c) as defined in Theorem 9.2, we have

(10.2) |pT,R(n)(y)||pT,R(n)(y;a)|+C𝒞δCΔa(C)|pT,R(n)(y;a,δC)|,\left\lvert p_{T,R}^{(n)}(y)\right\rvert\ll\left\lvert p_{T,R}^{(n)}(y;-a)\right\rvert+\sum_{C\in\mathcal{C}}\sum_{\delta_{C}\in\Delta_{a}(C)}\left\lvert p_{T,R}^{(n)}(y;-a,\delta_{C})\right\rvert,

where

(10.3) |pT,R(n)(y;a)|j=1n1yjn(nj)2+2ajTε+(n+4)(n1)4+R2((2nn)2n)j=1n1B(aj),\left|p_{T,R}^{(n)}(y;-a)\right|\ll\prod_{j=1}^{n-1}y_{j}^{\frac{n(n-j)}{2}+2a_{j}}\cdot T^{\varepsilon+\frac{(n+4)(n-1)}{4}+\frac{R}{2}\cdot\left(\binom{2n}{n}-2^{n}\right)-\sum\limits_{j=1}^{n-1}B(a_{j})},

and

(10.4) |pT,R(n)(y;a,δC)|j=1n1yjn(nj)2+2ajTε+(n+4)(n1)4+R2((2nn)2n)j=1n1B(aj)12k=1r1(nk+nk+1)(an^kδk).\big{\lvert}p_{T,R}^{(n)}(y;-a,\delta_{C})\big{\rvert}\ll\prod_{j=1}^{n-1}y_{j}^{\frac{n(n-j)}{2}+2a_{j}}\cdot T^{\varepsilon+\frac{(n+4)(n-1)}{4}+\frac{R}{2}\cdot\left(\binom{2n}{n}-2^{n}\right)-\sum\limits_{j=1}^{n-1}B(a_{j})-\frac{1}{2}\sum\limits_{k=1}^{r-1}(n_{k}+n_{k+1})(a_{\widehat{n}_{k}}-\delta_{k})}.

The implicit constant depends on both ε\varepsilon and nn.

Remark 10.5.

Note that (10.4) is bounded by (10.3). Therefore, letting D(n)=12(2nn)n(n1)22n1D(n)=\frac{1}{2}\binom{2n}{n}-\frac{n(n-1)}{2}-2^{n-1} as in (1.2), Theorem 10.1 implies that

|pT,R(n)(y)|j=1n1yjn(nj)2+2ajTε+(n+4)(n1)4+R(D(n)+n(n+1)2)j=1n1B(aj)\left\lvert p_{T,R}^{(n)}(y)\right\rvert\ll\prod_{j=1}^{n-1}y_{j}^{\frac{n(n-j)}{2}+2a_{j}}\cdot T^{\varepsilon+\frac{(n+4)(n-1)}{4}+R\cdot\big{(}D(n)+\frac{n(n+1)}{2}\big{)}-\sum\limits_{j=1}^{n-1}B(a_{j})}

10.1. Explicit single residue formula

In order to bound the terms pT,R(n)(y;a,δC)p_{T,R}^{(n)}(y;-a,\delta_{C}) we need an explicit formula for the residues of the Mellin transform of the GL(n)\operatorname{GL}(n) Whittaker function. The following result establishes this for the case of single residues (i.e., when the composition CC has length 22) as a corollary of Conjecture 8.3 combined with a theorem of Stade [Sta01] for the “first” residues, i.e., for those residues corresponding, in the notation of the theorem, to δ=0\delta=0.

Theorem 10.1.

Let W~m,α(s)\widetilde{W}_{m,\alpha}(s) be the Mellin transform of the Whittaker function on GL(n,)\operatorname{GL}(n,\mathbb{R}) with purely imaginary parameters α=(α1,,αn)\alpha=(\alpha_{1},\ldots,\alpha_{n}) in general position. Let σSn\sigma\in S_{n} act on α\alpha via

σα:=(ασ(1),ασ(2),,ασ(n)).\sigma\cdot\alpha:=(\alpha_{\sigma(1)},\alpha_{\sigma(2)},\ldots,\alpha_{\sigma(n)}).

The poles of W~n,α(s)\widetilde{W}_{n,\alpha}(s) occur, for each 1mn11\leq m\leq n-1, at

sm{σα^mδ|σSn,δ0,}.s_{m}\in\big{\{}-\sigma\cdot\widehat{\alpha}_{m}-\delta\,\big{|}\,\sigma\in S_{n},\delta\in{\mathbb{Z}}_{\geq 0},\big{\}}.

The residue at sm=α^mδs_{m}=-\widehat{\alpha}_{m}-\delta is equal to a sum over shifts L=(1,2,,n1)L=(\ell_{1},\ell_{2},\ldots,\ell_{n-1}) of terms of the form

K{1,2,,n}#(K{1,2,,m})m1#K=m((iKαi)α^mδ)δ1(i=1mj=m+1nΓ(αjαiδ))𝒫((nm)2)δ2|L|(s,α)W~m,β(s+L)W~nm,γ(s′′+L′′),\prod_{\begin{subarray}{c}K\subseteq\{1,2,\ldots,n\}\\ \#(K\cap\{1,2,\ldots,m\})\neq m-1\\ \#K=m\end{subarray}}\left(\Big{(}\sum_{i\in K}\alpha_{i}\Big{)}-\widehat{\alpha}_{m}-\delta\right)_{\delta}^{-1}\left(\prod_{i=1}^{m}\prod_{j=m+1}^{n}\Gamma(\alpha_{j}-\alpha_{i}-\delta)\right)\\ \cdot\mathcal{P}_{\left(\binom{n}{m}-2\right)\delta-2\lvert L\rvert}(s,\alpha)\widetilde{W}_{m,\beta}(s^{\prime}+L^{\prime})\widetilde{W}_{n-m,\gamma}(s^{\prime\prime}+L^{\prime\prime}),

where

(10.2) s=(sj+jmα^m)|1jm,s′′=(sm+j+nmjnmα^m)|1jnm,s^{\prime}=\left.\Big{(}s_{j}+\tfrac{j}{m}\widehat{\alpha}_{m}\Big{)}\right|_{1\leq j\leq m},\quad\quad s^{\prime\prime}=\left.\Big{(}s_{m+j}+\tfrac{n-m-j}{n-m}\widehat{\alpha}_{m}\Big{)}\right|_{1\leq j\leq n-m},

with L=(1,,m1)L^{\prime}=(\ell_{1},\ldots,\ell_{m-1}) and L′′=(m+1,,n1)L^{\prime\prime}=(\ell_{m+1},\ldots,\ell_{n-1}) being the portion of LL corresponding to ss^{\prime} and s′′s^{\prime\prime} respectively. It is the case that m1=m+1=0\ell_{m-1}=\ell_{m+1}=0. Note that we take as definition that W~1:=1\widetilde{W}_{1}:=1. The same formula holds for the residue at sm=σα^mδs_{m}=-\sigma\cdot\widehat{\alpha}_{m}-\delta by replacing each instance of αj\alpha_{j} with ασ(j)\alpha_{\sigma(j)}.

Remark 10.3.

Another way of writing the above expression for the residue would be to take the product over all K{1,,n}K\subseteq\{1,\ldots,n\} with #K=m\#K=m and replace Γ(αjαiδ)\Gamma(\alpha_{j}-\alpha_{i}-\delta) with Γ(αjαi)\Gamma(\alpha_{j}-\alpha_{i}). The two versions are equivalent because if K{1,,m}={j}K\setminus\{1,\ldots,m\}=\{j\}, then {1,,m}K={k}\{1,\ldots,m\}\setminus K=\{k\} and

((iKαi)α^mδ)δ1Γ(αjαk)=Γ(αjαkδ).\left(\Big{(}\sum_{i\in K}\alpha_{i}\Big{)}-\widehat{\alpha}_{m}-\delta\right)_{\delta}^{-1}\Gamma(\alpha_{j}-\alpha_{k})=\Gamma(\alpha_{j}-\alpha_{k}-\delta).
Sketch of proof.

In the case that δ=0\delta=0, this result (for L=(0,,0)n1L=(0,\ldots,0)\in{\mathbb{C}}^{n-1}) agrees with [Sta01, Theorem 3.1]. If δ>0\delta>0, we need to first apply Conjecture 8.3 to rewrite the expression for W~n,α(s)\widetilde{W}_{n,\alpha}(s) around sm=αmδs_{m}=-\alpha_{m}-\delta as a sum over shifts L=(1,,n1)(0)n1L=(\ell_{1},\ldots,\ell_{n-1})\in({\mathbb{Z}}_{\geq 0})^{n-1} (with mδ\ell_{m}\geq\delta for each LL) of terms W~n,α(s+L)\widetilde{W}_{n,\alpha}(s+L). Of all of these terms, the only ones for which there is a pole at sm=α^mδs_{m}=-\widehat{\alpha}_{m}-\delta are those for which m=δ\ell_{m}=\delta, in which case we can use the above referenced theorem of Stade to write down the residue. Doing so, we obtain the alternate expression referenced to in Remark 10.3. ∎

10.2. Explicit higher residue formulae

In order to generalize Theorem 10.1, we first establish notation related to the (r1)(r-1)-fold residue of W~n,α(s)\widetilde{W}_{n,\alpha}(s) at

sn^=α^n^δn^,=1,,r1.s_{\widehat{n}_{\ell}}=-\widehat{\alpha}_{\widehat{n}_{\ell}}-\delta_{\widehat{n}_{\ell}},\qquad\ell=1,\ldots,r-1.

To this end, let s(j):=(s1(j),,snj1(j))s^{(j)}:=(s_{1}^{(j)},\ldots,s_{n_{j}-1}^{(j)}) where sk(j)=sn^j1+ks_{k}^{(j)}=s_{\widehat{n}_{j-1}+k}. By abuse of notation, we write

s:=(s1(1),s2(1),,sn11(1)=:s(1),s1(2),s2(2),,sn21(2)=:s(2),,s1(k),s2(k),,snk1(k)=:s(k))nr,s:=\big{(}\underbrace{s_{1}^{(1)},s_{2}^{(1)},\ldots,s_{n_{1}-1}^{(1)}}_{=:s^{(1)}}\ ,\ \underbrace{s_{1}^{(2)},s_{2}^{(2)},\ldots,s_{n_{2}-1}^{(2)}}_{=:s^{(2)}}\ ,\ \ldots\ ,\ \underbrace{s_{1}^{(k)},s_{2}^{(k)},\ldots,s_{n_{k}-1}^{(k)}}_{=:s^{(k)}}\big{)}\in{\mathbb{C}}^{n-r},

which agrees with the original s=(s1,,sn1)s=(s_{1},\ldots,s_{n-1}) but removes sn^1,,sn^r1s_{\widehat{n}_{1}},\ldots,s_{\widehat{n}_{r-1}}.

Similarly, if α=(α1,,αn)\alpha=(\alpha_{1},\ldots,\alpha_{n}), we define

α():=(α1(),,α())n,αj():=αn^1+j1n(α^n^α^n^1),\alpha^{(\ell)}:=\big{(}\alpha_{1}^{(\ell)},\ldots,\alpha_{\ell}^{(\ell)}\big{)}\in{\mathbb{C}}^{n_{\ell}},\qquad\alpha_{j}^{(\ell)}:=\alpha_{\widehat{n}_{\ell-1}+j}-\tfrac{1}{n_{\ell}}\big{(}\widehat{\alpha}_{\widehat{n}_{\ell}}-\widehat{\alpha}_{\widehat{n}_{\ell-1}}\big{)},

and

|α(j)|2:=(α1(j))2+(α2(j))2++(αnj(j))2.\big{|}\alpha^{(j)}\big{|}^{2}:=\big{(}\alpha_{1}^{(j)}\big{)}^{2}+\big{(}\alpha_{2}^{(j)}\big{)}^{2}+\cdots+\big{(}\alpha_{n_{j}}^{(j)}\big{)}^{2}.

If an1a\in\mathbb{R}^{n-1} then by Re(s)=a\operatorname{Re}(s)=-a we mean that Re(sj)=aj\operatorname{Re}(s_{j})=-a_{j} for each jn^1,,n^r1j\neq\widehat{n}_{1},\ldots,\widehat{n}_{r-1}.

With this notation in place, we can now state a generalization of Theorem 10.1.

Corollary 10.1.

Let n=n1++nrn=n_{1}+\cdots+n_{r} (r2r\geq 2), and set n^:=j=1nj\widehat{n}_{\ell}:=\sum\limits_{j=1}^{\ell}n_{j} as above. For each =1,,r1\ell=1,\ldots,r-1, let b()=(b1(),b2(),,bn1())b^{(\ell)}=(b_{1}^{(\ell)},b_{2}^{(\ell)},\ldots,b_{n_{\ell}-1}^{(\ell)}) with

bj()=α^i1+jn(α^n^α^n^1)for each 1jn1.b_{j}^{(\ell)}=\widehat{\alpha}_{i_{\ell-1}}+\tfrac{j}{n_{\ell}}\big{(}\widehat{\alpha}_{\widehat{n}_{\ell}}-\widehat{\alpha}_{\widehat{n}_{\ell-1}}\big{)}\qquad\mbox{for each $1\leq j\leq n_{\ell}-1$}.

Let δj0\delta_{j}\in{\mathbb{Z}}_{\geq 0} for j=1,,r1j=1,\ldots,r-1. There exist positive shifts L=(L(1),,L(r))L=(L^{(1)},\ldots,L^{(r)}) with L()=(L1(),,Ln1())(0)rL^{(\ell)}=(L_{1}^{(\ell)},\ldots,L_{n_{\ell}-1}^{(\ell)})\in\big{(}{\mathbb{Z}}_{\geq 0}\big{)}^{r} such that the iterated residue of W~n,α(s)\widetilde{W}_{n,\alpha}(s) at

sn^r1=α^n^r1δr1,,sn^1=α^n^1δ1,s_{\widehat{n}_{r-1}}=-\widehat{\alpha}_{\widehat{n}_{r-1}}-\delta_{r-1}\ ,\ \ldots\ ,\ s_{\widehat{n}_{1}}=-\widehat{\alpha}_{\widehat{n}_{1}}-\delta_{1},

is equal to a sum over all such shifts of

𝒫d(s,α)(=1rW~n,α()(s()+b()+L()))j=1r1K{1,2,,n^j+1}#(K{1,,n^j})n^j1#K=n^j((iKαi)α^n^jδj)δj11k<mri=1nkj=1nmΓ(αj(m)αi(k)+1nm(α^n^mα^n^m1)1nk(α^n^kα^n^k1)δm),\mathcal{P}_{d}(s,\alpha)\left(\prod_{\ell=1}^{r}\widetilde{W}_{n_{\ell},\alpha^{(\ell)}}(s^{(\ell)}+b^{(\ell)}+L^{(\ell)})\right)\prod_{j=1}^{r-1}\prod_{\begin{subarray}{c}K\subseteq\{1,2,\ldots,\widehat{n}_{j+1}\}\\ \#(K\cap\{1,\ldots,\widehat{n}_{j}\})\neq\widehat{n}_{j}-1\\ \#K=\widehat{n}_{j}\end{subarray}}\left(\Big{(}\sum_{i\in K}\alpha_{i}\Big{)}-\widehat{\alpha}_{\widehat{n}_{j}}-\delta_{j}\right)_{\delta_{j}}^{-1}\\ \prod_{1\leq k<m\leq r}\prod_{i=1}^{n_{k}}\prod_{j=1}^{n_{m}}\Gamma\bigg{(}\alpha_{j}^{(m)}-\alpha_{i}^{(k)}+\tfrac{1}{n_{m}}\big{(}\widehat{\alpha}_{\widehat{n}_{m}}-\widehat{\alpha}_{\widehat{n}_{m-1}}\big{)}-\tfrac{1}{n_{k}}\big{(}\widehat{\alpha}_{\widehat{n}_{k}}-\widehat{\alpha}_{\widehat{n}_{k-1}}\big{)}-\delta_{m}\bigg{)},

where

d=[=1r1δ((n^+1n^)2)]2|L|.d=\left[\sum_{\ell=1}^{r-1}\delta_{\ell}\left(\binom{\widehat{n}_{\ell+1}}{\widehat{n}_{\ell}}-2\right)\right]-2\lvert L\rvert.
Proof.

This follows easily by induction with the base case being Theorem 10.1. ∎

Remark 10.2.

Although it is possible to rewrite each of the terms (iKαi)α^n^δ\bigg{(}\sum\limits_{i\in K}\alpha_{i}\bigg{)}-\widehat{\alpha}_{\widehat{n}_{\ell}}-\delta_{\ell} appearing in the statement of Corollary 10.1 in terms of the variables α(j)\alpha^{(j)} and α^m(j)\widehat{\alpha}_{m}^{(j)} for various jj and mm, the exact description is unnecessary for our purposes.

10.3. Proof of Theorem 10.1

Proof.

As a first step, note that Proposition 8.6 implies that (10.2) follows from (10.3) and (10.4).

As shown in Section 8.5, the shifted pT,R(n)p_{T,R}^{(n)} term satisfies

|pT,R(n)(y,a)|\displaystyle\big{|}p_{T,R}^{(n)}(y,-a)\big{|} (j=1n1yjj(nj)22aj)T,R(n)(a).\displaystyle\ll\bigg{(}\prod_{j=1}^{n-1}y_{j}^{\frac{j(n-j)}{2}-2a_{j}}\bigg{)}\mathcal{I}_{T,R}^{(n)}(-a).

Combined with the bound from Theorem 9.2, this gives (10.3).

To complete the proof, we need to show that (10.4) holds. We do this in Section 10.5. Although this proof is valid for any r2r\geq 2, as a warmup, we first prove the special case r=2r=2 (i.e., the case of single residues) in Section 10.4. ∎

10.4. Bounds for single residue terms

In this section111Note that this section will be superseded by Section 10.5 which will prove the bound for any admissible CC with length(C)2\operatorname{length}(C)\geq 2. This section treats the case length(C)=2\operatorname{length}(C)=2. we bound pT,R(n)(y;a,δC)p_{T,R}^{(n)}(y;-a,\delta_{C}) in the case that of the composition C=(m,nm)C=(m,n-m). Since CC is a composition of length two, we may take (see Definition 8.3) δC=δ0\delta_{C}=\delta\in{\mathbb{Z}}_{\geq 0}.

Proof of (10.4) when r=2r=2.

Using Lemmas A.19, A.26 and A.28, we can rewrite

eα12+αn2T2/2R(n)(α)1jknΓR(αjαk)e^{\frac{\alpha_{1}^{2}+\cdots\alpha_{n}^{2}}{T^{2}/2}}\mathcal{F}_{R}^{(n)}(\alpha)\prod_{1\leq j\neq k\leq n}\Gamma_{R}(\alpha_{j}-\alpha_{k})

in terms of β\beta, γ\gamma and αn\alpha_{n}. Thus, together with Theorem 10.1, we see that Definition 8.3 in the case of a single residue term (i.e., r=2r=2) satisfies the bound

pT,R(n)(y;a,δC)\displaystyle p_{T,R}^{(n)}(y;-a,\delta_{C}) Re(α^m)=0ymm(nm)2+α^m+δenm(nm)α^m2T2/2β^m=0Re(β)=0e|β|2T2/2γ^nm=0Re(γ)=0e|γ|2T2/2\displaystyle\ll\hskip-4.0pt\int\limits_{\operatorname{Re}(\widehat{\alpha}_{m})=0}y_{m}^{\frac{m(n-m)}{2}+\widehat{\alpha}_{m}+\delta}\cdot e^{\frac{n}{m(n-m)}\frac{\widehat{\alpha}_{m}^{2}}{T^{2}/2}}\int\limits_{\begin{subarray}{c}\widehat{\beta}_{m}=0\\ \operatorname{Re}(\beta)=0\end{subarray}}e^{\frac{|\beta|^{2}}{T^{2}/2}}\int\limits_{\begin{subarray}{c}\widehat{\gamma}_{n-m}=0\\ \operatorname{Re}(\gamma)=0\end{subarray}}e^{\frac{|\gamma|^{2}}{T^{2}/2}}
(R(m)(β)1ijmΓR(βiβj))(R(nm)(γ)1ijnmΓR(γiγj))\displaystyle\hskip 18.0pt\cdot\left(\mathcal{F}_{R}^{(m)}(\beta)\cdot\prod_{1\leq i\neq j\leq m}\Gamma_{R}(\beta_{i}-\beta_{j})\right)\left(\mathcal{F}_{R}^{(n-m)}(\gamma)\cdot\prod_{1\leq i\neq j\leq n-m}\Gamma_{R}(\gamma_{i}-\gamma_{j})\right)
i=1mj=1nm(ΓR(βiγj+nα^mm(nm))ΓR(γjβinα^mm(nm))Γ(γjβinα^mm(nm)δ))\displaystyle\hskip-24.0pt\cdot\prod_{i=1}^{m}\prod_{j=1}^{n-m}\bigg{(}\Gamma_{R}\big{(}\beta_{i}-\gamma_{j}+\tfrac{n\widehat{\alpha}_{m}}{m(n-m)}\big{)}\Gamma_{R}\big{(}\gamma_{j}-\beta_{i}-\tfrac{n\widehat{\alpha}_{m}}{m(n-m)}\big{)}\Gamma\big{(}\gamma_{j}-\beta_{i}-\tfrac{n\widehat{\alpha}_{m}}{m(n-m)}-\delta\big{)}\bigg{)}
(jmRe(sj)=ajjmyjj(nj)2sj)𝒫R(D(n)D(m)D(nm))δ((nm)m(nm)1)(s,α)\displaystyle\hskip 12.0pt\cdot\Bigg{(}\prod_{j\neq m}\int\limits_{\begin{subarray}{c}\operatorname{Re}(s_{j})=-a_{j}\\ j\neq m\end{subarray}}y_{j}^{\frac{j(n-j)}{2}-s_{j}}\Bigg{)}\mathcal{P}_{R\big{(}D(n)-D(m)-D(n-m)\big{)}-\delta\left(\binom{n}{m}-m(n-m)-1\right)}(s,\alpha)
𝒫((nm)2)δ2|L|(s,α)W~m,β(s+L)W~nm,γ(s′′+L′′)dsdγdβdα^m.\displaystyle\hskip 38.0pt\cdot\mathcal{P}_{\left(\binom{n}{m}-2\right)\delta-2\lvert L\rvert}(s,\alpha)\cdot\widetilde{W}_{m,\beta}(s^{\prime}+L^{\prime})\widetilde{W}_{n-m,\gamma}(s^{\prime\prime}+L^{\prime\prime})\,ds\;d\gamma\;d\beta\;d\widehat{\alpha}_{m}.

In order to have the correct power of ymy_{m} we need to shift the line of integration in α^m\widehat{\alpha}_{m} to Re(α^m)=amδ\operatorname{Re}(\widehat{\alpha}_{m})=a_{m}-\delta. Note that by Lemma A.14, no poles are crossed in doing so, and by Lemma A.15, taking β=βiγj\beta=\beta_{i}-\gamma_{j} and z=nα^mm(nm)z=\frac{n\widehat{\alpha}_{m}}{m(n-m)}, we may replace the third to last line by

𝒫m(nm)Rn(amδ)m(nm)δ(s,α^m,β,γ).\mathcal{P}_{m(n-m)R-n(a_{m}-\delta)-m(n-m)\delta}(s,\widehat{\alpha}_{m},\beta,\gamma).

Let |β|2:=β12++βm2\lvert\beta\rvert^{2}:=\beta_{1}^{2}+\cdots+\beta_{m}^{2}, and define |γ|2\lvert\gamma\rvert^{2} similarly. Replacing the integral over α^m\widehat{\alpha}_{m} by Tε+1T^{\varepsilon+1} and factoring out the powers of yjy_{j}, we see that

|pT,R(n)(y;a,δC)|\displaystyle\Big{|}p_{T,R}^{(n)}(y;-a,\delta_{C})\Big{|} (j=1n1yjj(nj)2+aj)Tε+((nm)2)δ+R(D(n)D(m)D(nm))δ((nm)m(nm)1)\displaystyle\ll\bigg{(}\prod_{j=1}^{n-1}y_{j}^{\frac{j(n-j)}{2}+a_{j}}\bigg{)}\cdot T^{\varepsilon+\left(\binom{n}{m}-2\right)\delta+R\big{(}D(n)-D(m)-D(n-m)\big{)}-\delta\left(\binom{n}{m}-m(n-m)-1\right)}
T2|L|+m(nm)Rn(amδ)m(nm)δ+1β^m=0Re(β)=0e|β|2T2/2γ^nm=0Re(γ)=0e|γ|2T2/2\displaystyle\hskip 12.0pt\cdot T^{-2\lvert L\rvert+m(n-m)R-n(a_{m}-\delta)-m(n-m)\delta+1}\cdot\int\limits_{\begin{subarray}{c}\widehat{\beta}_{m}=0\\ \operatorname{Re}(\beta)=0\end{subarray}}e^{\frac{\lvert\beta\rvert^{2}}{T^{2}/2}}\int\limits_{\begin{subarray}{c}\widehat{\gamma}_{n-m}=0\\ \operatorname{Re}(\gamma)=0\end{subarray}}e^{\frac{\lvert\gamma\rvert^{2}}{T^{2}/2}}
(R(m)(β)1i,jkΓR(βiβj))(R(nm)(γ)1i,jnkΓR(γiγj))\displaystyle\hskip 30.0pt\cdot\left(\mathcal{F}_{R}^{(m)}(\beta)\cdot\prod_{1\leq i,j\leq k}\Gamma_{R}(\beta_{i}-\beta_{j})\right)\left(\mathcal{F}_{R}^{(n-m)}(\gamma)\cdot\prod_{1\leq i,j\leq n-k}\Gamma_{R}(\gamma_{i}-\gamma_{j})\right)
Re(sj)=aj1jn1jm|W~m,β(s+L)||W~nm,γ(s′′+L′′)|dsdγdβ.\displaystyle\hskip 72.0pt\cdot\hskip-6.0pt\int\limits_{\begin{subarray}{c}\operatorname{Re}(s_{j})=-a_{j}\\ 1\leq j\leq n-1\\ j\neq m\end{subarray}}\Big{\lvert}\widetilde{W}_{m,\beta}(s^{\prime}+L^{\prime})\Big{\rvert}\cdot\Big{\lvert}\widetilde{W}_{n-m,\gamma}(s^{\prime\prime}+L^{\prime\prime})\Big{\rvert}\,ds\;d\gamma\;d\beta.

Note that by Proposition 9.2 we may remove the dependence on the shift LL. Hence

|pT,R(n)(y;a,δC)|\displaystyle\Big{|}p_{T,R}^{(n)}(y;-a,\delta_{C})\Big{|} (j=1n1yjj(nj)2+aj)Tε+R(D(n)D(m)D(nm)+m(nm))+δ(n1)\displaystyle\ll\bigg{(}\prod_{j=1}^{n-1}y_{j}^{\frac{j(n-j)}{2}+a_{j}}\bigg{)}\cdot T^{\varepsilon+R\big{(}D(n)-D(m)-D(n-m)+m(n-m)\big{)}+\delta(n-1)}
Tnam+1β^m=0Re(β)=0eβ12++βm2T2/2γ^nm=0Re(γ)=0eγ12++γn2T2/2\displaystyle\hskip 36.0pt\cdot T^{-na_{m}+1}\int\limits_{\begin{subarray}{c}\widehat{\beta}_{m}=0\\ \operatorname{Re}(\beta)=0\end{subarray}}e^{\frac{\beta_{1}^{2}+\cdots+\beta_{m}^{2}}{T^{2}/2}}\int\limits_{\begin{subarray}{c}\widehat{\gamma}_{n-m}=0\\ \operatorname{Re}(\gamma)=0\end{subarray}}e^{\frac{\gamma_{1}^{2}+\cdots+\gamma_{n}^{2}}{T^{2}/2}}
(R(m)(β)1ijkΓR(βiβj))(R(nm)(γ)1ijnkΓR(γiγj))\displaystyle\hskip 0.0pt\cdot\left(\mathcal{F}_{R}^{(m)}(\beta)\cdot\prod_{1\leq i\neq j\leq k}\Gamma_{R}(\beta_{i}-\beta_{j})\right)\left(\mathcal{F}_{R}^{(n-m)}(\gamma)\cdot\prod_{1\leq i\neq j\leq n-k}\Gamma_{R}(\gamma_{i}-\gamma_{j})\right)
Re(sj)=aj1jn1jm|W~m,β(s)||W~nm,γ(s′′)|dsdγdβ.\displaystyle\hskip 72.0pt\cdot\hskip-6.0pt\int\limits_{\begin{subarray}{c}\operatorname{Re}(s_{j})=-a_{j}\\ 1\leq j\leq n-1\\ j\neq m\end{subarray}}\Big{\lvert}\widetilde{W}_{m,\beta}(s^{\prime})\Big{\rvert}\cdot\Big{\lvert}\widetilde{W}_{n-m,\gamma}(s^{\prime\prime})\Big{\rvert}\,ds\;d\gamma\;d\beta.

By (10.2),

sj=sjjm(α^mδ),andsj′′=sm+jnmjnm(α^mδ).s_{j}^{\prime}=s_{j}-\tfrac{j}{m}(\widehat{\alpha}_{m}-\delta),\quad\mbox{and}\quad s_{j}^{\prime\prime}=s_{m+j}-\tfrac{n-m-j}{n-m}(\widehat{\alpha}_{m}-\delta).

Thus the integrals in β\beta and γ\gamma above are essentially the product of T,R(m)(a)\mathcal{I}_{T,R}^{(m)}(-a^{\prime}) and T,R(nm)(a′′)\mathcal{I}_{T,R}^{(n-m)}(-a^{\prime\prime}). The only issue is that because, as seen in the fact that the variables ss^{\prime} and s′′s^{\prime\prime} are shifted, we have

aj=ajjm(amδ),andaj′′=am+jnmjnm(amδ).a_{j}^{\prime}=a_{j}-\tfrac{j}{m}(a_{m}-\delta),\quad\mbox{and}\quad a_{j}^{\prime\prime}=a_{m+j}-\tfrac{n-m-j}{n-m}(a_{m}-\delta).

Therefore, we can rewrite the previous formula as

|pT,R(n)(y;a,δC)|\displaystyle\Big{|}p_{T,R}^{(n)}(y;-a,\delta_{C})\Big{|} (j=1n1yjj(nj)2+aj)Tε+R(D(n)D(m)D(nm)+m(nm))\displaystyle\ll\bigg{(}\prod_{j=1}^{n-1}y_{j}^{\frac{j(n-j)}{2}+a_{j}}\bigg{)}\cdot T^{\varepsilon+R\big{(}D(n)-D(m)-D(n-m)+m(n-m)\big{)}}
Tδ(n1)nam+1T,R(m)(a)T,R(nm)(a′′).\displaystyle\hskip 36.0pt\cdot T^{\delta(n-1)-na_{m}+1}\cdot\mathcal{I}_{T,R}^{(m)}(-a^{\prime})\cdot\mathcal{I}_{T,R}^{(n-m)}(-a^{\prime\prime}).

By Theorem 9.2, we have

|pT,R(n)(y;a,δC)|\displaystyle\Big{|}p_{T,R}^{(n)}(y;-a,\delta_{C})\Big{|} (j=1n1yjj(nj)2+aj)Tε+R(D(n)D(m)D(nm)+m(nm))\displaystyle\ll\bigg{(}\prod_{j=1}^{n-1}y_{j}^{\frac{j(n-j)}{2}+a_{j}}\bigg{)}\cdot T^{\varepsilon+R\big{(}D(n)-D(m)-D(n-m)+m(n-m)\big{)}}
Tδ(n1)nam+1Tε+C(m)+R(D(m)+m(m1)2)j=1m1B(aj)\displaystyle\hskip 12.0pt\cdot T^{\delta(n-1)-na_{m}+1}\cdot T^{\varepsilon+C(m)+R\cdot\big{(}D(m)+\frac{m(m-1)}{2}\big{)}-\sum\limits_{j=1}^{m-1}B(a_{j}^{\prime})}
Tε+C(nm)+R(D(nm)+(nm)(nm1)2)j=1nm1B(aj′′),\displaystyle\hskip 12.0pt\cdot T^{\varepsilon+C(n-m)+R\cdot\big{(}D(n-m)+\frac{(n-m)(n-m-1)}{2}\big{)}-\sum\limits_{j=1}^{n-m-1}B(a_{j}^{\prime\prime})},

Recall that C(k)=(k+4)(k1)4C(k)=\frac{(k+4)(k-1)}{4}. Hence, using the elementary identity

C(m)+C(nm)=C(n)m(nm)21C(m)+C(n-m)=C(n)-\frac{m(n-m)}{2}-1

together with Lemma A.6,

|pT,R(n)(y;a,δC)|\displaystyle\Big{|}p_{T,R}^{(n)}(y;-a,\delta_{C})\Big{|} (j=1n1yjj(nj)2+aj)Tε+R(D(n)+m(nm)+m(m1)2+(nm)(nm1)2)\displaystyle\ll\bigg{(}\prod_{j=1}^{n-1}y_{j}^{\frac{j(n-j)}{2}+a_{j}}\bigg{)}\cdot T^{\varepsilon+R\big{(}D(n)+m(n-m)+\frac{m(m-1)}{2}+\frac{(n-m)(n-m-1)}{2}\big{)}}
Tδ(n1)+C(n)m(nm)2namj=1m1B(aj)+n22(amδ+1)+B(am)\displaystyle\hskip 12.0pt\cdot T^{\delta(n-1)+C(n)-\frac{m(n-m)}{2}-na_{m}-\sum\limits_{j=1}^{m-1}B(a_{j})+\frac{n-2}{2}(a_{m}-\delta+1)+B(a_{m})}
(j=1n1yjj(nj)2+aj)Tε+C(n)+R(D(n)+n(n1)2)j=1n1B(aj)\displaystyle\ll\bigg{(}\prod_{j=1}^{n-1}y_{j}^{\frac{j(n-j)}{2}+a_{j}}\bigg{)}\cdot T^{\varepsilon+C(n)+R\big{(}D(n)+\frac{n(n-1)}{2}\big{)}-\sum\limits_{j=1}^{n-1}B(a_{j})}
Tn22(δam+1)m(nm)2n(δam)+B(am)δ\displaystyle\hskip 48.0pt\cdot T^{\frac{n-2}{2}(\delta-a_{m}+1)-\frac{m(n-m)}{2}-n(\delta-a_{m})+B(a_{m})-\delta}

This gives the desired bound provided that the exponent of the final TT is negative. Using the facts that m(nm)2-\frac{m(n-m)}{2} is maximized when m=1m=1 or m=n1m=n-1 and B(am)am+12B(a_{m})\leq a_{m}+\frac{1}{2}, we see that the final exponent is

(10.1) n2(amδ)+n12m(nm)2\displaystyle-\frac{n}{2}\big{(}a_{m}-\delta\big{)}+\frac{n-1}{2}-\frac{m(n-m)}{2} n2(amδ),\displaystyle\leq\ -\frac{n}{2}\big{(}a_{m}-\delta\big{)},

as claimed. ∎

10.5. Bounds for (r1)(r-1)-fold residues

We consider a composition CC of nn of length r2r\geq 2 given by n=n1++nrn=n_{1}+\cdots+n_{r}. We may also write C=(n1,,nr)C=(n_{1},\ldots,n_{r}). Let n^=j=1nj\widehat{n}_{\ell}=\sum\limits_{j=1}^{\ell}n_{j} as usual.

As a final piece of notation, let β=(β1,,βr)\beta=(\beta_{1},\ldots,\beta_{r}) be defined via

βi:=α^n^iα^n^i1\beta_{i}:=\widehat{\alpha}_{\widehat{n}_{i}}-\widehat{\alpha}_{\widehat{n}_{i-1}}

Note that i=1rβi=0\sum\limits_{i=1}^{r}\beta_{i}=0 and more generally, defining β^m=i=1mβi\widehat{\beta}_{m}=\sum\limits_{i=1}^{m}\beta_{i}, α^n^i=β^i\widehat{\alpha}_{\widehat{n}_{i}}=\widehat{\beta}_{i}. Since (assuming that α^n=0\widehat{\alpha}_{n}=0) the Jacobians of the change of variables

α(α(1),α^n^1,α(2),α^n^2,,α^n^r1,α(r))\alpha\mapsto(\alpha^{(1)},\widehat{\alpha}_{\widehat{n}_{1}},\alpha^{(2)},\widehat{\alpha}_{\widehat{n}_{2}},\ldots,\widehat{\alpha}_{\widehat{n}_{r-1}},\alpha^{(r)})

and

(α^n^1,,α^n^r1)(β1,,βr1)(\widehat{\alpha}_{\widehat{n}_{1}},\ldots,\widehat{\alpha}_{\widehat{n}_{r-1}})\mapsto(\beta_{1},\ldots,\beta_{r-1})

are trivial, we see that (for β1++βr=0\beta_{1}+\cdots+\beta_{r}=0)

(10.1) dα=dβdα(1)dα(2)dα(r).d\alpha=d\beta\;d\alpha^{(1)}d\alpha^{(2)}\cdots d\alpha^{(r)}.
Proof of (10.4) when r2r\geq 2.

Note that

bj()=α^n^1+jn(α^n^α^n^1)=β^1+jnβfor each 1jn1.b_{j}^{(\ell)}=\widehat{\alpha}_{\widehat{n}_{\ell-1}}+\tfrac{j}{n_{\ell}}\big{(}\widehat{\alpha}_{\widehat{n}_{\ell}}-\widehat{\alpha}_{\widehat{n}_{\ell-1}}\big{)}=\widehat{\beta}_{\ell-1}+\tfrac{j}{n_{\ell}}\beta_{\ell}\qquad\mbox{for each $1\leq j\leq n_{\ell}-1$}.

Recall that by Definition 8.3,

pT,R(n)(y;a,δC)\displaystyle p_{T,R}^{(n)}(y;-a,\delta_{C}) :=α^n=0Re(α)=0eα12++αn2T2/2R(n)(α)(1jknΓR(αjαk))\displaystyle:=\hskip-6.0pt\int\limits_{\begin{subarray}{c}\widehat{\alpha}_{n}=0\\ \operatorname{Re}(\alpha)=0\end{subarray}}e^{\frac{\alpha_{1}^{2}+\cdots+\alpha_{n}^{2}}{T^{2}/2}}\cdot\mathcal{F}_{R}^{(n)}(\alpha)\bigg{(}\prod_{1\leq j\neq k\leq n}\hskip-4.0pt\Gamma_{R}\big{(}\alpha_{j}-\alpha_{k}\big{)}\bigg{)}\hskip-3.0pt
(i=1r1yn^in^i(nn^i)2+α^n^i+δi)Re(sj)=ajj{n^1,,n^r1}(j{n^1,,n^r1}yjj(nj)2sj)\displaystyle\hskip 12.0pt\cdot\bigg{(}\prod_{i=1}^{r-1}y_{\widehat{n}_{i}}^{\frac{\widehat{n}_{i}(n-\widehat{n}_{i})}{2}+\widehat{\alpha}_{\widehat{n}_{i}}+\delta_{i}}\bigg{)}\cdot\int\limits_{\begin{subarray}{c}\operatorname{Re}(s_{j})=-a_{j}\\ j\notin\{\widehat{n}_{1},\ldots,\widehat{n}_{r-1}\}\end{subarray}}\bigg{(}\prod_{j\notin\{\widehat{n}_{1},\ldots,\widehat{n}_{r-1}\}}y_{j}^{\frac{j(n-j)}{2}-s_{j}}\bigg{)}
Ressn^1=α^n^1δ1(Ressn^2=α^n^2δ2((Ressn^r1=α^n^r1δr1W~n,α(s))))dsdα.\displaystyle\hskip-24.0pt\cdot\underset{s_{\widehat{n}_{1}}=-\widehat{\alpha}_{\widehat{n}_{1}}-\delta_{1}}{\operatorname{Res}}\left(\underset{s_{\widehat{n}_{2}}=-\widehat{\alpha}_{\widehat{n}_{2}}-\delta_{2}}{\operatorname{Res}}\left(\cdots\left(\underset{s_{\widehat{n}_{r-1}}=-\widehat{\alpha}_{\widehat{n}_{r-1}}-\delta_{r-1}}{\operatorname{Res}}\widetilde{W}_{n,\alpha}(s)\right)\cdots\right)\right)\,ds\,d\alpha.

Using Remark A.29 and Corollary 10.1, we can bound |pT,R(n)(y;a,δC)|\big{\lvert}p_{T,R}^{(n)}(y;-a,\delta_{C})\big{\rvert} by a sum over certain shifts LL each of the form

β^r=0Re(β)=0e(β12n1++βr2nr)2T2(j=1r1yn^jn^j(nn^j)2+β^j+δjα^nj(j)=0Re(α(j))=0e|α(j)|2T2/2)\displaystyle\int\limits_{\begin{subarray}{c}\widehat{\beta}_{r}=0\\ \operatorname{Re}(\beta)=0\end{subarray}}e^{\big{(}\frac{\beta_{1}^{2}}{n_{1}}+\cdots+\frac{\beta_{r}^{2}}{n_{r}}\big{)}\frac{2}{T^{2}}}\cdot\Bigg{(}\prod_{j=1}^{r-1}\hskip 6.0pty_{\widehat{n}_{j}}^{\frac{\widehat{n}_{j}(n-\widehat{n}_{j})}{2}+\widehat{\beta}_{j}+\delta_{j}}\int\limits_{\begin{subarray}{c}\widehat{\alpha}^{(j)}_{n_{j}}=0\\ \operatorname{Re}(\alpha^{(j)})=0\end{subarray}}e^{\frac{\left\lvert\alpha^{(j)}\right\rvert^{2}}{T^{2}/2}}\Bigg{)}
𝒫d12|L|(α)Re(s)=a(j{n^1,,n^r1}yjj(nj)2sj)𝒫d2(s,α)\displaystyle\hskip 108.0pt\cdot\mathcal{P}_{d_{1}-2\lvert L\rvert}(\alpha)\cdot\int\limits_{\begin{subarray}{c}\operatorname{Re}(s)=-a\end{subarray}}\bigg{(}\prod_{j\notin\{\widehat{n}_{1},\ldots,\widehat{n}_{r-1}\}}y_{j}^{\frac{j(n-j)}{2}-s_{j}}\bigg{)}\cdot\mathcal{P}_{d_{2}}(s,\alpha)
1k<mri=1nkj=1nmΓ(αj(m)αi(k)+βmnmβknkδk)ϵ{±1}ΓR(ϵ(αj(m)αi(k)+βmnmβknk))\displaystyle\hskip 12.0pt\cdot\prod_{1\leq k<m\leq r}\prod_{i=1}^{n_{k}}\prod_{j=1}^{n_{m}}\Gamma\bigg{(}\alpha_{j}^{(m)}-\alpha_{i}^{(k)}+\tfrac{\beta_{m}}{n_{m}}-\tfrac{\beta_{k}}{n_{k}}-\delta_{k}\bigg{)}\prod_{\epsilon\in\{\pm 1\}}\Gamma_{R}\bigg{(}\epsilon\Big{(}\alpha_{j}^{(m)}-\alpha_{i}^{(k)}+\tfrac{\beta_{m}}{n_{m}}-\tfrac{\beta_{k}}{n_{k}}\Big{)}\bigg{)}
=1r(R(n)(α())(1jknΓR(αj()αk()))W~n,α()(s()+b()+L()))\displaystyle\hskip 90.0pt\cdot\prod_{\ell=1}^{r}\left(\mathcal{F}_{R}^{(n_{\ell})}\big{(}\alpha^{(\ell)}\big{)}\bigg{(}\prod_{1\leq j\neq k\leq n_{\ell}}\hskip-4.0pt\Gamma_{R}\big{(}\alpha_{j}^{(\ell)}-\alpha_{k}^{(\ell)}\big{)}\bigg{)}\widetilde{W}_{n_{\ell},\alpha^{(\ell)}}(s^{(\ell)}+b^{(\ell)}+L^{(\ell)})\right)
dsdα(1)dα(2)dα(r)dβ,\displaystyle\hskip 324.0ptds\ d\alpha^{(1)}d\alpha^{(2)}\cdots d\alpha^{(r)}\ d\beta,

where

d1==1r1δ((n^+1n^)2)d_{1}=\sum_{\ell=1}^{r-1}\delta_{\ell}\left(\binom{\widehat{n}_{\ell+1}}{\widehat{n}_{\ell}}-2\right)

and

d2=R(D(n)=1rD(n))=1r1[δ((n^+1n^)n+1n^1)]d_{2}=R\cdot\left(D(n)-\sum_{\ell=1}^{r}D(n_{\ell})\right)-\sum\limits_{\ell=1}^{r-1}\left[\delta_{\ell}\left(\binom{\widehat{n}_{\ell+1}}{\widehat{n}_{\ell}}-n_{\ell+1}\widehat{n}_{\ell}-1\right)\right]

are the degrees coming from Remark A.29 and Corollary 10.1, respectively, and b()b^{(\ell)} is as in Corollary 10.1. Note that, in addition to using the change of variables (10.1), we have used Lemma A.18 and Lemma A.20 to break up e2|α|2/T2e^{2\lvert\alpha\rvert^{2}/T^{2}} and rewrite the product of Γ(αjαk)\Gamma(\alpha_{j}-\alpha_{k}) in terms of α(1),,α(r)\alpha^{(1)},\ldots,\alpha^{(r)} and β\beta.

The next step is to shift the lines of integration in the variables βj\beta_{j} for j=1,,r1j=1,\ldots,r-1 (or, equivalently, β^j\widehat{\beta}_{j} for j=1,,r1j=1,\ldots,r-1) such that the real part of the exponent of each term yn^jy_{\widehat{n}_{j}} is n^j(nn^j)2+aj\frac{\widehat{n}_{j}(n-\widehat{n}_{j})}{2}+a_{j}. In particular, this implies that we must shift the line of integration of β^j\widehat{\beta}_{j} to

(10.2) Re(β^j)=an^jδjRe(βj)=Re(β^jβ^j1)=(an^jδj)(an^j1δj1).\operatorname{Re}(\widehat{\beta}_{j})=a_{\widehat{n}_{j}}-\delta_{j}\Longleftrightarrow\operatorname{Re}(\beta_{j})=\operatorname{Re}(\widehat{\beta}_{j}-\widehat{\beta}_{j-1})=(a_{\widehat{n}_{j}}-\delta_{j})-(a_{\widehat{n}_{j-1}}-\delta_{j-1}).

Provided that RR is sufficiently large, Lemma A.14 implies that this shift can be made without passing any poles. Moreover, Lemma A.15 implies that

(10.3) 1k<mri=1nkj=1nmΓ(αi(k)αj(m)+βknkβmnmδm)ϵ{±1}ΓR(ϵ(αi(k)αj(m)+βknkβmnm))1k<mri=1nkj=1nm(1+|Im(αi(k)αj(m)+βknkβmnm)|)RRe(βknkβmnm)δm\prod_{1\leq k<m\leq r}\prod_{i=1}^{n_{k}}\prod_{j=1}^{n_{m}}\Gamma\bigg{(}\alpha_{i}^{(k)}-\alpha_{j}^{(m)}+\tfrac{\beta_{k}}{n_{k}}-\tfrac{\beta_{m}}{n_{m}}-\delta_{m}\bigg{)}\prod_{\epsilon\in\{\pm 1\}}\Gamma_{R}\bigg{(}\epsilon\Big{(}\alpha_{i}^{(k)}-\alpha_{j}^{(m)}+\tfrac{\beta_{k}}{n_{k}}-\tfrac{\beta_{m}}{n_{m}}\Big{)}\bigg{)}\\ \asymp\prod_{1\leq k<m\leq r}\prod_{i=1}^{n_{k}}\prod_{j=1}^{n_{m}}\Big{(}1+\big{|}\operatorname{Im}\big{(}\alpha_{i}^{(k)}-\alpha_{j}^{(m)}+\tfrac{\beta_{k}}{n_{k}}-\tfrac{\beta_{m}}{n_{m}}\big{)}\big{|}\Big{)}^{R-\operatorname{Re}\big{(}\tfrac{\beta_{k}}{n_{k}}-\tfrac{\beta_{m}}{n_{m}}\big{)}-\delta_{m}}

Note that the presence of the term e(β12n1++βr2nr)2T2e^{\big{(}\frac{\beta_{1}^{2}}{n_{1}}+\cdots+\frac{\beta_{r}^{2}}{n_{r}}\big{)}\frac{2}{T^{2}}} implies that there is exponential decay for |Im(βj)|T1+ε\lvert\operatorname{Im}(\beta_{j})\rvert\gg T^{1+\varepsilon}. As we will see momentarily, besides the polynomial terms 𝒫d1(α)\mathcal{P}_{d_{1}}(\alpha), 𝒫d2(s,α)\mathcal{P}_{d_{2}}(s,\alpha) and (10.3), we just get a product of T,R(nj)(c(j))\mathcal{I}^{(n_{j})}_{T,R}(-c^{(j)}) for some (to be determined) values c()-c^{(\ell)}. The upshot is that all of these polynomials can be bounded by TT to the degree of the polynomial plus ε\varepsilon. Hence, we can bound the expression above by

(10.4) Tε+r1+d2|L|(j=1n1yjj(nj)2+aj)=1r(α^n()=0Re(α())=0e|α()|2T2/2R(n)(α())Re(s())=a()(1jknΓR(αj()αk()))|W~n,α()(s()+b()+L())|ds()dα()),T^{\varepsilon+r-1+d-2\lvert L\rvert}\cdot\left(\prod_{j=1}^{n-1}\hskip 6.0pty_{j}^{\frac{j(n-j)}{2}+a_{j}}\right)\cdot\prod_{\ell=1}^{r}\Bigg{(}\int\limits_{\begin{subarray}{c}\widehat{\alpha}^{(\ell)}_{n_{\ell}}=0\\ \operatorname{Re}(\alpha^{(\ell)})=0\end{subarray}}e^{\frac{\left\lvert\alpha^{(\ell)}\right\rvert^{2}}{T^{2}/2}}\cdot\mathcal{F}_{R}^{(n_{\ell})}\big{(}\alpha^{(\ell)}\big{)}\int\limits_{\begin{subarray}{c}\operatorname{Re}(s^{(\ell)})=-a^{(\ell)}\end{subarray}}\\ \cdot\bigg{(}\prod_{1\leq j\neq k\leq n_{\ell}}\hskip-4.0pt\Gamma_{R}\big{(}\alpha_{j}^{(\ell)}-\alpha_{k}^{(\ell)}\big{)}\bigg{)}\Big{|}\widetilde{W}_{n_{\ell},\alpha^{(\ell)}}(s^{(\ell)}+b^{(\ell)}+L^{(\ell)})\Big{|}\ ds^{(\ell)}\,d\alpha^{(\ell)}\Bigg{)},

where d=d1+d2+d3d=d_{1}+d_{2}+d_{3} with d1d_{1} and d2d_{2} as above and

d3=R=1rnn^k=1r1((nk+nk+1)(an^kδk)+δknk+1n^k)d_{3}=R\cdot\sum_{\ell=1}^{r}n_{\ell}\widehat{n}_{\ell}-\sum_{k=1}^{r-1}\Big{(}(n_{k}+n_{k+1})(a_{\widehat{n}_{k}}-\delta_{k})+\delta_{k}n_{k+1}\widehat{n}_{k}\Big{)}

is the bound coming from the terms described in (10.3), simplified using Lemma A.21. Combining everything, we find that dd equals

R(D(n)=1rD(n)+1k<mrnknm)k=1r1(δk+(nk+nk+1)(an^kδk)).R\cdot\bigg{(}\hskip-3.0ptD(n)-\sum_{\ell=1}^{r}D(n_{\ell})+\sum_{1\leq k<m\leq r}n_{k}n_{m}\hskip-3.0pt\bigg{)}-\sum_{k=1}^{r-1}\Big{(}\delta_{k}+(n_{k}+n_{k+1})(a_{\widehat{n}_{k}}-\delta_{k})\Big{)}.

Recall that the bound on pT,R(n)(y;a,δC)p_{T,R}^{(n)}(y;-a,\delta_{C}) is a sum of expressions of the form given in (10.4) for various shifts LL. However, using Proposition 9.2, we can remove the dependence on the shifts. Hence,

(10.5) |pT,R(n)(y;a,δC)|Tε+d+r1(j=1n1yjj(nj)2+aj)=1r(α^n()=0Re(α())=0e|α()|2T2/2R(n)(α())Re(s())=a()(1jknΓR(αj()αk()))|W~n,α()(s()+b())|ds()dα()),\big{\lvert}p_{T,R}^{(n)}(y;-a,\delta_{C})\big{\rvert}\ll T^{\varepsilon+d+r-1}\cdot\left(\prod_{j=1}^{n-1}\hskip 6.0pty_{j}^{\frac{j(n-j)}{2}+a_{j}}\right)\cdot\prod_{\ell=1}^{r}\Bigg{(}\int\limits_{\begin{subarray}{c}\widehat{\alpha}^{(\ell)}_{n_{\ell}}=0\\ \operatorname{Re}(\alpha^{(\ell)})=0\end{subarray}}e^{\frac{\left\lvert\alpha^{(\ell)}\right\rvert^{2}}{T^{2}/2}}\\ \cdot\mathcal{F}_{R}^{(n_{\ell})}\big{(}\alpha^{(\ell)}\big{)}\hskip-6.0pt\int\limits_{\begin{subarray}{c}\operatorname{Re}(s^{(\ell)})=-a^{(\ell)}\end{subarray}}\bigg{(}\prod_{1\leq j\neq k\leq n_{\ell}}\hskip-6.0pt\Gamma_{R}\big{(}\alpha_{j}^{(\ell)}-\alpha_{k}^{(\ell)}\big{)}\bigg{)}\Big{|}\widetilde{W}_{n_{\ell},\alpha^{(\ell)}}(s^{(\ell)}+b^{(\ell)})\Big{|}\ ds^{(\ell)}\,d\alpha^{(\ell)}\Bigg{)},

Hence, setting c()=a()Re(b()),c^{(\ell)}=a^{(\ell)}-\operatorname{Re}\big{(}b^{(\ell)}\big{)}, where

b()=(b1(),,bn^j()),bj()=β^1+jnβ,b^{(\ell)}=(b_{1}^{(\ell)},\ldots,b_{\widehat{n}_{j}}^{(\ell)}),\qquad b_{j}^{(\ell)}=\widehat{\beta}_{\ell-1}+\tfrac{j}{n_{\ell}}\beta_{\ell},

we find that

|pT,R(n)(y;a,δC)|(j=1n1yjj(nj)2+aj)Tε+r1+d=1rT,R()(c()).\displaystyle\big{\lvert}p_{T,R}^{(n)}(y;-a,\delta_{C})\big{\rvert}\ll\left(\prod_{j=1}^{n-1}\hskip 6.0pty_{j}^{\frac{j(n-j)}{2}+a_{j}}\right)\cdot T^{\varepsilon+r-1+d}\cdot\prod_{\ell=1}^{r}\mathcal{I}_{T,R}^{(\ell)}(-c^{(\ell)}).

Let C(m):=(m+4)(m1)4C(m):=\frac{(m+4)(m-1)}{4}. We now now apply Theorem 9.2 to each T,R(n)\mathcal{I}_{T,R}^{(n_{\ell})} to obtain

|pT,R(n)(y;a,δC)|Tε+r1+d+=1r(R(D(n)+n(n1)2)+C(n)k=1n1B(ck()))j=1n1yjj(nj)2+aj.\displaystyle\big{\lvert}p_{T,R}^{(n)}(y;-a,\delta_{C})\big{\rvert}\ll T^{\varepsilon+r-1+d+\sum\limits_{\ell=1}^{r}\Big{(}R\big{(}D(n_{\ell})+\frac{n_{\ell}(n_{\ell}-1)}{2}\big{)}+C(n_{\ell})-\sum\limits_{k=1}^{n_{\ell}-1}B(c_{k}^{(\ell)})\Big{)}}\cdot\prod_{j=1}^{n-1}\hskip 6.0pty_{j}^{\frac{j(n-j)}{2}+a_{j}}.

Now we generalize the proof of Lemma A.6, keeping in mind that a<B(a)<a+12a<B(a)<a+\frac{1}{2}, to simplify the expression

=1rj=1n1B(cj())\displaystyle\sum_{\ell=1}^{r}\sum_{j=1}^{n_{\ell}-1}B(c_{j}^{(\ell)}) =1rj=1n1(an^+jRe(β^1)jnRe(β))\displaystyle\geq\sum_{\ell=1}^{r}\sum_{j=1}^{n_{\ell}-1}\big{(}a_{\widehat{n}_{\ell}+j}-\operatorname{Re}(\widehat{\beta}_{\ell-1})-\tfrac{j}{n_{\ell}}\operatorname{Re}(\beta_{\ell})\big{)}
=(j=1n1aj)(k=1r1an^k)=1r[(n1)Re(β^1)+n12Re(β)]\displaystyle=\left(\sum_{j=1}^{n-1}a_{j}\right)-\left(\sum_{k=1}^{r-1}a_{\widehat{n}_{k}}\right)-\sum_{\ell=1}^{r}\Big{[}(n_{\ell}-1)\operatorname{Re}(\widehat{\beta}_{\ell-1})+\tfrac{n_{\ell}-1}{2}\operatorname{Re}(\beta_{\ell})\Big{]}
j=1n1(B(aj)12)k=1r1an^k=1r[(n1)Re(β^12β)]\displaystyle\geq\sum_{j=1}^{n-1}\Big{(}B(a_{j})-\tfrac{1}{2}\Big{)}-\sum_{k=1}^{r-1}a_{\widehat{n}_{k}}-\sum_{\ell=1}^{r}\Big{[}(n_{\ell}-1)\operatorname{Re}\left(\widehat{\beta}_{\ell}-\tfrac{1}{2}\beta_{\ell}\right)\Big{]}
=n12+j=1n1B(aj)k=1r1an^k=1r[(n1)(A12(AA1)]\displaystyle=-\tfrac{n-1}{2}+\sum_{j=1}^{n-1}B(a_{j})-\sum_{k=1}^{r-1}a_{\widehat{n}_{k}}-\sum_{\ell=1}^{r}\Big{[}(n_{\ell}-1)\left(A_{\ell}-\tfrac{1}{2}(A_{\ell}-A_{\ell-1}\right)\Big{]}
=n12+j=1n1B(aj)k=1r1an^k12=1r[(n1)(A+A1)].\displaystyle=-\tfrac{n-1}{2}+\sum_{j=1}^{n-1}B(a_{j})-\sum_{k=1}^{r-1}a_{\widehat{n}_{k}}-\frac{1}{2}\sum_{\ell=1}^{r}\Big{[}(n_{\ell}-1)\left(A_{\ell}+A_{\ell-1}\right)\Big{]}.

Next, we write the sum over \ell as

=1r[(n1)(A+A1)]\displaystyle\sum_{\ell=1}^{r}\Big{[}(n_{\ell}-1)\left(A_{\ell}+A_{\ell-1}\right)\Big{]} ==1r(n1)A+=1r(n1)A1\displaystyle=\sum_{\ell=1}^{r}(n_{\ell}-1)A_{\ell}+\sum_{\ell=1}^{r}(n_{\ell}-1)A_{\ell-1}
==1r(n1)A+=0r1(n+11)A\displaystyle=\sum_{\ell=1}^{r}(n_{\ell}-1)A_{\ell}+\sum_{\ell=0}^{r-1}(n_{\ell+1}-1)A_{\ell}
=(n11)A0+(nr1)Ar+=1r1(n+n+12)A\displaystyle=(n_{1}-1)A_{0}+(n_{r}-1)A_{r}+\sum_{\ell=1}^{r-1}(n_{\ell}+n_{\ell+1}-2)A_{\ell}
=k=1r1(nk+nk+12)(an^kδk)\displaystyle=\sum_{k=1}^{r-1}(n_{k}+n_{k+1}-2)(a_{\widehat{n}_{k}}-\delta_{k})

We plug this back in to get

=1rj=1n1B(cj())n12j=1n1B(a)+k=1r1an^k+12k=1r1(nk+nk+12)(an^kδk),\displaystyle-\sum_{\ell=1}^{r}\sum_{j=1}^{n_{\ell}-1}B(c_{j}^{(\ell)})\leq\tfrac{n-1}{2}-\sum_{j=1}^{n-1}B(a_{\ell})+\sum_{k=1}^{r-1}a_{\widehat{n}_{k}}+\frac{1}{2}\sum_{k=1}^{r-1}(n_{k}+n_{k+1}-2)(a_{\widehat{n}_{k}}-\delta_{k}),

from which it follows that the exponent of TT in the bound for |pT,R(n)(y;a,δC)|\big{\lvert}p_{T,R}^{(n)}(y;-a,\delta_{C})\big{\rvert} above is

ε+r1+d+=1r(R(D(n)+n(n1)2)+C(n))k=1n1B(ck()))\displaystyle\varepsilon+r-1+d+\sum\limits_{\ell=1}^{r}\Big{(}R\big{(}D(n_{\ell})+\tfrac{n_{\ell}(n_{\ell}-1)}{2}\big{)}+C(n_{\ell})\Big{)}-\sum\limits_{k=1}^{n_{\ell}-1}B(c_{k}^{(\ell)})\Big{)}
=ε+d+R(D(n)+n(n1)2)+C(n)j=1n1B(aj),\displaystyle\qquad=\varepsilon+d^{\prime}+R\Big{(}D(n)+\tfrac{n(n-1)}{2}\Big{)}+C(n)-\sum_{j=1}^{n-1}B(a_{j}),

where

d\displaystyle d^{\prime} =r1+d′′+n12+12k=1r1(nk+nk+12)(an^kδk)C(n)+=1rC(n)+k=1r1an^k\displaystyle=r-1+d^{\prime\prime}+\tfrac{n-1}{2}+\frac{1}{2}\sum_{k=1}^{r-1}(n_{k}+n_{k+1}-2)(a_{\widehat{n}_{k}}-\delta_{k})-C(n)+\sum_{\ell=1}^{r}C(n_{\ell})+\sum_{k=1}^{r-1}a_{\widehat{n}_{k}}
=d′′+n12+12k=1r1(nk+nk+12)(an^kδk)+k=1r1an^k121k<mrnknm,\displaystyle=d^{\prime\prime}+\tfrac{n-1}{2}+\frac{1}{2}\sum_{k=1}^{r-1}(n_{k}+n_{k+1}-2)(a_{\widehat{n}_{k}}-\delta_{k})+\sum_{k=1}^{r-1}a_{\widehat{n}_{k}}-\tfrac{1}{2}\sum_{1\leq k<m\leq r}n_{k}n_{m},

and

d′′\displaystyle d^{\prime\prime} =dR(D(n)=1rD(n)+1k<mrnknm)\displaystyle=d-R\cdot\bigg{(}\hskip-3.0ptD(n)-\sum_{\ell=1}^{r}D(n_{\ell})+\sum_{1\leq k<m\leq r}n_{k}n_{m}\hskip-3.0pt\bigg{)} =k=1r1(δk+(nk+nk+1)(an^kδk)).\displaystyle=-\sum_{k=1}^{r-1}\Big{(}\delta_{k}+(n_{k}+n_{k+1})(a_{\widehat{n}_{k}}-\delta_{k})\Big{)}.

Hence,

d\displaystyle d^{\prime} =n12k=1r1(δk+(nk+nk+1)(an^kδk))+\displaystyle=\frac{n-1}{2}-\sum_{k=1}^{r-1}\Big{(}\delta_{k}+(n_{k}+n_{k+1})(a_{\widehat{n}_{k}}-\delta_{k})\Big{)}+
+12k=1r1(nk+nk+12)(an^kδk)+k=1r1an^k121k<mrnknm\displaystyle\qquad+\frac{1}{2}\sum_{k=1}^{r-1}(n_{k}+n_{k+1}-2)(a_{\widehat{n}_{k}}-\delta_{k})+\sum_{k=1}^{r-1}a_{\widehat{n}_{k}}-\frac{1}{2}\sum_{1\leq k<m\leq r}n_{k}n_{m}
=n12k=1r1(nk+nk+1)(an^kδk)+12k=1r1(nk+nk+1)(an^kδk)121k<mrnknm\displaystyle=\frac{n-1}{2}-\sum_{k=1}^{r-1}(n_{k}+n_{k+1})(a_{\widehat{n}_{k}}-\delta_{k})+\frac{1}{2}\sum_{k=1}^{r-1}(n_{k}+n_{k+1})(a_{\widehat{n}_{k}}-\delta_{k})-\frac{1}{2}\sum_{1\leq k<m\leq r}n_{k}n_{m}
=12(n1k=1r1(nk+nk+1)(an^kδk)1k<mrnknm)\displaystyle=\frac{1}{2}\left(n-1-\sum_{k=1}^{r-1}(n_{k}+n_{k+1})(a_{\widehat{n}_{k}}-\delta_{k})-\sum_{1\leq k<m\leq r}n_{k}n_{m}\right)

Note that if r=2r=2 and n1=mn_{1}=m and δ1=δ\delta_{1}=\delta, then this expression becomes

n12n2(amδ)m(nm)2,\frac{n-1}{2}-\frac{n}{2}(a_{m}-\delta)-\frac{m(n-m)}{2},

which agrees with (10.1).

Therefore, to complete the proof, we need only show that n11k<mrnknm0.n-1-\sum\limits_{1\leq k<m\leq r}n_{k}n_{m}\leq 0. Indeed,

n11k<mrnknm=n1k=1r1m=k+1rnknm=n1k=1r1nk(nn^k)n1n1(nn1)0,n-1-\sum_{1\leq k<m\leq r}n_{k}n_{m}=n-1-\sum_{k=1}^{r-1}\sum_{m=k+1}^{r}n_{k}n_{m}=n-1-\sum_{k=1}^{r-1}n_{k}(n-\widehat{n}_{k})\leq n-1-n_{1}(n-n_{1})\leq 0,

(with the final inequality being equality if and only if n1=1n_{1}=1 or n1=n1n_{1}=n-1), as desired. ∎

Remark 10.6.

A critical step in the proof of (10.4) (either in the case of single residues, as is proved in Section 10.4 or higher order residues, as in Section 10.5) is to shift the lines of integration in the variables α^m\widehat{\alpha}_{m} or β^j\widehat{\beta}_{j}. A feature of this work that is quite different from the case of GL(4)\operatorname{GL}(4) as proved in [GSW21], is that no poles are crossed when making these shifts. This represents a major simplification. Recall from the discussion of Section 8.4 that in the case of n=4n=4 there are two fundamentally different types of single residues, two different types of double residues and a triple residue. As it turned out, when making the additional shift for each of the single and double residues, one ends up with five additional residue terms. Taken all together, it was necessary to complete the analysis of writing down explicitly what the residues are in terms of gamma function s, finding the exponential zero set, applying Stirling’s formula and then obtaining a bound for ten(!) separate residues integrals. All of this was in addition to performing these steps for the shifted pT,R(4)p_{T,R}^{(4)} term.

Appendix A Auxiliary results

In an effort to avoid obstructing the flow of the argument in the main body of this paper, we will include here the many technical results that are used throughout. We remind the reader that the notational conventions that are used throughout the paper and this appendix are given in Section 2.1.

Lemma A.1.

Suppose that w=w(n1,n2,,nr)w=w_{(n_{1},n_{2},\ldots,n_{r})} for some composition n=n1++n2n=n_{1}+\cdots+n_{2} with r2r\geq 2. Then, if y=(y1,,yn1)y=(y_{1},\ldots,y_{n-1}), wyw1w\,y\,w^{-1} is equal to

(ynn^1+1,ynn^1+2,,yn1n11 terms,(k=nn^2n1yk)1,,(k=nn^inn^i21yk)1,ynn^i+1,ynn^i+2,,ynn^i11ni1 terms,(k=nn^i+1nn^i11yk)1,,(k=1nn^s21yk)1,ynn^1+1,ynn^1+2,,yn1nr1 terms)\Bigg{(}\underbrace{y_{n-\widehat{n}_{1}+1}\,,\,y_{n-\widehat{n}_{1}+2}\,,\,\ldots\,,\,y_{n-1}}_{\mbox{\scriptsize{$n_{1}-1$ terms}}}\,,\,\left(\prod_{k=n-\widehat{n}_{2}}^{n-1}y_{k}\right)^{-1}\,,\,\ldots\\ \ldots\,,\,\left(\prod_{k=n-\widehat{n}_{i}}^{n-\widehat{n}_{i-2}-1}y_{k}\right)^{-1}\,,\,\underbrace{y_{n-\widehat{n}_{i}+1}\,,\,y_{n-\widehat{n}_{i}+2}\,,\,\ldots\,,\,y_{n-\widehat{n}_{i-1}-1}}_{\mbox{\scriptsize{$n_{i}-1$ terms}}}\,,\,\left(\prod_{k=n-\widehat{n}_{i+1}}^{n-\widehat{n}_{i-1}-1}y_{k}\right)^{-1}\,,\,\ldots\\ \ldots\,,\,\left(\prod_{k=1}^{n-\widehat{n}_{s-2}-1}y_{k}\right)^{-1}\,,\,\underbrace{y_{n-\widehat{n}_{1}+1}\,,\,y_{n-\widehat{n}_{1}+2}\,,\,\ldots\,,\,y_{n-1}}_{\mbox{\scriptsize{$n_{r}-1$ terms}}}\Bigg{)}

In particular,

wyw1ak=i=1rj=1niynn^i+jan^i1+an^i1+jan^i.\big{\lVert}wyw^{-1}\big{\rVert}^{a_{k}}=\prod_{i=1}^{r}\prod_{j=1}^{n_{i}}y_{n-\widehat{n}_{i}+j}^{-a_{\widehat{n}_{i-1}}+a_{\widehat{n}_{i-1}+j}-a_{\widehat{n}_{i}}}.
Proof.

Let w=w(n1,n2,,nr)w=w_{(n_{1},n_{2},\ldots,n_{r})} as above. In order to carefully analyze y=wyw1y^{\prime}=w\,y\,w^{-1}, we define xi:=j=1iyj.x_{i}:=\prod\limits_{j=1}^{i}y_{j}. This notation implies that y=diag(xn1,xn2,,x1,1)y=\operatorname{diag}(x_{n-1},x_{n-2},\ldots,x_{1},1). Now, let us think of the matrix yy as a block diagonal of the form y=diag(A1,A2,,Ar)y=\operatorname{diag}(A_{1},A_{2},\ldots,A_{r}) where

Ai=diag(xnn^i11,xnn^i12,,xnn^i1ni)GL(ni,).A_{i}=\operatorname{diag}(x_{n-\widehat{n}_{i-1}-1},x_{n-\widehat{n}_{i-1}-2},\ldots,x_{n-\widehat{n}_{i-1}-n_{i}})\in\operatorname{GL}(n_{i},\mathbb{R}).

Thus,

y=wyw1=diag(Ar,Ar1,,A1)=xnn1diag(Br,Br1,,B1).y^{\prime}=w\,y\,w^{-1}=\operatorname{diag}(A_{r},A_{r-1},\ldots,A_{1})=x_{n-n_{1}}\operatorname{diag}(B_{r},B_{r-1},\ldots,B_{1}).

Let 1ir1\leq i\leq r and 0jni10\leq j\leq n_{i}-1 and set

zn^i1+j:=xnn^i+jxnn1.z_{\widehat{n}_{i-1}+j}:=\frac{x_{n-\widehat{n}_{i}+j}}{x_{n-n_{1}}}.

Then (y1,y2,,yn1)(y^{\prime}_{1},y^{\prime}_{2},\ldots,y^{\prime}_{n-1}), the Iwasawa yy-variables of yy^{\prime}, satisfy yi=zi/zi1y_{i}^{\prime}=z_{i}/z_{i-1}. For j0j\neq 0, therefore, we see that

yn^i1+j=xnn^i+jxnn^i+j1=k=1nn^i+jyk=1nn^i+j1y=ynn^i+j,y^{\prime}_{\widehat{n}_{i-1}+j}=\frac{x_{n-\widehat{n}_{i}+j}}{x_{n-\widehat{n}_{i}+j-1}}=\frac{\prod\limits_{k=1}^{n-\widehat{n}_{i}+j}y_{k}}{\prod\limits_{\ell=1}^{n-\widehat{n}_{i}+j-1}y_{\ell}}=y_{n-\widehat{n}_{i}+j},

and for j=0j=0,

yn^i=xnn^i+1xnn^i+11=xnn^i+1xnn^i+ni1=k=1nn^i+1yk=1nn^i11y=(k=1ni+ni+11ynn^i+1+k)1,y^{\prime}_{\widehat{n}_{i}}=\frac{x_{n-\widehat{n}_{i+1}}}{x_{n-\widehat{n}_{i+1}-1}}=\frac{x_{n-\widehat{n}_{i+1}}}{x_{n-\widehat{n}_{i}+n_{i}-1}}=\frac{\prod\limits_{k=1}^{n-\widehat{n}_{i+1}}y_{k}}{\prod\limits_{\ell=1}^{n-\widehat{n}_{i-1}-1}y_{\ell}}=\left(\prod_{k=1}^{n_{i}+n_{i+1}-1}y_{n-\widehat{n}_{i+1}+k}\right)^{-1},

from which the statement of the lemma follows directly. ∎

Definition A.2.

We say that α=(α1,,αn)n\alpha=(\alpha_{1},\ldots,\alpha_{n})\in{\mathbb{C}}^{n} is in jj-general position if the set

{kJαk|J{1,,n},#J=j}\left\{\left.\sum_{k\in J}\alpha_{k}\,\right|\,J\subseteq\{1,\ldots,n\},\,\#J=j\right\}

consists of (nj)\binom{n}{j} distinct elements. We say that α\alpha is in general position if it is in jj-general position for each j=1,,n1j=1,\ldots,n-1.

Lemma A.3.

Suppose that there exists ε>0\varepsilon>0 such that for each j=1,,n1j=1,\ldots,n-1, the real part of sjs_{j} is bounded by at least ε\varepsilon from any integer. Assume that α\alpha is in jj-general position, Re(αi)=0\operatorname{Re}(\alpha_{i})=0 for each i=1,,n1i=1,\ldots,n-1 , and rj0r_{j}\in{\mathbb{Z}}_{\geq 0}. Assume that

Im(α1)Im(α2)Im(αn),\operatorname{Im}(\alpha_{1})\geq\operatorname{Im}(\alpha_{2})\geq\cdots\geq\operatorname{Im}(\alpha_{n}),

and let Ij=[Im(α1++αj),Im(αn++αnj+1)]I_{j}=[-\operatorname{Im}(\alpha_{1}+\cdots+\alpha_{j}),-\operatorname{Im}(\alpha_{n}+\cdots+\alpha_{n-j+1})]. If rj2r_{j}\geq 2, then

Re(sj)=σjIm(sj)IjJ{1,,n}#J=j|sj+kJαk|rjdsjL{1,,n}#L=jK{1,,n}#K=jKL(1+|LαkKαk|)rj.\displaystyle\int\limits_{\begin{subarray}{c}\operatorname{Re}(s_{j})=\sigma_{j}\\ \operatorname{Im}(s_{j})\in I_{j}\end{subarray}}\hskip-3.0pt\prod_{\begin{subarray}{c}J\subseteq\{1,\ldots,n\}\\ \#J=j\end{subarray}}\hskip-3.0pt\Big{|}s_{j}+\sum_{k\in J}\alpha_{k}\Big{|}^{-r_{j}}\,ds_{j}\ll\hskip-2.0pt\sum_{\begin{subarray}{c}L\subseteq\{1,\ldots,n\}\\ \#L=j\end{subarray}}\prod_{\begin{subarray}{c}K\subseteq\{1,\ldots,n\}\\ \#K=j\\ K\neq L\end{subarray}}\left(1+\Big{|}\sum_{\ell\in L}\alpha_{\ell}-\sum_{k\in K}\alpha_{k}\Big{|}\right)^{-r_{j}}.

If rj=1r_{j}=1 there is an extra power of ε\varepsilon in the exponent (in which case the implicit constant will depend on ε\varepsilon), and if rj=0r_{j}=0, the integral is bounded by

(1+k=1jαk=1jαn+1).\Big{(}1+\sum_{k=1}^{j}\alpha_{k}-\sum_{\ell=1}^{j}\alpha_{n+1-\ell}\Big{)}.
Remark A.4.

The implicit \ll–constant depends on σj\sigma_{j}, but in applications this will always be bounded.

Proof.

The bound in the case of rj=0r_{j}=0 is obvious, so we may assume henceforth that rj1r_{j}\geq 1. Consider the set

𝒜j:={kJαk|J{1,,n},#J=j}.\mathcal{A}_{j}:=\left\{\left.\sum_{k\in J}\alpha_{k}\,\right|\,J\subseteq\{1,\ldots,n\},\,\#J=j\right\}.

For a fixed choice α\alpha in jj-general position, let A1A_{1} be the element of 𝒜j\mathcal{A}_{j} that has the greatest imaginary part, A2A_{2} the next greatest imaginary part and so on. Hence Im(A1)<Im(A2)<<Im(A(nj))-\operatorname{Im}(A_{1})<-\operatorname{Im}(A_{2})<\cdots<-\operatorname{Im}(A_{\binom{n}{j}}).

Write sj=σj+itjs_{j}=\sigma_{j}+it_{j}. Note that Ij=[Im(A1),Im(A(nj))]I_{j}=[-\operatorname{Im}(A_{1}),-\operatorname{Im}(A_{\binom{n}{j}})]. Upon applying Lemma A.3 from [GSW21], one obtains the bound

IjJ{1,,n}#J=j|sj+kJαk|rjdsj(1+Im(A1)Im(A(nj)))εk=1(nj)1(1+Im(AkAk+1))rj.\displaystyle\int\limits_{I_{j}}\prod_{\begin{subarray}{c}J\subseteq\{1,\ldots,n\}\\ \#J=j\end{subarray}}\Big{|}s_{j}+\sum_{k\in J}\alpha_{k}\Big{|}^{-r_{j}}\,ds_{j}\ll\Big{(}1+\operatorname{Im}(A_{1})-\operatorname{Im}(A_{\binom{n}{j}})\big{)}^{\varepsilon}\hskip-2.0pt\prod_{k=1}^{\binom{n}{j}-1}\Big{(}1+\operatorname{Im}(A_{k}-A_{k+1})\Big{)}^{-r_{j}}.

This is one of the possible summands on the right hand side of the statement of the lemma. Hence, regardless of the specific ordering which may arise for the given choice of α\alpha, the claim follows. ∎

Lemma A.5.

Let aa\in\mathbb{R}. Then

max{0,2(aa)1}a{a if a(a12,a]a2(aa) if a(a,a+12].\max\{0,2(\lceil a\rceil-a)-1\}-\lceil a\rceil\leq\begin{cases}-\lceil a\rceil&\mbox{ if }a\in{(\lceil a\rceil-\frac{1}{2},\lceil a\rceil]}\\ -\lfloor a\rfloor-2\big{(}a-\lfloor a\rfloor\big{)}&\mbox{ if }a\in(\lfloor a\rfloor,\lfloor a\rfloor+\frac{1}{2}].\end{cases}
Proof.

First, let us assume that a(a12,a]a\in{(\lceil a\rceil-\frac{1}{2},\lceil a\rceil]}. Then aa<12\lceil a\rceil-a<\frac{1}{2}, hence

max{0,2(aa)1}a=a.\max\{0,2(\lceil a\rceil-a)-1\}-\lceil a\rceil=-\lceil a\rceil.

On the other hand, assuming that a(a,a+12]a\in(\lfloor a\rfloor,\lfloor a\rfloor+\frac{1}{2}], we see that

max{0,2(aa)1}a\displaystyle\max\{0,2(\lceil a\rceil-a)-1\}-\lceil a\rceil =a2a1=a2a=a2(aa),\displaystyle=\lceil a\rceil-2a-1=\lfloor a\rfloor-2a=-\lfloor a\rfloor-2\big{(}a-\lfloor a\rfloor\big{)},

as claimed. ∎

Lemma A.6.

Suppose that a1,,an>0a_{1},\ldots,a_{n}\in\mathbb{R}_{>0}. Let

B(a):={0 if a<0a+2(aa) if 0<a+ε<aa+12,a if 12<a12a<aε.B(a):=\begin{cases}0&\mbox{ if }a<0\\ \lfloor a\rfloor+2(a-\lfloor a\rfloor)&\mbox{ if }0<\lfloor a\rfloor+\varepsilon<a\leq\lfloor a\rfloor+\frac{1}{2},\\ \lceil a\rceil&\mbox{ if }\frac{1}{2}<\lceil a\rceil-\frac{1}{2}\leq a<\lceil a\rceil-\varepsilon.\end{cases}

Then for any δm0\delta_{m}\in{\mathbb{Z}}_{\geq 0} with 0<amδm0<a_{m}-\delta_{m},

j=1m1B(ajjm(amδm))+j=1nm1B(am+jnmjnm(amδm))(j=1n1B(aj))n22(amδm+1)B(am).\phantom{xxx}\sum_{j=1}^{m-1}B\left(a_{j}-\tfrac{j}{m}(a_{m}-\delta_{m})\right)+\sum_{j=1}^{n-m-1}B\left(a_{m+j}-\tfrac{n-m-j}{n-m}(a_{m}-\delta_{m})\right)\\ \geq\left(\sum_{j=1}^{n-1}B(a_{j})\right)-\tfrac{n-2}{2}(a_{m}-\delta_{m}+1)-B(a_{m}).\phantom{xxx}
Proof.

We consider first the case of r12aj<rr-\frac{1}{2}\leq a_{j}<r for some rr\in{\mathbb{Z}} and all j=1,2,,n1j=1,2,\ldots,n-1. For any aa\in\mathbb{R}, note that

(A.7) aB(a)a+12,a\leq B(a)\leq a+\tfrac{1}{2},

hence

j=1m1B(ajjm(amδm))\displaystyle\sum_{j=1}^{m-1}B(a_{j}-\tfrac{j}{m}(a_{m}-\delta_{m})) (j=1m1aj)m12(amδm)(j=1m1B(aj)12)m12(amδm)\displaystyle\geq\left(\sum_{j=1}^{m-1}a_{j}\right)-\tfrac{m-1}{2}(a_{m}-\delta_{m})\geq\left(\sum_{j=1}^{m-1}B(a_{j})-\tfrac{1}{2}\right)-\tfrac{m-1}{2}(a_{m}-\delta_{m})
=(j=1m1B(aj))m12(amδm+1).\displaystyle=\left(\sum_{j=1}^{m-1}B(a_{j})\right)-\tfrac{m-1}{2}(a_{m}-\delta_{m}+1).

Combining this with the other terms (which are easily shown to satisfy the analogous bound), the desired result is immediate. ∎

112233441-111223344
Figure 1. Comparing graph of B(x)B(x) (thick black) to B4(x)B_{4}(x) (dotted red) and B3(x)B_{3}(x) (dotted blue) bounds.
Remark A.8.

The function B(x)B(x) appears prominently in Theorem 10.1 and is critical in bounding the geometric side of the Kuznetsov trace formmula. Its graph is shown in Figure 1 in comparison to two other functions B4B_{4} and B3B_{3}.

In the case of GL(4)\operatorname{GL}(4), the function B4B_{4} appears [GSW21] (see Theorem 4.0.1) as a bound for the pT,Rp_{T,R} function. Indeed, making necessary adjustments due to a different choice of normalization factors (see Remark 1.2), the result of [GSW21] is that

|pT,R(4)(1;a)|Tε+27R+12i=13B4(ai).\big{\lvert}p_{T,R}^{(4)}(1;-a)\big{\rvert}\ll T^{\varepsilon+27R+12-\sum\limits_{i=1}^{3}B_{4}(a_{i})}.

Theorem 10.1 establishes the same result but with B4B_{4} replaced by BB. Although the improvement is slight, we remark that it is essential in Lemma A.6 and evidently allows the inductive method of the present paper to lead to the same asymptotic orthogonality relation as was established directly in [GSW21].

With a bit of work, one can show that the function B3B_{3}, also graphed in Figure 1, appeared in [GK13] as a bound for

|pT,R(3)(1;a)|Tε+6R+7i=12B3(ai).\big{\lvert}p_{T,R}^{(3)}(1;-a)\big{\rvert}\ll T^{\varepsilon+6R+7-\sum\limits_{i=1}^{2}B_{3}(a_{i})}.

Although this looks to be an improvement on our result here, the method of [GK13] contained an error which the present method (and the method of [GSW21]) corrects.

Lemma A.9.

Let ε>0\varepsilon>0. Then for any ρ12+\rho\in\frac{1}{2}+{\mathbb{Z}} there exists 0<ε<120<\varepsilon^{\prime}<\frac{1}{2} sufficiently small such that, setting δ=2εn2\delta=\frac{2\varepsilon^{\prime}}{n^{2}}, if a=(a1,,an1)a=(a_{1},\ldots,a_{n-1}) where

aj:=ρ+j(nj)2(1+δ),a_{j}:=\rho+\frac{j(n-j)}{2}\left(1+\delta\right),

and, for w=w(n1,,nr)w=w_{(n_{1},\ldots,n_{r})}, b(a,w)=b=(b1,,bn1)b(a,w)=b=(b_{1},\ldots,b_{n-1}) where

bnn^i+j:=an^i1an^i1+j+an^i±δ2,b_{n-\widehat{n}_{i}+j}:=a_{\widehat{n}_{i-1}}-a_{\widehat{n}_{i-1}+j}+a_{\widehat{n}_{i}}\pm\frac{\delta}{2},

(meaning that aa and bb satisfy (6.1) and (6.2), respectively), then, letting BB be the function defined in Theorem 9.2,

j=1n1(B(aj)+B(bj))n12+nρ+Φ(n1,,nr)ε,\sum_{j=1}^{n-1}\big{(}B(a_{j})+B(b_{j})\big{)}\geq\left\lfloor\frac{n-1}{2}\right\rfloor+n\rho+\Phi(n_{1},\ldots,n_{r})-\varepsilon,

where

Φ(n1,,nr):=k=1r1(nk+nk+1)(nn^k)n^k2.\Phi(n_{1},\ldots,n_{r}):=\sum_{k=1}^{r-1}(n_{k}+n_{k+1})\frac{(n-\widehat{n}_{k})\widehat{n}_{k}}{2}.
Proof.

We first note that although the bound B(x)xB(x)\geq x holds for any xx\in\mathbb{R}, for any ε>0\varepsilon>0, B(x)x+12εB(x)\geq x+\frac{1}{2}-\varepsilon provided that xx is sufficiently close to a half integer. Lemma A.11 (as justified in Remark A.12) asserts that if nn is odd then n1n-1 elements from the set of all the possible values of aka_{k} and bkb_{k} are indeed within ε\varepsilon of a half integer, and if nn is even then n2n-2 of values have this property. Hence,

(A.10) k=1n1(B(ak)+B(bk))n12+k=1n1(ak+bk)ε.\sum_{k=1}^{n-1}\big{(}B(a_{k})+B(b_{k})\big{)}\geq{\left\lfloor\frac{n-1}{2}\right\rfloor}+\sum_{k=1}^{n-1}(a_{k}+b_{k})-\varepsilon.

Since bnn^i+j=an^i1an^i1+j+an^i±δ2,b_{n-\widehat{n}_{i}+j}=a_{\widehat{n}_{i-1}}-a_{\widehat{n}_{i-1}+j}+a_{\widehat{n}_{i}}\pm\tfrac{\delta}{2}, we see that

j=1ni(bnn^i+j+an^i1+j)ni(an^i1+an^i).\sum_{j=1}^{n_{i}}\big{(}b_{n-\widehat{n}_{i}+j}+a_{\widehat{n}_{i-1}+j}\big{)}\sim n_{i}\big{(}a_{\widehat{n}_{i-1}}+a_{\widehat{n}_{i}}\big{)}.

Therefore, summing over ii, we see (making use of the fact that a0=an=0a_{0}=a_{n}=0) that

k=1n1(bk+ak)\displaystyle\sum_{k=1}^{n-1}\big{(}b_{k}+a_{k}\big{)} =i=1rni(an^i1+an^i)=i=1r1(ni+ni+1)an^i\displaystyle=\sum_{i=1}^{r}n_{i}\big{(}a_{\widehat{n}_{i-1}}+a_{\widehat{n}_{i}}\big{)}=\sum_{i=1}^{r-1}(n_{i}+n_{i+1})a_{\widehat{n}_{i}}
=k=1r1(nk+nk+1)(ρ+(nn^k)n^k2+ε)\displaystyle=\sum_{k=1}^{r-1}(n_{k}+n_{k+1})\left(\rho+\frac{(n-\widehat{n}_{k})\widehat{n}_{k}}{2}+\varepsilon^{\prime}\right)
ρ(2nn1nr)+k=1r1(nk+nk+1)(nn^k)n^k2=:Φ(n1,,nr).\displaystyle\sim\rho\big{(}2n-n_{1}-n_{r}\big{)}+\underbrace{\sum_{k=1}^{r-1}(n_{k}+n_{k+1})\frac{(n-\widehat{n}_{k})\widehat{n}_{k}}{2}}_{=:\Phi(n_{1},\ldots,n_{r})}.

Combining this with (A.10), the desired result is now immediate. ∎

Lemma A.11.

Let C=(n1,,nr)C=(n_{1},\ldots,n_{r}) be a composition of nn with r2r\geq 2. Suppose that ρ12+\rho\in\frac{1}{2}+{\mathbb{Z}}. Set a0:=0a_{0}:=0, an:=0a_{n}:=0 and for each 1kn11\leq k\leq n-1 we have ak:=ρ+k(nk)2a_{k}:=\rho+\frac{k(n-k)}{2} and for each 1ir1\leq i\leq r and 1jni1\leq j\leq n_{i} we let bi,j:=an^i1an^i1+j+an^i.b_{i,j}:=a_{\widehat{n}_{i-1}}-a_{\widehat{n}_{i-1}+j}+a_{\widehat{n}_{i}}. Then

#{kak}+#{(i,j)bi,j}={2nn1nr1 if n is odd,n21+n12+nr2+i=2r1ni2 if n is even.\#\big{\{}k\mid a_{k}\notin{\mathbb{Z}}\big{\}}+\#\big{\{}(i,j)\mid b_{i,j}\notin{\mathbb{Z}}\big{\}}=\begin{cases}2n-n_{1}-n_{r}-1&\mbox{ if $n$ is odd},\\ \frac{n}{2}-1+\big{\lfloor}\frac{n_{1}}{2}\big{\rfloor}+\big{\lfloor}\frac{n_{r}}{2}\big{\rfloor}+\sum\limits_{i=2}^{r-1}\big{\lceil}\frac{n_{i}}{2}\big{\rceil}&\mbox{ if $n$ is even}.\end{cases}
Remark A.12.

Note that the quantity given in Lemma A.11 in the case of nn odd is 2nn1nr1n12n-n_{1}-n_{r}-1\geq n-1 for any composition CC (with equality precisely when r=2r=2). If nn is even then

n21+n12+nr2+i=2r1ni2n2+n12+nr22+i=2r1ni2=n2.\tfrac{n}{2}-1+\big{\lfloor}\tfrac{n_{1}}{2}\rfloor+\big{\lfloor}\tfrac{n_{r}}{2}\big{\rfloor}+\sum\limits_{i=2}^{r-1}\big{\lceil}\tfrac{n_{i}}{2}\big{\rceil}\geq\tfrac{n}{2}+\tfrac{n_{1}}{2}+\tfrac{n_{r}}{2}-2+\sum_{i=2}^{r-1}\tfrac{n_{i}}{2}=n-2.

Equality in this case occurs precisely when n1n_{1} and nrn_{r} are both odd and all other nin_{i} are even.

Proof.

For notational purposes, set

A(n)\displaystyle A(n) :=#{1kn1ak},\displaystyle:=\#\big{\{}1\leq k\leq n-1\mid a_{k}\notin{\mathbb{Z}}\big{\}},
B(C)\displaystyle B(C) :=#{(i,j), 1ir, 1jnibi,j}.\displaystyle:=\#\big{\{}(i,j),\ 1\leq i\leq r,\ 1\leq j\leq n_{i}\ \mid\ b_{i,j}\notin{\mathbb{Z}}\big{\}}.

We first consider the case of nn odd, for which k(nk)2\frac{k(n-k)}{2}\in{\mathbb{Z}} for all integers kk. Therefore, A(n)=n1A(n)=n-1. As for B(C)B(C), note that bi,jb_{i,j} is equal to ρ\rho plus an integer as long as i1,ri\neq 1,r. Otherwise, b1,j,br,jb_{1,j},b_{r,j}\in{\mathbb{Z}}. Hence B(C)=nn1nrB(C)=n-n_{1}-n_{r}.

In the case of nn even, k(nk)2\frac{k(n-k)}{2}\in{\mathbb{Z}} exactly when kk is even. Hence A(n)=n21A(n)=\frac{n}{2}-1. To the end of finding B(C)B(C), we introduce the notation

Bi(C):=#{1jnibi,j},B_{i}(C):=\#\{1\leq j\leq n_{i}\mid b_{i,j}\notin{\mathbb{Z}}\},

for which it is clear that B(C)=i=1rBi(C)B(C)=\sum\limits_{i=1}^{r}B_{i}(C).

The cardinality of Bi(C)B_{i}(C) depends, obviously, on the integrality of bi,jb_{i,j}. To determine this, we first assume that i=1i=1. Then

b1,j=j(nj)2+n1(nn1)2.b_{1,j}=-\frac{j(n-j)}{2}+\frac{n_{1}(n-n_{1})}{2}.

Therefore (since nn is even), bi,jb_{i,j}\in{\mathbb{Z}} if and only if jn1(mod2)j\equiv n_{1}\pmod{2}. This implies that

B1(C):={n112 if n1 is odd,n12 if n1 is even.,B_{1}(C):=\begin{cases}\frac{n_{1}-1}{2}&\mbox{ if $n_{1}$ is odd},\\ \frac{n_{1}}{2}&\mbox{ if $n_{1}$ is even}.\end{cases},

or more concisely, #B1(C)=n12\#B_{1}(C)=\lfloor\frac{n_{1}}{2}\rfloor. The determination of Br(C)B_{r}(C) is similar: #Br(C)=nr2\#B_{r}(C)=\lfloor\frac{n_{r}}{2}\rfloor.

For 1<i<r1<i<r, we see that

bi,j\displaystyle b_{i,j} =ρ+n^i1(nn^i1)2(n^i1+j)(nn^i1j)2+(n^i1+ni)(nn^i1ni)2\displaystyle=\rho+\frac{\widehat{n}_{i-1}(n-\widehat{n}_{i-1})}{2}-\frac{(\widehat{n}_{i-1}+j)(n-\widehat{n}_{i-1}-j)}{2}+\frac{(\widehat{n}_{i-1}+n_{i})(n-\widehat{n}_{i-1}-n_{i})}{2}
=ρ+n^i1(nn^i1ni)(n^i1+j)(nn^i1j)2+ni(nni)2\displaystyle=\rho+\widehat{n}_{i-1}(n-\widehat{n}_{i-1}-n_{i})-\frac{(\widehat{n}_{i-1}+j)(n-\widehat{n}_{i-1}-j)}{2}+\frac{n_{i}(n-n_{i})}{2}
12+(n^i1+j)(nn^i1j)2+ni(nni)2(mod).\displaystyle\equiv\frac{1}{2}+\frac{(\widehat{n}_{i-1}+j)(n-\widehat{n}_{i-1}-j)}{2}+\frac{n_{i}(n-n_{i})}{2}\pmod{{\mathbb{Z}}}.

We see again that the integrality of bi,jb_{i,j} depends on the parity of nin_{i}. If nin_{i} is odd,

Bi(C)=#{1jni|(n^i1+j)(nn^i1j)2},B_{i}(C)=\#\left\{1\leq j\leq n_{i}\left\lvert\frac{(\widehat{n}_{i-1}+j)(n-\widehat{n}_{i-1}-j)}{2}\notin{\mathbb{Z}}\right.\right\},

and if nin_{i} is even,

Bi(C)=#{1jni|(n^i1+j)(nn^i1j)2}.B_{i}(C)=\#\left\{1\leq j\leq n_{i}\left\lvert\frac{(\widehat{n}_{i-1}+j)(n-\widehat{n}_{i-1}-j)}{2}\in{\mathbb{Z}}\right.\right\}.

One can check, arguing case by case as above, that in any event, the answer is Bi(C)=ni2B_{i}(C)=\lceil\frac{n_{i}}{2}\rceil. ∎

Lemma A.13.

Suppose that (n1,,nr)r(n_{1},\ldots,n_{r})\in{\mathbb{C}}^{r}. The function

Φ(n1,,nr):=k=1r1(nk+nk+1)(n1++nk)(nk+1++nr)2\Phi(n_{1},\ldots,n_{r}):=\sum_{k=1}^{r-1}(n_{k}+n_{k+1})\frac{(n_{1}+\cdots+n_{k})(n_{k+1}+\cdots+n_{r})}{2}

is invariant under permutations, i.e., for any σSr\sigma\in S_{r}, we have Φ(n1,,nr)=Φ(nσ(1),,nσ(r)).\Phi(n_{1},\ldots,n_{r})=\Phi(n_{\sigma(1)},\ldots,n_{\sigma(r)}). In particular, if P=n1++nrP=n_{1}+\cdots+n_{r} is a partition of nn then Φ(P):=Φ(n1,,nr)\Phi(P):=\Phi(n_{1},\ldots,n_{r}) is well defined. Moreover, among all partitions PP of nn (with r2r\geq 2),

Φ(P)Φ(n1,1)=Φ(1,n1)=n(n1)2.\Phi(P)\geq\Phi(n-1,1)=\Phi(1,n-1)=\frac{n(n-1)}{2}.
Proof.

Suppose that n=n1++nr=m1++mrn=n_{1}+\cdots+n_{r}=m_{1}+\cdots+m_{r} where

mj={nj if jk,k+1,nk+1 if j=k,nk if j=k+1.m_{j}=\begin{cases}n_{j}&\mbox{ if }j\neq k,k+1,\\ n_{k+1}&\mbox{ if }j=k,\\ n_{k}&\mbox{ if }j=k+1.\end{cases}

Then one can show by an elementary (albeit tedious) computation that Φ(n1,,nr)=Φ(m1,,mr).\Phi(n_{1},\ldots,n_{r})=\Phi(m_{1},\ldots,m_{r}). In other words, Φ\Phi is invariant under any transposition τSr\tau\in S_{r}, hence invariant under all of SrS_{r}.

Suppose that n=n1++nrn=n_{1}+\cdots+n_{r}. If nk=nk+nkn_{k}=n_{k}^{\prime}+n_{k}^{\prime\prime} for some 1kr1\leq k\leq r, then one shows via a straightforward computation that Φ(n1,,nk1,nk,nk,nk+1,nr1)Φ(n1,,nr)=nknknk2.\Phi(n_{1},\ldots,n_{k-1},n_{k}^{\prime},n_{k}^{\prime\prime},n_{k+1},\ldots n_{r-1})-\Phi(n_{1},\ldots,n_{r})=\frac{n_{k}n_{k}^{\prime}n_{k}^{\prime\prime}}{2}. If n=n1++nrn=n_{1}+\cdots+n_{r} with r>2r>2, it then follows, setting n0:=min{n1,n2,,nr}n_{0}:=\min\{n_{1},n_{2},\ldots,n_{r}\}, that

Φ(n1,,nr)>Φ(n0,nn0)=nn0(nn0)2.\Phi(n_{1},\ldots,n_{r})>\Phi(n_{0},n-n_{0})=\frac{nn_{0}(n-n_{0})}{2}.

Among all 1n0n21\leq n_{0}\leq\frac{n}{2}, the right hand side is minimized when n0=1n_{0}=1. ∎

Recall that where ΓR(z):=Γ(12+R+z2)Γ(z)\Gamma_{R}(z):=\frac{\Gamma\left(\frac{\frac{1}{2}+R+z}{2}\right)}{\Gamma(z)}, as defined at the beginning of Section 8.

Lemma A.14.

If δ\delta\in{\mathbb{Z}} and βi\beta\in i\mathbb{R} are fixed, then the function ΓR(β+z)ΓR(βz)Γ(βzδ)\Gamma_{R}(\beta+z)\Gamma_{R}(-\beta-z)\Gamma(-\beta-z-\delta) is holomorphic for all zz with |Re(z)|<R\lvert\operatorname{Re}(z)\rvert<R.

Proof.

The fact that |z|<R\lvert z\rvert<R implies that ΓR(±z)\Gamma_{R}(\pm z) is holomorphic is immediate, so the only question is what happens at the (simple) poles of Γ(βzδ)\Gamma(-\beta-z-\delta). But these occur at z=β+kz=-\beta+k for some integer kk which correspond to zeros of ΓR(β+z)\Gamma_{R}(\beta+z) or ΓR(βz)\Gamma_{R}(-\beta-z). ∎

Lemma A.15.

For δ\delta\in{\mathbb{Z}} fixed and z,βz,\beta\in{\mathbb{C}} and |Re(z+β)+δ|<R\lvert\operatorname{Re}(z+\beta)+\delta\rvert<R, we have the bound

ΓR(β+z)ΓR(βz)Γ(βzδ)(1+|Im(β+z)|)RRe(β+z)δ.\Gamma_{R}(\beta+z)\Gamma_{R}(-\beta-z)\Gamma(-\beta-z-\delta)\asymp\big{(}1+\lvert\operatorname{Im}(\beta+z)\rvert\big{)}^{R-\operatorname{Re}(\beta+z)-\delta}.
Proof.

This follows immediately from the Stirling bound |Γ(σ+it)|2π|t|σ12eπ|t|/2\lvert\Gamma(\sigma+it)\rvert\sim\sqrt{2\pi}\lvert t\rvert^{\sigma-\frac{1}{2}}e^{\pi\lvert t\rvert/2}. ∎

Definition A.16.

Let α=(α1,,αn)n\alpha=(\alpha_{1},\ldots,\alpha_{n})\in\mathbb{C}^{n} be Langlands parameters satisfying α^n=0.\widehat{\alpha}_{n}=0. Let n=n1++nrn=n_{1}+\cdots+n_{r} be a partition of nn with n1,,nr+.n_{1},\ldots,n_{r}\in{\mathbb{Z}}_{+}. Then for each =1,,r\ell=1,\ldots,r we define α():=(α1(),,αn())n\alpha^{(\ell)}:=\big{(}\alpha_{1}^{(\ell)},\ldots,\alpha_{n_{\ell}}^{(\ell)}\big{)}\in{\mathbb{C}}^{n_{\ell}} where

αj():=αn^1+j1n(α^n^α^n^1),|α()|2:=j=1n(αj())2.\alpha_{j}^{(\ell)}:=\alpha_{\widehat{n}_{\ell-1}+j}-\tfrac{1}{n_{\ell}}\big{(}\widehat{\alpha}_{\widehat{n}_{\ell}}-\widehat{\alpha}_{\widehat{n}_{\ell-1}}\big{)},\qquad\quad\lvert\alpha^{(\ell)}\rvert^{2}:=\sum\limits_{j=1}^{n_{\ell}}\big{(}\alpha_{j}^{(\ell)}\big{)}^{2}.
Remark A.17.

Note that j=1nαj()=0\sum\limits_{j=1}^{n_{\ell}}\alpha_{j}^{(\ell)}=0 for each .\ell. In particular n=1n_{\ell}=1 implies α1()=0.\alpha_{1}^{(\ell)}=0.

Lemma A.18.

We have |α|2=i=1nαi2==1r(|α()|2+1n(α^n^α^n^1)2).\lvert\alpha\rvert^{2}=\sum\limits_{i=1}^{n}\alpha_{i}^{2}=\sum\limits_{\ell=1}^{r}\left(\big{\lvert}\alpha^{(\ell)}\big{\rvert}^{2}+\frac{1}{n_{\ell}}\big{(}\widehat{\alpha}_{\widehat{n}_{\ell}}-\widehat{\alpha}_{\widehat{n}_{\ell-1}}\big{)}^{2}\right).

Proof.

Computing directly, and using the fact that j=1nαj()=0\sum\limits_{j=1}^{n_{\ell}}\alpha_{j}^{(\ell)}=0, we find that

j=1nαj2\displaystyle\sum_{j=1}^{n}\alpha_{j}^{2} ==1rj=1nαn^1+j2==1rj=1n(αj()+1n(α^n^α^n^1))2\displaystyle=\sum_{\ell=1}^{r}\sum_{j=1}^{n_{\ell}}\alpha_{\widehat{n}_{\ell-1}+j}^{2}=\sum_{\ell=1}^{r}\sum_{j=1}^{n_{\ell}}\big{(}\alpha_{j}^{(\ell)}+\tfrac{1}{n_{\ell}}\big{(}\widehat{\alpha}_{\widehat{n}_{\ell}}-\widehat{\alpha}_{\widehat{n}_{\ell-1}}\big{)}\big{)}^{2}
==1rj=1n((αj())2+2nαj()(α^n^α^n^1)+1n2(α^n^α^n^1)2)\displaystyle=\sum_{\ell=1}^{r}\sum_{j=1}^{n_{\ell}}\left(\big{(}\alpha_{j}^{(\ell)}\big{)}^{2}+\tfrac{2}{n_{\ell}}\alpha_{j}^{(\ell)}\big{(}\widehat{\alpha}_{\widehat{n}_{\ell}}-\widehat{\alpha}_{\widehat{n}_{\ell-1}}\big{)}+\tfrac{1}{n_{\ell}^{2}}\big{(}\widehat{\alpha}_{\widehat{n}_{\ell}}-\widehat{\alpha}_{\widehat{n}_{\ell-1}}\big{)}^{2}\right)
==1r(|α()|2+1n(α^n^α^n^1)2),\displaystyle=\sum_{\ell=1}^{r}\left(\big{\lvert}\alpha^{(\ell)}\big{\rvert}^{2}+\tfrac{1}{n_{\ell}}\big{(}\widehat{\alpha}_{\widehat{n}_{\ell}}-\widehat{\alpha}_{\widehat{n}_{\ell-1}}\big{)}^{2}\right),

as claimed. ∎

Lemma A.19.

Suppose that n2n\geq 2 and α1,α2,,αn\alpha_{1},\alpha_{2},\ldots,\alpha_{n}\in{\mathbb{C}} satisfies α1+α2+αn=0.\alpha_{1}+\alpha_{2}+\cdots\alpha_{n}=0. Set α^k=j=1kαj\widehat{\alpha}_{k}=\sum\limits_{j=1}^{k}\alpha_{j} for fixed k{1,2,,n}k\in\{1,2,\ldots,n\}, and define βj:=αj1kα^k,γj:=αj+k+1nkα^k.\beta_{j}:=\alpha_{j}-\frac{1}{k}\widehat{\alpha}_{k},\qquad\gamma_{j}:=\alpha_{j+k}+\frac{1}{n-k}\widehat{\alpha}_{k}. Then

i=1nαi2=i=1kβi2+i=1nkγi2+nk(nk)α^k2.\sum_{i=1}^{n}\alpha_{i}^{2}=\sum_{i=1}^{k}\beta_{i}^{2}+\sum_{i=1}^{n-k}\gamma_{i}^{2}+\frac{n}{k(n-k)}\widehat{\alpha}_{k}^{2}.
Proof.

This is easily deduced as a special case of Lemma A.18 in the case that r=2r=2, n1=kn_{1}=k, n2=nkn_{2}=n-k, β=α(1)\beta=\alpha^{(1)} and γ=α(2)\gamma=\alpha^{(2)}. ∎

Lemma A.20.

We continue the notation of Lemma A.18. Then

1ijnΓR(αiαj)==1r(1i,jnΓR(αi()αj()))1k<mri=1nkj=1nmϵ{±1}ΓR(ϵ(αi(k)αj(m)+1nk(α^n^kα^n^k1)1nm(α^n^mα^n^m1))).\prod_{1\leq i\neq j\leq n}\Gamma_{R}(\alpha_{i}-\alpha_{j})=\prod_{\ell=1}^{r}\Bigg{(}\prod_{1\leq i,j\leq n_{\ell}}\Gamma_{R}\big{(}\alpha_{i}^{(\ell)}-\alpha_{j}^{(\ell)}\big{)}\Bigg{)}\\ \cdot\prod_{1\leq k<m\leq r}\prod_{i=1}^{n_{k}}\prod_{j=1}^{n_{m}}\prod_{\epsilon\in\{\pm 1\}}\Gamma_{R}\left(\epsilon\bigg{(}\alpha_{i}^{(k)}-\alpha_{j}^{(m)}+\tfrac{1}{n_{k}}\big{(}\widehat{\alpha}_{\widehat{n}_{k}}-\widehat{\alpha}_{\widehat{n}_{k-1}}\big{)}-\tfrac{1}{n_{m}}\big{(}\widehat{\alpha}_{\widehat{n}_{m}}-\widehat{\alpha}_{\widehat{n}_{m-1}}\big{)}\bigg{)}\right).
Proof.

Note that if kmk\neq m, then for any 1ink1\leq i\leq n_{k} and 1jnm1\leq j\leq n_{m},

αn^k1+iαn^m1+j=αi(k)αj(m)+1nk(α^n^kα^n^k1)1nm(α^n^mα^n^m1),\alpha_{\widehat{n}_{k-1}+i}-\alpha_{\widehat{n}_{m-1}+j}=\alpha_{i}^{(k)}-\alpha_{j}^{(m)}+\tfrac{1}{n_{k}}\big{(}\widehat{\alpha}_{\widehat{n}_{k}}-\widehat{\alpha}_{\widehat{n}_{k-1}}\big{)}-\tfrac{1}{n_{m}}\big{(}\widehat{\alpha}_{\widehat{n}_{m}}-\widehat{\alpha}_{\widehat{n}_{m-1}}\big{)},

and for any 1ijn1\leq i\neq j\leq n_{\ell} we have αn^1+iαn^1+j=αi()αj().\alpha_{\widehat{n}_{\ell-1}+i}-\alpha_{\widehat{n}_{\ell-1}+j}=\alpha_{i}^{(\ell)}-\alpha_{j}^{(\ell)}. This immediately implies the desired formula. ∎

Lemma A.21.

Suppose that (β1,,βr)(\beta_{1},\ldots,\beta_{r}) satisfies β^r=0\hat{\beta}_{r}=0. Suppose that n=n1++nrn=n_{1}+\cdots+n_{r} and set n^k=j=1knj\widehat{n}_{k}=\sum\limits_{j=1}^{k}n_{j}. Then 1k<mri=1nkj=1nm(βknkβmnm)=j=1r1(nj+nj+1)β^j.\sum\limits_{1\leq k<m\leq r}\sum\limits_{i=1}^{n_{k}}\sum\limits_{j=1}^{n_{m}}\left(\frac{\beta_{k}}{n_{k}}-\frac{\beta_{m}}{n_{m}}\right)=\sum\limits_{j=1}^{r-1}(n_{j}+n_{j+1})\widehat{\beta}_{j}.

Proof.

We calculate

1k<mri=1nkj=1nm(βknkβmnm)\displaystyle\sum_{1\leq k<m\leq r}\sum_{i=1}^{n_{k}}\sum_{j=1}^{n_{m}}\left(\frac{\beta_{k}}{n_{k}}-\frac{\beta_{m}}{n_{m}}\right) =m=2rk=1m1i=1nk(nmβknkβm)=m=2rk=1m1(nmβknkβm)\displaystyle=\sum_{m=2}^{r}\sum_{k=1}^{m-1}\sum_{i=1}^{n_{k}}\left(n_{m}\frac{\beta_{k}}{n_{k}}-\beta_{m}\right)=\sum_{m=2}^{r}\sum_{k=1}^{m-1}\big{(}n_{m}\beta_{k}-n_{k}\beta_{m}\big{)}
=m=2r(nmβ^m1n^m1βm)=m=2r(nmβ^m1n^m1(β^mβ^m1))\displaystyle=\sum_{m=2}^{r}\big{(}n_{m}\widehat{\beta}_{m-1}-\widehat{n}_{m-1}\beta_{m}\big{)}=\sum_{m=2}^{r}\big{(}n_{m}\widehat{\beta}_{m-1}-\widehat{n}_{m-1}(\widehat{\beta}_{m}-\widehat{\beta}_{m-1})\big{)}
=m=2r((nm+n^m1)β^m1n^m1β^m)=m=2r(n^mβ^m1n^m1β^m).\displaystyle=\sum_{m=2}^{r}\big{(}(n_{m}+\widehat{n}_{m-1})\widehat{\beta}_{m-1}-\widehat{n}_{m-1}\widehat{\beta}_{m}\big{)}=\sum_{m=2}^{r}\big{(}\widehat{n}_{m}\widehat{\beta}_{m-1}-\widehat{n}_{m-1}\widehat{\beta}_{m}\big{)}.

This final sum telescopes to give j=1r1(n^j+1n^j1)β^j\sum\limits_{j=1}^{r-1}\big{(}\widehat{n}_{j+1}-\widehat{n}_{j-1}\big{)}\widehat{\beta}_{j}. Since n^j+1n^j1=nj+nj+1\widehat{n}_{j+1}-\widehat{n}_{j-1}=n_{j}+n_{j+1}, this implies the claimed result. ∎

The following result can be interpreted as a consequence—by counting (half) the number of gamma factors on each side of the equality—of Lemma A.20. Alternatively, proving it independent of Lemma A.20 gives further evidence that the product decomposition is correct.

Lemma A.22.

Let n=n1++nrn=n_{1}+\cdots+n_{r}. We have 1k<krnknk+k=1rnk(nk1)2=n(n1)2.\sum\limits_{1\leq k<k^{\prime}\leq r}\hskip-5.0ptn_{k}\cdot n_{k^{\prime}}\;+\sum\limits_{k=1}^{r}\frac{n_{k}(n_{k}-1)}{2}=\frac{n(n-1)}{2}.

Proof.

We use induction on rr. If r=1r=1, the formula obviously holds. Let n=m+nrn=m+n_{r} where m=n1++nr1m=n_{1}+\cdots+n_{r-1}. Then, by induction,

n(n1)2\displaystyle\frac{n(n-1)}{2} =(m+nr)(m+nr1)2=m(m1)2+mnr+(m1)nr2+nr22\displaystyle=\frac{(m+n_{r})(m+n_{r}-1)}{2}=\frac{m(m-1)}{2}+\frac{mn_{r}+(m-1)n_{r}}{2}+\frac{n_{r}^{2}}{2}
=k=1r1nk(nk1)2+1k<kr1nknk+mnr+nr(nr1)2.\displaystyle=\sum_{k=1}^{r-1}\frac{n_{k}(n_{k}-1)}{2}+\sum_{1\leq k<k^{\prime}\leq r-1}n_{k}\cdot n_{k^{\prime}}+mn_{r}+\frac{n_{r}(n_{r}-1)}{2}.

Since mnr=n1nr+n2nr+nr1nrmn_{r}=n_{1}n_{r}+n_{2}n_{r}+\cdots n_{r-1}n_{r}, it is evident that the desired formula holds. ∎

Lemma A.23.

Suppose n=n1++nrn=n_{1}+\cdots+n_{r}. Then n2+=1r(n(n1)2nn^)=n(n1)2.n^{2}+\sum\limits_{\ell=1}^{r}\Big{(}\frac{n_{\ell}(n_{\ell}-1)}{2}-n_{\ell}\widehat{n}_{\ell}\Big{)}=\frac{n(n-1)}{2}.

Proof.

If r=1r=1 the result is obviously true. Suppose that the result holds for r=kr=k. Write n=n1++nk+nk+1=n^k+nk+1n=n_{1}+\cdots+n_{k}+n_{k+1}=\widehat{n}_{k}+n_{k+1}. Then

n2+=1k+1(n(n1)2nn^)\displaystyle n^{2}+\sum_{\ell=1}^{k+1}\Big{(}\frac{n_{\ell}(n_{\ell}-1)}{2}-n_{\ell}\widehat{n}_{\ell}\Big{)} =n2+=1k(n(n1)2nn^)+nk+1(nk+11)2nk+1n\displaystyle=n^{2}+\sum_{\ell=1}^{k}\Big{(}\frac{n_{\ell}(n_{\ell}-1)}{2}-n_{\ell}\widehat{n}_{\ell}\Big{)}+\frac{n_{k+1}(n_{k+1}-1)}{2}-n_{k+1}n
=n2n^k2+(n^k2+=1k(n(n1)2nn^)+nk+1(nk+11)2nk+1n)\displaystyle=n^{2}-\widehat{n}_{k}^{2}+\left(\widehat{n}_{k}^{2}+\sum_{\ell=1}^{k}\Big{(}\frac{n_{\ell}(n_{\ell}-1)}{2}-n_{\ell}\widehat{n}_{\ell}\Big{)}+\frac{n_{k+1}(n_{k+1}-1)}{2}-n_{k+1}n\right)
=n2n^k2+n^k(n^k+1)2+nk+1(nk+11)2nk+1n\displaystyle=n^{2}-\widehat{n}_{k}^{2}+\frac{\widehat{n}_{k}(\widehat{n}_{k}+1)}{2}+\frac{n_{k+1}(n_{k+1}-1)}{2}-n_{k+1}n
=n2n^k2+n^k(n^k+1)2+(nn^k)(nn^k1)2(nn^k),\displaystyle=n^{2}-\widehat{n}_{k}^{2}+\frac{\widehat{n}_{k}(\widehat{n}_{k}+1)}{2}+\frac{(n-\widehat{n}_{k})(n-\widehat{n}_{k}-1)}{2}-(n-\widehat{n}_{k}),

which can easily be shown now to simplify to n(n1)2\frac{n(n-1)}{2}, as claimed. ∎

Remark A.24.

Note that Lemma A.22 and Lemma A.23 are equivalent provided that

(A.25) n2=1rnn^=1k<krnknk.n^{2}-\sum_{\ell=1}^{r}n_{\ell}\widehat{n}_{\ell}=\sum_{1\leq k<k^{\prime}\leq r}n_{k}n_{k^{\prime}}.

This can be verified by expanding the left hand side as follows:

n2=1rnn^=(n1++nr)n^r=1rnn^==1r(n(nn^)nn^)==1rn(nn).\displaystyle n^{2}-\sum_{\ell=1}^{r}n_{\ell}\widehat{n}_{\ell}=(n_{1}+\cdots+n_{r})\widehat{n}_{r}-\sum_{\ell=1}^{r}n_{\ell}\widehat{n}_{\ell}=\sum_{\ell=1}^{r}\big{(}n_{\ell}(n-\widehat{n}_{\ell})-n_{\ell}\widehat{n}_{\ell}\big{)}=\sum_{\ell=1}^{r}n_{\ell}(n-n_{\ell}).

That this final expression is equal to right hand side of (A.25) is clear.

Lemma A.26.

Let α1,α2,,αn\alpha_{1},\alpha_{2},\ldots,\alpha_{n}\in{\mathbb{C}} satisfy α1+α2+αn=0\alpha_{1}+\alpha_{2}+\cdots\alpha_{n}=0. Set α^k:=j=1kαj\widehat{\alpha}_{k}:=\sum\limits_{j=1}^{k}\alpha_{j}, and let βi\beta_{i} (1ik1\leq i\leq k) and γj\gamma_{j} (1jnk1\leq j\leq n-k) be as in the previous lemma. We have

1ijnΓR(αiαj)=(1ijkΓR(βiβj))(1ijnkΓR(γiγj))i=1kj=1nkΓR(βiγj+nk(nk)α^k)ΓR(γjβink(nk)α^k)\prod_{1\leq i\neq j\leq n}\Gamma_{R}(\alpha_{i}-\alpha_{j})=\left(\prod_{1\leq i\neq j\leq k}\Gamma_{R}(\beta_{i}-\beta_{j})\right)\left(\prod_{1\leq i\neq j\leq n-k}\Gamma_{R}(\gamma_{i}-\gamma_{j})\right)\\ \cdot\prod_{i=1}^{k}\prod_{j=1}^{n-k}\Gamma_{R}\left(\beta_{i}-\gamma_{j}+\frac{n}{k(n-k)}\widehat{\alpha}_{k}\right)\Gamma_{R}\left(\gamma_{j}-\beta_{i}-\frac{n}{k(n-k)}\widehat{\alpha}_{k}\right)
Proof.

This is easily deduced as a special case of Lemma A.20 when r=2r=2, n1=kn_{1}=k, n2=nkn_{2}=n-k, β=α(1)\beta=\alpha^{(1)} and γ=α(2)\gamma=\alpha^{(2)}. ∎

We recall the definition of the polynomial given in Definition 1.1:

R(n)(α):=j=1n2K,L(1,2,,n)#K=#L=j(1+kKαkLα)R2.\mathcal{F}_{R}^{(n)}(\alpha):=\prod_{j=1}^{n-2}\;\underset{\#K=\#L=j}{\prod_{K,L\,\subseteq\,(1,2,\ldots,n)}}\left(1+\sum_{k\in K}\alpha_{k}-\sum_{\ell\in L}\alpha_{\ell}\right)^{\frac{R}{2}}.

Also, we remind the reader of the polynomial notation 𝒫\mathcal{P} given in Definition 2.3.

Lemma A.27.

Let n=n1++nrn=n_{1}+\cdots+n_{r}, α\alpha, and α()\alpha^{(\ell)} be as in Definition A.16. Set D(n)=deg(1(n)(α))D(n)=\deg(\mathcal{F}_{1}^{(n)}(\alpha)). Then

R(n)(α)=𝒫dR(α)=1rn1R(n)(α())whered=d(n1,,nr)=D(n)=1rn1D(n).\mathcal{F}_{R}^{(n)}(\alpha)=\mathcal{P}_{dR}(\alpha)\underset{n_{\ell}\neq 1}{\prod_{\ell=1}^{r}}\mathcal{F}_{R}^{(n_{\ell})}(\alpha^{(\ell)})\quad\mbox{where}\quad d=d(n_{1},\ldots,n_{r})=D(n)-\underset{n_{\ell}\neq 1}{\sum_{\ell=1}^{r}}D(n_{\ell}).
Proof.

This follows from the fact that if I,J{1,2,,n}I,J\subseteq\{1,2,\ldots,n_{\ell}\} with #I=#J\#I=\#J then

(iIαi())(jJαj())=(iIαn^1+i)(jJαn^1+j).\left(\sum_{i\in I}\alpha_{i}^{(\ell)}\right)-\left(\sum_{j\in J}\alpha_{j}^{(\ell)}\right)=\left(\sum_{i\in I}\alpha_{\widehat{n}_{\ell-1}+i}\right)-\left(\sum_{j\in J}\alpha_{\widehat{n}_{\ell-1}+j}\right).

Therefore, each R(n)(α())\mathcal{F}_{R}^{(n_{\ell})}(\alpha^{(\ell)}) constitutes a unique factor of R(n)(α)\mathcal{F}_{R}^{(n)}(\alpha) for each =1,,r\ell=1,\ldots,r. ∎

Lemma A.28.

Suppose that δ0\delta\in{\mathbb{Z}}_{\geq 0} and R>δR>\delta. Then

R(n)(α)K{1,2,,n}#(K{1,2,,m})m1#K=m((iKαi)α^mδ)δ1R(m)(β)R(nm)(γ)𝒫d(α),\mathcal{F}_{R}^{(n)}(\alpha)\cdot\hskip-17.0pt\prod_{\begin{subarray}{c}K\subseteq\{1,2,\ldots,n\}\\ \#(K\cap\{1,2,\ldots,m\})\neq m-1\\ \#K=m\end{subarray}}\left(\Big{(}\sum_{i\in K}\alpha_{i}\Big{)}-\widehat{\alpha}_{m}-\delta\right)_{\delta}^{-1}\ll\mathcal{F}_{R}^{(m)}(\beta)\cdot\mathcal{F}_{R}^{(n-m)}(\gamma)\cdot\mathcal{P}_{d}(\alpha),

where d=R(D(n)D(m)D(nm))δ((nm)m(nm)1)d=R\big{(}D(n)-D(m)-D(n-m)\big{)}-\delta\big{(}\binom{n}{m}-m(n-m)-1\big{)}.

Proof.

Let M:={1,2,,m}M:=\{1,2,\ldots,m\}. Then

#{K{1,2,,n}|#K=m,#(KM)0,1}=(nm)m(nm)1.\#\big{\{}K\subseteq\{1,2,\ldots,n\}\,\big{|}\,\#K=m,\,\#(K\cap M)\neq 0,1\big{\}}=\binom{n}{m}-m(n-m)-1.

From the definition of R(n)(α)\mathcal{F}_{R}^{(n)}(\alpha) given in Definition 1.1, we see that for each such KK there are factors

(1+iKαijMαj)R/2(1iKαi+jMαj)R/2\left(1+\sum_{i\in K}\alpha_{i}-\sum_{j\in M}\alpha_{j}\right)^{R/2}\left(1-\sum_{i\in K}\alpha_{i}+\sum_{j\in M}\alpha_{j}\right)^{R/2}

of R(n)(α)/[R(m)(β)R(nm)(γ)]\mathcal{F}_{R}^{(n)}(\alpha)/[\mathcal{F}_{R}^{(m)}(\beta)\mathcal{F}_{R}^{(n-m)}(\gamma)] for which

(1+iKαijMαj)R/2(1iKαi+jMαj)R/2(iKαijMαjδ)δ(1(iKαijMαj)2)Rδ2.\frac{\left(1+\sum\limits_{i\in K}\alpha_{i}-\sum\limits_{j\in M}\alpha_{j}\right)^{R/2}\left(1-\sum\limits_{i\in K}\alpha_{i}+\sum\limits_{j\in M}\alpha_{j}\right)^{R/2}}{\left(\sum\limits_{i\in K}\alpha_{i}-\sum\limits_{j\in M}\alpha_{j}-\delta\right)_{\delta}}\ll\left(1-\Big{(}\sum\limits_{i\in K}\alpha_{i}-\sum\limits_{j\in M}\alpha_{j}\Big{)}^{2}\right)^{\frac{R-\delta}{2}}.

This bound holds because the degree of the Pochhammer symbol in the denominator is δ\delta, and by assumption, the degree of the numerator is R>δR>\delta. Combining all such terms with the remaining factors of R(n)(α)/[R(m)(β)R(nm)(γ)]\mathcal{F}_{R}^{(n)}(\alpha)/[\mathcal{F}_{R}^{(m)}(\beta)\mathcal{F}_{R}^{(n-m)}(\gamma)], gives a polynomial of degree dd. ∎

Remark A.29.

Let n=n1+n2++nrn=n_{1}+n_{2}+\cdots+n_{r} and n^=i=1ni\widehat{n}_{\ell}=\sum\limits_{i=1}^{\ell}n_{i}. The result of Lemma A.28 clearly generalizes to the case of taking multiple residues at sn^=α^n^δs_{\widehat{n}_{\ell}}=-\widehat{\alpha}_{\widehat{n}_{\ell}}-\delta_{\ell} for each =1,,r1\ell=1,\ldots,r-1 (in reverse order). In this case, taking the product on the left hand side over all of the terms we obtain

R(n)(α)=1r1K{1,2,,n^+1}#(K{1,,n^})n^1#K=n^((iKαi)α^n^δ)δ1𝒫d(α)=1rR(n)(α()),\mathcal{F}_{R}^{(n)}(\alpha)\cdot\prod_{\ell=1}^{r-1}\prod_{\begin{subarray}{c}K\subseteq\{1,2,\ldots,\widehat{n}_{\ell+1}\}\\ \#(K\cap\{1,\ldots,\widehat{n}_{\ell}\})\neq\widehat{n}_{\ell}-1\\ \#K=\widehat{n}_{\ell}\end{subarray}}\left(\Big{(}\sum_{i\in K}\alpha_{i}\Big{)}-\widehat{\alpha}_{\widehat{n}_{\ell}}-\delta_{\ell}\right)_{\delta_{\ell}}^{-1}\ll\mathcal{P}_{d}(\alpha)\cdot\prod_{\ell=1}^{r}\mathcal{F}_{R}^{(n_{\ell})}\big{(}\alpha^{(\ell)}\big{)},

where

d=R(D(n)=1rD(n))=1r1[δ((n^+1n^)n+1n^1)].d=R\cdot\left(D(n)-\sum_{\ell=1}^{r}D(n_{\ell})\right)-\sum\limits_{\ell=1}^{r-1}\left[\delta_{\ell}\left(\binom{\widehat{n}_{\ell+1}}{\widehat{n}_{\ell}}-n_{\ell+1}\widehat{n}_{\ell}-1\right)\right].

Acknowledgments

Eric Stade would like to thank Taku Ishii for many helpful converations, and for the ideas constituting the proof of Conjecture 8.3 in the case n=5n=5. Michael Woodbury would like to thank the University of Colorado for wonderful accommodations while hosting him during the Spring 2022 semester. We would also like to thank the referees for many helpful comments.


References

  • [Art79] James Arthur, Eisenstein series and the trace formula, Automorphic forms, representations and LL-functions (Proc. Sympos. Pure Math., Oregon State Univ., Corvallis, Ore., 1977), Part 1, Proc. Sympos. Pure Math., XXXIII, Amer. Math. Soc., Providence, R.I., 1979, pp. 253–274. MR 546601 (81b:10020)
  • [Blo13] Valentin Blomer, Applications of the Kuznetsov formula on GL(3)GL(3), Invent. Math. 194 (2013), no. 3, 673–729. MR 3127065
  • [Blo21] by same author, The relative trace formula in analytic number theory, Relative trace formulas, Simons Symp., Springer, Cham, [2021] ©2021, pp. 51–73. MR 4611943
  • [Bru78] R. W. Bruggeman, Fourier coefficients of cusp forms, Invent. Math. 45 (1978), no. 1, 1–18. MR 472701
  • [Bru06] Farrell Brumley, Effective multiplicity one on GLN{\rm GL}_{N} and narrow zero-free regions for Rankin-Selberg LL-functions, Amer. J. Math. 128 (2006), no. 6, 1455–1474. MR 2275908
  • [BZ20] Jack Buttcane and Fan Zhou, Plancherel distribution of Satake parameters of Maass cusp forms on GL3{\rm GL}_{3}, Int. Math. Res. Not. IMRN (2020), no. 5, 1417–1444. MR 4073945
  • [CDF97] J. B. Conrey, W. Duke, and D. W. Farmer, The distribution of the eigenvalues of Hecke operators, Acta Arith. 78 (1997), no. 4, 405–409. MR 1438595
  • [DR98] Romuald Dabrowski and Mark Reeder, Kloosterman sets in reductive groups, J. Number Theory 73 (1998), no. 2, 228–255. MR 1658031
  • [FM21] Tobias Finis and Jasmin Matz, On the asymptotics of Hecke operators for reductive groups, Math. Ann. 380 (2021), no. 3-4, 1037–1104. MR 4297181
  • [Fri87] Solomon Friedberg, Poincaré series for GL(n){\rm GL}(n): Fourier expansion, Kloosterman sums, and algebreo-geometric estimates, Math. Z. 196 (1987), no. 2, 165–188. MR 910824
  • [GJ72] Roger Godement and Hervé Jacquet, Zeta functions of simple algebras, Lecture Notes in Mathematics, Vol. 260, Springer-Verlag, Berlin-New York, 1972. MR 0342495
  • [GK12] Dorian Goldfeld and Alex Kontorovich, On the determination of the Plancherel measure for Lebedev-Whittaker transforms on GL(n){\rm GL}(n), Acta Arith. 155 (2012), no. 1, 15–26. MR 2982424
  • [GK13] by same author, On the GL(3){\rm GL}(3) Kuznetsov formula with applications to symmetry types of families of LL-functions, Automorphic representations and LL-functions, Tata Inst. Fundam. Res. Stud. Math., vol. 22, Tata Inst. Fund. Res., Mumbai, 2013, pp. 263–310. MR 3156855
  • [GL18] Dorian Goldfeld and Xiaoqing Li, A standard zero free region for Rankin-Selberg LL-functions, Int. Math. Res. Not. IMRN (2018), no. 22, 7067–7136. MR 3878594
  • [GMW21] Dorian Goldfeld, Stephen D. Miller, and Michael Woodbury, A template method for Fourier coefficients of Langlands Eisenstein series, Riv. Math. Univ. Parma (N.S.) 12 (2021), no. 1, 63–117. MR 4284774
  • [Gol06] Dorian Goldfeld, Automorphic forms and LL-functions for the group GL(n,R){\rm GL}(n,\mathbf{R}), Cambridge Studies in Advanced Mathematics, vol. 99, Cambridge University Press, Cambridge, 2006, With an appendix by Kevin A. Broughan. MR 2254662 (2008d:11046)
  • [Gol15] by same author, Automorphic forms and L-functions for the group GL(n,R){\rm GL}(n,\rm R), Cambridge Studies in Advanced Mathematics, vol. 99, Cambridge University Press, Cambridge, 2015, With an appendix by Kevin A. Broughan, Paperback edition of the 2006 original [ MR2254662]. MR 3468028
  • [GSW21] Dorian Goldfeld, Eric Stade, and Michael Woodbury, An orthogonality relation for GL(4,)\rm GL(4,\mathbb{R}) (with an appendix by Bingrong Huang), Forum Math. Sigma 9 (2021), Paper No. e47, 83. MR 4271357
  • [GSW24] by same author, The first coefficient of Langlands Eisenstein series for SL(n,)\mathrm{SL}(n,\mathbb{Z}), Acta Arith. 214 (2024), 179–189.
  • [HB19] Peter Humphries and Farrell Brumley, Standard zero-free regions for Rankin-Selberg LL-functions via sieve theory, Math. Z. 292 (2019), no. 3-4, 1105–1122. MR 3980284
  • [IO14] Taku Ishii and Takayuki Oda, Calculus of principal series Whittaker functions on SL(n,)SL(n,\mathbb{R}), J. Funct. Anal. 266 (2014), no. 3, 1286–1372. MR 3146819
  • [IS00] H. Iwaniec and P. Sarnak, Perspectives on the analytic theory of LL-functions, no. Special Volume, Part II, 2000, GAFA 2000 (Tel Aviv, 1999), pp. 705–741. MR 1826269
  • [IS07] Taku Ishii and Eric Stade, New formulas for Whittaker functions on GL(n,){\rm GL}(n,\mathbb{R}), J. Funct. Anal. 244 (2007), no. 1, 289–314. MR 2294485
  • [Jac67] Hervé Jacquet, Fonctions de Whittaker associées aux groupes de Chevalley, Bull. Soc. Math. France 95 (1967), 243–309. MR 0271275
  • [Jan21] Subhajit Jana, Applications of analytic newvectors for GL(n)\text{GL}(n), Math. Ann. 380 (2021), no. 3-4, 915–952. MR 4297178
  • [JL85] H. Jacquet and K. F. Lai, A relative trace formula, Compositio Math. 54 (1985), no. 2, 243–310. MR 783512
  • [JN19] Subhajit Jana and Paul D. Nelson, Analytic newvectors for GLn()\mathrm{GL}_{n}(\mathbb{R}), arXiv e-prints (2019), arXiv:1911.01880.
  • [Kim03] Henry H. Kim, Functoriality for the exterior square of GL4{\rm GL}_{4} and the symmetric fourth of GL2{\rm GL}_{2}, J. Amer. Math. Soc. 16 (2003), no. 1, 139–183, With appendix 1 by Dinakar Ramakrishnan and appendix 2 by Kim and Peter Sarnak. MR 1937203
  • [KS02] Henry H. Kim and Freydoon Shahidi, Functorial products for GL2×GL3{\rm GL}_{2}\times{\rm GL}_{3} and the symmetric cube for GL2{\rm GL}_{2}, Ann. of Math. (2) 155 (2002), no. 3, 837–893, With an appendix by Colin J. Bushnell and Guy Henniart. MR 1923967
  • [Lan76] Robert P. Langlands, On the functional equations satisfied by Eisenstein series, Lecture Notes in Mathematics, Vol. 544, Springer-Verlag, Berlin-New York, 1976. MR 0579181
  • [Lap13] Erez Lapid, On the Harish-Chandra Schwartz space of G(F)\G(𝔸)G(F)\backslash G(\mathbb{A}), Automorphic representations and LL-functions, Tata Inst. Fundam. Res. Stud. Math., vol. 22, Tata Inst. Fund. Res., Mumbai, 2013, With an appendix by Farrell Brumley, pp. 335–377. MR 3156857
  • [LM09] Erez Lapid and Werner Müller, Spectral asymptotics for arithmetic quotients of SL(n,)/SO(n){\rm SL}(n,\mathbb{R})/{\rm SO}(n), Duke Math. J. 149 (2009), no. 1, 117–155. MR 2541128
  • [LRS99] Wenzhi Luo, Zeév Rudnick, and Peter Sarnak, On the generalized Ramanujan conjecture for GL(n){\rm GL}(n), Automorphic forms, automorphic representations, and arithmetic (Fort Worth, TX, 1996), Proc. Sympos. Pure Math., vol. 66, Amer. Math. Soc., Providence, RI, 1999, pp. 301–310. MR 1703764
  • [Mac79] I. G. Macdonald, Symmetric functions and Hall polynomials, Oxford Mathematical Monographs, The Clarendon Press, Oxford University Press, New York, 1979. MR 553598
  • [Mil02] Stephen D. Miller, The highest lowest zero and other applications of positivity, Duke Math. J. 112 (2002), no. 1, 83–116. MR 1890648
  • [MT21] Jasmin Matz and Nicolas Templier, Sato-Tate equidistribution for families of Hecke-Maass forms on SL(n,)/SO(n){\rm SL}(n,\mathbb{R})/{\rm SO}(n), Algebra Number Theory 15 (2021), no. 6, 1343–1428. MR 4324829
  • [MW95] C. Mœglin and J.-L. Waldspurger, Spectral decomposition and Eisenstein series, Cambridge Tracts in Mathematics, vol. 113, Cambridge University Press, Cambridge, 1995, Une paraphrase de l’Écriture [A paraphrase of Scripture]. MR 1361168
  • [Ram00] Dinakar Ramakrishnan, Modularity of the Rankin-Selberg LL-series, and multiplicity one for SL(2){\rm SL}(2), Ann. of Math. (2) 152 (2000), no. 1, 45–111. MR 1792292
  • [Sar87] Peter Sarnak, Statistical properties of eigenvalues of the Hecke operators, Analytic number theory and Diophantine problems (Stillwater, OK, 1984), Progr. Math., vol. 70, Birkhäuser Boston, Boston, MA, 1987, pp. 321–331. MR 1018385
  • [Sar04] by same author, Nonvanishing of LL-functions on (s)=1\Re(s)=1, Contributions to automorphic forms, geometry, and number theory, Johns Hopkins Univ. Press, Baltimore, MD, 2004, pp. 719–732. MR 2058625
  • [Ser97] Jean-Pierre Serre, Répartition asymptotique des valeurs propres de l’opérateur de Hecke TpT_{p}, J. Amer. Math. Soc. 10 (1997), no. 1, 75–102. MR 1396897
  • [ST21] Eric Stade and Tien Trinh, Recurrence relations for Mellin transforms of GL(n,)GL(n,\mathbb{R}) Whittaker functions, J. Funct. Anal. 280 (2021), no. 2, Paper No. 108808, 25. MR 4166593
  • [Sta01] Eric Stade, Mellin transforms of GL(n,){\rm GL}(n,\mathbb{R}) Whittaker functions, Amer. J. Math. 123 (2001), no. 1, 121–161. MR 1827280
  • [Zha22] Qiao Zhang, Lower Bounds for Rankin-Selberg LL-functions on the Edge of the Critical Strip, arXiv e-prints (2022), arXiv:2211.05747.
  • [Zho14] Fan Zhou, Weighted Sato-Tate vertical distribution of the Satake parameter of Maass forms on PGL(N){\rm PGL}(N), Ramanujan J. 35 (2014), no. 3, 405–425. MR 3274875